0% found this document useful (1 vote)
290 views

Master Sat Math Book

This document provides an overview of working with algebraic expressions on the SAT Math test. It discusses the importance of algebra, noting that about 36% of SAT Math questions fall under the category of "Heart of Algebra". It emphasizes four core skills: creating and solving linear equations and inequalities, graphing, and working with systems of equations. The document then provides a step-by-step approach to solving word problems algebraically, including identifying relevant quantities, expressing them algebraically, translating the problem into equations, and analyzing and solving the equations. Finally, it discusses the order of operations and laws of arithmetic for simplifying complex expressions.

Uploaded by

Duy Bui Hoang
Copyright
© © All Rights Reserved
We take content rights seriously. If you suspect this is your content, claim it here.
Available Formats
Download as PDF, TXT or read online on Scribd
0% found this document useful (1 vote)
290 views

Master Sat Math Book

This document provides an overview of working with algebraic expressions on the SAT Math test. It discusses the importance of algebra, noting that about 36% of SAT Math questions fall under the category of "Heart of Algebra". It emphasizes four core skills: creating and solving linear equations and inequalities, graphing, and working with systems of equations. The document then provides a step-by-step approach to solving word problems algebraically, including identifying relevant quantities, expressing them algebraically, translating the problem into equations, and analyzing and solving the equations. Finally, it discusses the order of operations and laws of arithmetic for simplifying complex expressions.

Uploaded by

Duy Bui Hoang
Copyright
© © All Rights Reserved
We take content rights seriously. If you suspect this is your content, claim it here.
Available Formats
Download as PDF, TXT or read online on Scribd
You are on page 1/ 397

TH E SAT MATH TEST:

TH E H EART O F ALG E B RA

l. Wo rki n g with Expressions 281

2. Worki n g with Li n e a r Equations 296

3. Working with I neq u a l ities a n d Abso l u te Va l ues 3 07

4. Worki n g with Linea r System s 314

2 79
280 McGraw-Hill Education: SAT

The SAT Math:


Heart of Algebra
Why is algebra so important on the SAT Math test?

About 36% (2 1 out of 58) of the SAT Math questions fall under the category called the "Heart of Algebra."
Questions in this category test your ability to
analyze, fluently solve, and create linear equations, inequalities, [and} systems of equations using multiple
techniques.
These questions will also assess your skill in
interpreting the interplay between graphical and algebraic representations [and} solving as a process of
reasoning.
The specific topics i nclude
creating and solving linear equations i n one and two variables
graphing and interpreting linear equations
creating, interpreting, and solving linear systems
graphing and solving i nequalities and systems of inequalities
i nterpreting and solving algebraic word problems

Why are these skills important?

Algebra is a n essential tool of quantitative analysis not only in math but also in subjects l i ke engineering,
the physical sciences, and economics. When describing the relationships between or among different quanti­
ties, or exploring the nature of unknown quantities, algebra provides essential tools for analyzing and solv­
ing problems. Most colleges consider fluency in algebra to be a vital prerequisite to a college-level liberal arts
curriculu m .

Sound intimidating? It's not.

I f you take the time to master the four core skills presented in these 13 lessons, you will gain the knowledge and
practice you need to master even the toughest SAT Math "Heart of Algebra" questions .
CHAPTER 7 / THE SAT MATH TEST: THE HEART OF ALGEBRA 281

Ski l l 1 : Worki ng with Expressions


Lesson 1 : U s i ng a lgebra ic expressions

To solve tough SAT math problems, you must be flu­ Th is may seem like a lot, but as we will see, keeping track
ent i n defining, manipulating, and a nalyzing algebraic of them is quite manageable.
expressions.
Step 2. Express: The problem gives us enough informa­
Corrine drives to her office at an average speed of 50 tion to express all six quantities in terms of only two
miles per hour. When she returns home by the same "unknowns." I f d is the distance, in miles, from her home
route, the traffic is lighter and she averages 60 miles to her office, and t is the time, in hours, it takes her to get
per hour. I f her trip home is 10 minutes shorter than home from the office, then we can express our six quan­
her trip to her office, what is the distance, in miles, t ities, respectively, as
from Corrine's home to her office?
To Office From Office

I I I I I
50 mph 60 mph
d m iles d miles
(Medium-hard) Why does everyone hate "word prob ­ t + 1/6 hours (10 minutes = 1 /6) t hours
lems" like this one? For most of us, the problem is that
the equations aren't "set up" for us-we have to set them Step 3. Translate: To translate the facts of this problem
up ourselves, which can be a pain in the neck. But we into equations, we must know the formula distance =
can make these problems much easier by breaking them average speed x time. Applying this to each trip gives us
down i nto clear steps.
To Office From Office
Key Steps to Solving Tough Algebraic Problems d = 50(t + 1 /6) d = 60(t)
Solving tough problems in mathematics and sci­
ence frequently i nvolves four essential steps : Step 4. Analyze and Solve: We have now reduced the
problem to a "two by two system," that is, two equations
1 . identify the relevant quantities in the situation with two unknowns. Since the number of equations
2. express those quantities with algebraic equals the number of unknowns, we should be able to
expressions solve for those un knowns. (In Lessons 12 and 13, we
3. translate the fa cts of the problem situation will review these concepts and techn iques.) Since the
into equations i nvolving those expressions un known d is isolated in both equations, substitution is
4. a nalyze and solve those equations simple:

1 . Substitute for d: 50(t + 1 /6) = 60(t)


Step I. Identify: I n this problem, there are six relevant 2. Distribute: 50t + 50/6 = 60t
quantities:
3. Subtract 50t: 50/6 = lOt
the speed from home to work 4. Divide by 10: 5/6 = t
the distance from home to work
the time it takes to get from home to work Since t represents the time it took Corrine to return
the speed from work to home home, in hours, this means it took her 5/6 hours (or
the distance from work to home 50 mi nutes) to get from her office to her home, and
the time it takes to get from work to home 5/6 hour + 1 /6 hour = 1 hour to get to her office from
home. But remember, the question asks for the distance
from her home to her office, which we can find by substi­
tuting into either of our equations:

50(5/6 + 1 /6) or 60(5/6) = 50 miles


282 McGraw-Hill Education: SAT

Lesson 2: The Laws of Arithmetic


When expressing or simplifying a quantity, you fre­
quently have many options. For instance, the expression Step 3: MD (multiplication and division, from
4x2 - 1 2x can also b e expressed as 4x(x - 3) . Similarly, left to right)
3 . 2 can be expressed as 1 6/5 or 3 Vsor 32/10. Which way
Next, we do a ny multiplication inside the
is b etter? It depends on what you want to do with the
parentheses:
expression. Different forms of an expression can reveal
different characteristics of that quantity or the equation (3 + 1 2 ) �
____ +
i n which it appears. To gain fluency in expressing quanti­ 8 2
ties, you must understand the Laws of Arithmetic.
Step 4: AS (addition and subtraction, from left to
3+6x2 5-1 1 right)
What is the value of --- +
2 x l3 ·
--

2x4
Now we do any addition and subtraction
left in the parentheses:

I I I I I 15 4
-+-
8 2
Once all the "grouped" operations are com­
To simplify complex expressions, you must pleted, we run through the order of opera­
know the Order of Operations: tions once again to finish up. E xponents or
roots? No. Multiplication or division? Yes:
PG-ER-MD-AS
1 . 875 + 2
Step 1: PG (parentheses and other grouping
Addition or subtraction? Yes : 1 .875 + 2 =
symbols, from innermost to outermost and left
3.875
to right)
Since this expression contains no parentheses,
we don't have to worry about "grouped" oper­
ations, right? Wrong! Remember that fraction
What is the sum of the first 100 positive i ntegers?
bars and radicals are "grouping symbols"
just like parentheses are.

In other words, we can think of this expres­


sion as
(3 + 6 x 2) (5 - l ) (Hard) Here, following the order of operations would be,
_____ + ____
(2 x 4 ) (2 x l 3 ) shall we say, less than convenient: it would require 99
computations. Even with a calculator, it would b e a pain.
If a set of parentheses contains only one But here is a much simpler method:
operation, then we simply do that operation:
Original expression: 1 + 2 + 3 + 4+ · ·+ 97 +
·

(3 + 6 x 2) 4 98 + 99 + 100
+
(2 x l 3 )
---- ---

8 Rearrange and regroup: (1 + 100) + (2 + 99) + (3 + 98) +


. . . + (50 + 5 1)
If it contains more than one operation, then Simplify: (101) + (101) + (101) + · · ·

we must move on to the next step. + (101)


Step 2: ER (exponents and roots, from innermost Since we have 50 pairs, this equals: 50(101)
to outermost and left to right) Simplify: 5,050
Do any of the parentheses contain expo­
This gives us exactly the same result as the order of
nents or roots? Yes, so we must perform that
operations would give, but with just a few simple calcu­
operation next:
lations. How did we do it? By using three more laws of
(3 + 6 x 2) - 4- arithmetic: the commutative law of addition, the asso­
+
8 (2 x 1) ciative law of addition, and the distributive law of mul­
tiplication over addition.
CHAPTER 7 I THE SAT MATH TEST: THE HEART OF ALGEBRA 283

Use the Laws of Arithmetic to simplify expres­ Don't "over-distribute."


sions or reveal their properties.
Were you tempted to choose (A), (B), or (C) in the
The Commutative Law of Addition question above? I f so, you are not alone. You are
simply the victim of one of the most common mis­
When adding, order doesn' t matter.
takes i n algebra: over-distribution. I t comes from
e.g., 3 + 8 + 17 + 12 = 3 + 17 + 1 2 + 8 a misinterpretation of the L aw of Distribution.
The correct l aw is
The Commutative Law of Multiplication
When multiplying by a grouped sum, you
When multiplying, order doesn ' t matter. don 't have to do the grouped sum first; you
can multiply first, as long as you distribu te
e.g., 2 x 16 x 50 x 3=3 x 16 x 50 x 2
the multiplication over the entire sum.
The Associative Law of Addition It is not
When adding, grouping doesn' t matter. If something is outside parentheses, just
e.g., 1 + 10 0 + 2 + 99 + 3 + 98 + + 50 + 51 = · · ·
bring it inside and distribute.
(1 + 100) + (2 + 99) + (3 + 98) + . . + (50 + 51) .
Look at these examples of "over- distribution"
and verify that they are incorrect:
The Associative Law of Multiplication
3 (2 x 5) is not equal to (3 x 2) + (3 x 5) or
When multiplying, grouping doesn' t (3 x 2) x (3 x 5)
matter.
(2 + 3)2 is not e qual to 2 2 + 32
e.g., 1 x 2 x 3 x 4 x 5 = (1 x 2 x 3) x (4 x 5)

The Distributive Law of Multiplication over


Addition I f x ,,., 0, which of the following equals
When multiplying by a grouped sum, 3x 2 + 6x + 9x 2
?
you don ' t have to do the grouped sum 3x
first; you can multiply first, as long as 1
you distribute the multiplication over the A) 2x + -
2
entire sum. 1
B) 4x + -
e.g., 5 (20 + 7) = 5 x 20 + 5 x 7 = 100 + 35 = 135 2
C) 2x2 + 2
D) 4x + 2

(Medium) 3x 2 + 6x + 9x 2
Which of the following is equivalent to 3(34 x 53)? 3x
3x 2 + 9x 2 + 6x
[No calculator] Com mutative Law of Addition:
3x
A) 3 (34) x 3(53) 2 + 9x 2 ) + 6x
Associative Law of Addition : (3x
B) 94 + 153 3x
C) 94 x 1 53 (3 + 9)x 2 + 6x 1 2x 2 + 6x
Distribut ive Law:
D) 35 x 53 3x 3x
Division by a number is
Before making your choice, check the laws of arithme­ 1
multiplication by its reciprocal: - ( 1 2x 2 + 6x)
tic; don't make up your own laws. Which laws of arithme­ 3x
tic can we use? Since the expression is a product, we can Distributive Law: 1 2x 2 6x
use the commutative law of multiplication and jumble + - = 4x + 2
--

3x 3x
up the terms, or the associative law of multiplication
and regroup the terms any way we want (or not at all). So the correct answer is (D). Look at each step care­
Using the associative law gives us fully and notice how each one uses a particular Law
of Arithmetic. I n particular, notice that the "combin­
3(34 x 53) = (3 x 34) x 53 = 35 x 53 Therefore, the ing of like terms" i n steps 1-3 is really a n example of
correct answer is (D).
284 McGraw-Hill Education: SAT

commuti ng, assoc1atmg, and (un)distributing. Even 1. Add 4x: x2 + 4x + 4 = 0


more interesting, notice that steps 4-5 show that divi­ 2. Since 4x = 2x + 2x: x2 + 2x + 2x + 4 = 0
sion distributes just like multiplication does.
Step 2 might seem a bit mysterious. Why did we write
4x as 2x + 2x? Here we are using the Product-Sum Method
You can also distribute division over addition for factoring quadratics, which is explained in a bit more
just as you can distribute multiplication. detail in Chapter 9, Lesson 4. For now, though, just notice
that each step follows a particular Law of Arithmetic.
25a + 5ab 25a 5ab 5a
e.g., =-+-=-+a 3 . Associative Law of Addition: (x2 + 2x) + (2x + 4) = 0
5b 5b 5b b
4 . Distributive Law: x(x + 2) + 2(x + 2) = 0
5. Distribut ive Law: (x + 2)(x + 2) = 0
How many distinct values of x are solutions to the If the product of two numbers is 0, then one of those
equation x2 + 4 = - 4x? numbers must be 0 . (This is the Zero Product Property. )
A) none Therefore x + 2 = 0 and so x = - 2 . Since the other factor
is the same, we only get one solution to this equation, and
B) one the answer is ( B ) .
C) two To check t h e equation i n step 5, we can FOIL the prod­
D) three uct of binomials on the left side to make sure we get the
same expression we had back i n step 1: (x + 2)(x + 2) =
(Medium) You might recognize that this equation is a x2 + 4x + 4, which i s precisely the expression we started
quadratic equation (which we will discuss i n much with i n step 1 , confirming that our work is correct.
more detail in Chapter 9) and remember that such equa­ This means that the factoring process in steps 2-5 can
tions usually h ave two distinct solutions, but not always, be thought of as un-FOI Ling. We will look at this method
so we must look at this equation more carefully. of factoring more carefully i n Chapter 9.

Make sure you know how to FOIL and un-FOIL.

FOILing is simply the shortcut for multiplying two binomials, which requires applying the distributive law
twice. For example:
(x + 4)(x - 5)
F ( product of the two "first" terms): X X X = X2
0 ( product of the two "outside" terms): x x - 5 = - 5x
I (product of the two "inside" terms) : 4 X X = 4X
L (product o f t h e two "last" terms): 4 x -5 = -20
F + O + l + L: x2 + - 5x + 4x + -20 = x2 - x - 20
CHAPTER 7 / THE SAT MATH TEST: THE HEART OF ALGEBRA 285

• ..
( 1 - ( 1 - ( 1 - 2))) - ( 1 - (1 - (1 - 3))) = The product of x and y is 36. I f both x and y are
integers, then what is the least possible value

I I I I I of x - y?
A) -37
B) -36
• C) -35
When 14 is subtracted from 6 times a number, 40 is
D) - 9
left . What is half the number?

I I I I I ..
I f a factory can manufacture b computer screens i n
n days a t a cost o f e dollars p e r screen, t h e n which of
.. the following represents the total cost, in dollars, of
Four consecutive even numbers have a sum of 76. the computer screens that can be manufactured, at
What is the greatest of these numbers? that rate, in m days?
bem
I I I I I A)
n
bmn
B)
e
• me
C)
5x bn
I f - + 3 = 7, then lOx + 12 =
2 be
D)

I I I I
mn

1111
Iii Which of the following is equivalent to
What number decreased by 7 equals the opposite of 5x(2x x 3) - 5x2 for all real values of x?
five times the number?
A) 5x2 + 1 5x

I I I I I
B) 25x2
C) 5x2 - 1 5x
D) 1 0x2 x 1 5x - 5x2

I f 5d + 1 2 = 24, then 5d - 1 2 =

I I I I I The symbol 0 represents one of the fu ndamental
operators: +, - , x , or +. I f (x 0 y) x (y 0 x) = 1 for
all posit ive values of x and y, then 0 can represent
• A) +
2 2
I f _L = y 2 , then y + 5 = B) x
5

I I I
C)
D)
286 McGraw-Hill Education: SAT

Exerc ise Set 1 (Ca lculator)

Ill Ill
The difference of two numbers is 4 and their sum For all real numbers x and y, 4x(x) - 3xy(2x) =

is 14. What is their product?


A) 1 2x2y(x - 2y)

I I I I I B) 2x2(2 - 3y)
C) 2x2(2 + 3y)
D ) 4xy(x - 3y)
Ill
If x + y - 1 = 1 - ( 1 - x), what is the value of y ?
Ill
I I I I If a = 60(99)99 + 30(99)99, b = 99 1 00, and c = 90(90)99,
then which of the following expresses the correct
ordering of a , b, and c?
ID A) c <b<a
If 3x2 + 2x = 40, then 1 5x2 + lOx = B) b < c < a

I I I I
C) a < b < c
D) c < a < b

ID •
Ellen is currently twice as old as Maria, but in
Which of the following statements must b e true for
6 years, Maria will b e 2/3 as old as Ellen. How old is
all values of x, y, and z?
Ellen now?
I . (x + y) + z = (z + y) + x
I I . (x - y) - z = (z - y) - x
I I I . (x -7 y) -7 z = (z -7 y) -7 x

• A) I only
If 2x - 2y = 5 and x + y = 6, what is the value B) I and I I only
of x2 - y2? C ) I and I I I only
D ) I I and I I I only
I I I I
Ill
Ill Carlos began with twice as much money as David
On a typical day, a restaurant sells n grilled cheese sand­ had. After Carlos gave $ 1 2 to David, Carlos still had
wiches for p dollars each. Today, however, the manager $10 more than David. How much money did they
reduced the price of grilled cheese sandwiches by have combined at the start?
30% and as a result sold 50% more of them than usual.
Which of the following represents the revenue for A) $34
today's grilled cheese sandwich sales, in dollars? B) $68
A) O . S np - 0.3 C ) $ 102
B) l .OSnp D ) $108
C ) l .20np
D) l . SOnp
CHAPTER 7 I THE SAT MATH TEST: THE HEART OF ALGEBRA 287

EXERCISE SET 1 ANSWER KEY

No Calculator
1. 1 ( 1 - ( 1 - ( 1 - 2))) - (1 - (1 - (1 - 3))) b screens c dollars bcm
m d ays x x ---

Parentheses: (1 - (1 - (- 1))) - (1 - (1 - (-2))) n days l screen n


Next parentheses: (1 - (2)) - (1 - (3))
10. B Original expression: 5x(2x x 3) - 5x2
Next parentheses: (-1) - (-2)
-1 + 2 = 1 Parentheses: 5x(6x) - 5x2
Subtract:
Multiply: 30x2 - 5x2
2 . 9/2 or 4.5 6x - 14 = 40 Subtract: 25x2
6x = 54 Remember: The Law of Distribution does not apply in
Add 14:
Divide by 6: X=9 the first step, because the grouped expression doesn't
1 9 include addition or subtraction.
Multiply by ..!_ : -X = -
2 2 2
1 1 . D The simplest approach is perhaps to choose
3 . 22 Let n be the least of these numbers. The sum of simple values for x and y, like 2 and 3, and see which
four consecutive even numbers is therefore n + (n + 2) + operator yields a true equation. Since (2 -:- 3) x (3 -:- 2) = l,
(n + 4) + (n + 6) = 76. the answer is (D).

Simplify: 4n + 1 2 = 76
Subtract 1 2 : 4n = 64
Divide by 4: n = 16 Calcu lator
Therefore the largest of these nu mbers is 16 + 6 = 22.
12. 45 a- b=4
5 a + b = 14
4. 2 8 -x + 3 = 7
2 Add equations: 2a = 18
Multiply by 4: l Ox + 1 2 = 28 Divide by 2: a=9
Substitute a = 9: 9 + b = 14
5 . 7/6 or 1 . 1 6 o r 1 . 1 7 x - 7 = - 5x Subtract 9 : h=5
Subtract x: -7 = - 6x Evaluate ab: ab = 9 x 5 = 45

Divide by - 6 : 7
-=X
13. 1 x + y - 1 = 1 - ( l - x)
6 Distribute: x+y - l = l - l +x
6. 0 5d + 1 2 = 24 Subtract x: y- 1=1 - 1
Simplify: y- 1=0
Subtract 24: 5d - 1 2 = 0 Add 1 : y= l
2y 2 2
7. 5 -=y 14. 200 3x2 + 2x = 40
5
M ultiply by 5: 1 5x2 + l Ox = 200
3y 2
Subtract y2: -- = 0
5
Multiply by - 5/3: y2 = 0 1 5 . 12 Let e = Ellen's current age and m = Maria's
Take square root: y= O c urrent age.
Add 5: y+5=5 El len is twice as old as Maria: e = 2m
I n 6 years, Maria will be 2/3 as 2
8. C If.xy = 36 and x and y are integers, then x and y are old as Ellen:
m + 6 = - (e + 6)
3
both factors of 36. In order to minimize the value of x - y, Substitute e = 2m: 2
we must find the greatest separation between x and y. The m + 6 = - (2m + 6)
3
greatest separation between a fa ctor pair is 1 - 36 = - 35. Multiply by 3: 3m + 1 8 = 2 (2 m + 6)
Distribute: 3m + 18 = 4m + 1 2
9. A We should regard this as a "conversion" prob ­ Subtract 3m and 1 2 : 6=m
lem from m days into a corresponding number of dollars. Therefore e = 2m = 2(6) = 1 2 .
288 McGraw-Hill Education: SAT

just too large. However, comparing these numbers is


16. 1 5 First equation: 2x - 2y = 5
straightforward if we can express them i n a common
Divide by 2 : x - y = 2.5 format.
Second equation: x+y=6
a = 60(99)99 + 30(99)99 = 9 0 (99)99
Multiply: (x - y)(x + y) = x2 - y2 = (2 . 5)(6) = 1 5
b = 99 100 = 9 9 (9 9)99
Alternately, w e could solve t h e system using either c = 90(90)99 = 9 0 (9 0)99
substitution or linear combination and get x = 4.25 and
y = 1 .75, and evaluate x2 - y2 = (4.25)2 - (1 .75)2 = 18.0625 -
20. A Only statement I is true, by the Commutative
3.0625 = 15.
and Associative Laws of Addition. Choosing simple val­
ues like x = 1 , y = 2, and z = 3 will demonstrate that state­
17. B The revenue is equal to the number of items ments I I and III do not yield true equations .
sold times the price per item . I f the restaurant typically
sells n sandwiches per day, but today sold 50% more, it 2 1 . C Let x = the number of dollars David had to start.
sold l . 5n sandwiches. I f the price p was reduced 30%, I f Carlos started with twice as much money as David,
today's price i s 0.70p. Therefore, the total revenue is then Carlos started with 2x dollars. A fter Carlos gave
( l . 5 n)(0.70p) = l .05np. David $ 1 2 , Carlos had 2x - 12 dollars and D avid had x +
1 2 dollars. I f Carlos still had $ 1 0 more than D avid, then
18. B 4x(x) - 3xy(2x) 2x - 12 = 10 + x + 1 2
Multiply: 4x2 - 6x2y Simplify: 2x - 12 = x + 22
Largest common factor: 2x2(2 - 3y) Add 12: 2x = x + 3 4
Subtract x: X = 34
1 9 . D Although a calculator i s permitted for this ques­ Therefore, David started with $34 and Carlos started with
tion, most calculators will give an "overflow error" when 2 ($34) = $68, so t hey had $34 + $68 = $102 combined to
trying to calculate numbers like 991 00 , because they're start.
CHAPTER 7 I THE SAT MATH TEST: THE HEART OF ALGEBRA 289

Lesson 3: Simplifying exp ressions and operations

I f x and y a r e positive numbers a n d 3 x - 2y = 7, what the expression we are asked to evaluate, it makes sense
is the value of 2 Y + 7 ? to replace it with 3x:
6x ·
2y + 7
6x
A) - 3x
7 Substitute 3x for 2y + 7:
6x
1
B) 1
3 Simplify:
2
2
C)
7
1 When a question asks you to analyze a complex
D)
2 expression, don't be intimidated. Look for sim­
ple relationships that a llow you to simplify them
(Easy) Working with algebraic equations doesn't always using techniques like the L aw of Substitution.
mean "solving for x. " Notice that this particular ques­
tion doesn't ask for the values of x or y, but rather for the
value of a more complicated expression. This may seem Increasing a positive number x by 25% and then
h arder, but it's actually pretty simple if you understand decreasing the result by 50% is equivalent to divid­
the Law of Substitution ing x by what number?
A) 1 .333
B) 1 . 5
The Law of Substitution
C ) 1 .6
If two expressions are equal, then you may substi­ D) 1 .625
tute one for the other at any point in the problem.
(Medium) Increasing a quantity by 25% is equivalent to
multiplying it by 1 .25, because the final amount is 1 25%
of the original amount (Chapter 8, Lesson 8). Decreasing
How does this help us here? Notice that if we simply a quantity by 50% is equivalent to multiplying it by .5,
add 2y to both sides of the equation, w e g et because the final amount is 50% of the original amount.
Therefore, performing both changes is equivalent to mul­
3x - 2y = 7 tiplying by 1 .25 x 0.50, or 0.625, which is equal to 5/8. But
the question asks us for the equivalent division. Here, we
Add 2y: 3x = 2y + 7 need to remember a simple rule: multiplying by a num­
ber is equivalent to dividing by its reciprocal. Therefore,
Therefore, by the Law of Substitution, we can substi­ multiplying by 5/8 is the same as dividing by 8/5, which is
tute 3x for 2y + 7 or vice-versa. Since 2y + 7 appears in 1 .6. Therefore, the correct answer is (C).
290 McGraw-Hill Education: SAT

Every operation can be expressed in terms of its inverse operation.

Adding (x) is equivalent to subtracting (-x) .

( �) .
Subtracting (x) is equivalent to adding ( -x) .
Multiplying by (x) is equivalent to dividing by

Dividing by [�] is equivalent to multiplyi ng by ( �) .

Taking t h e n t h root o f a number is equivalent to raising the number to the ( �) th power.


And here are two more handy equivalences:

I ncreasing a number by x% is equivalent to multiplying that number by (1 + 1 00 )� .

multiplying that number by (1 -


Decreasing a number by x% is equivalent to
100 )� .

m2 - n2 9
If = - what is the value of m + n? (Easy) When a problem includes a complicated expres­
2m - 2n 2
sion, we should try to simplify it, but always keep an eye

I I I I I
on what the question is asking. In this case, simplifying
to find the value of m + n requires knowing some factor­
ing identities.

Useful factoring identities

The difference of squares equals the product of conjugates: x2 - y2 = (x + y)(x - y)


Perfect square polynomials: x2 + 2ax + a2 = (x + a)(x + a)
x2 - 2ax + a2 = (x - a)(x - a)

The first of these identities helps us m2 - n2 9


factor our numerator: 2m - 2n 2

Factor numerator and (m + n)(m - n) 9


denom inator: 2(m - n) 2
Ca ncel common factors: (m + n )
---
9
2 2
Mu ltiply by 2: m+n=9
CHAPTER 7 / THE SAT MATH TEST: THE HEART OF ALGEBRA 291

Lesson 4: Using conversion as a problem-solvin g tool

Niko is 27 inches shorter than his father, who is I f a factory can manufacture b computer screens in
5 feet 10 i nches tall. How tall is Niko? (1 foot = n days at a cost of c dollars per screen, then which of
1 2 i nches) the following represents the total cost, i n dollars, of
the computer screens that can b e manufactured, at
A) 3 feet 4 inches
that rate, in m days?
B) 3 feet 6 inches
bcm
C) 3 feet 7 inches A)
n
D) 3 feet 1 0 i nches bmn
B)
(Easy) Solving this problem requires unit conversions . c
To convert i nches to feet, we multiply by the conver­ me
C)
sion factor (1 foot/ 1 2 inches). To convert feet to i nches, bn
we multiply by its reciprocal ( 1 2 i nches/ l foot). If Niko's be
D)
father is 5 feet 10 i nches tall, he is 5 feet x ( 1 2 inches/ mn
1 foot) + 10 i nches = 70 i nches tall. I f Niko is 27 inches
shorter, he is 70 - 27 = 43 i nches tall, which is equivalent (Medium) This problem, from the previous exercise set, can
to 43 i nches x ( 1 foot/ 1 2 i nches) = 3 7/ 1 2 feet, or 3 feet be solved in several different ways. One method is to simply
7 i nches, so the correct answer is (C). choose values for the unknowns and turn the problem into
an arithmetic problem instead of an algebra problem. But
here we will look at it as a conversion problem.
Conversion factors as problem-solving tools We can think of this problem as being a "conversion"
A conversion factor is simply a fraction in which from a quantity of days to an equivalent quantity of dol­
the quantities in the numerator and the denomi­ lars. We are given that this factory is working for m days,
nator represent equal quantities. Sometimes the so we write this quantity down, including the units, and
equivalence is universal-for instance, 1 pound we multiply by the conversion factors until we get dollars:
is always equal to 16 ounces-and sometimes it is
problem-specific-for instance when a machine b screens c dollars bcm
3
pump waters at a rate gallons per hour, 1 hour m d ays x x = -- d o 11 ars
n days 1 screen n
of pumping is "equal" to 3 gallons b eing pumped.
So the correct answer is (A).
292 McGraw-Hill Education: SAT

• •
I f bag A weighs 4 pounds 5 ounces and bag B weighs (x - 9)(x - a) = x2 - 4ax + b
6 pounds 2 ounces, how much heavier, in ounces, is
I n the equation above, a and b are constants. I f the
bag B than bag A? (1 pound = 16 ounces)
equation is true for all values of x, what is the value

I I I I I
of b?
A) -27
B) - 1 2

C) 12
3a + b 7 . a
If-- = -, what 1s the value of - ? D) 27
b 5 b

I I I •
If � + 7_ = l , what is the value of x?
..
x 5
2x A) - 2 5-

I f x - 2y = 10 and x ""' 0, what i s the value of --


? 2
y+5 B) -7
24
I I I
C)
7
D) 7
• 5
I f a - b = 4 and a2 - b2 = 3, what is the value of a + b? 1D1

I I I
(p + 2)2 = (p - 5)2
The equation above is true for which of the following
values of p?
.. A) -2 and 5
I f 6 gricks a re equivalent to 5 merts, then 2 merts are
B) 2 and -5
equivalent to how many gricks?
C) 1 . 5 only

I I I D) 5 only

• •
3x . .
If the fu nction {x} is defined by the equation {x} = If --- = 2 for all pos1t1ve values of m and n, then
m - nx
(1 - x)2, what is the value of {{4}}?
which of the following is equal to x?

I I I A) 2m - 2n
3
• B) 2m - 3
2n
a+b a+c . b 3 + 2n
I f -- = 3 and = 5 , what 1s the value of - ?
-- C)
b c c 2m
2m
D)
3 + 2n
CHAPTER 7 / THE SAT MATH TEST: THE HEART OF ALGEBRA 293

Exerc ise Set 2 (Ca lcu lator)

IFI
Let m b e a positive real number. Increasing m by Subtracting 3 from a number and then multiplying
60% then decreasing the result by 50% is equivalent this result by 4 is equivalent to multiplying the origi­
to dividing m by what number? nal number by 4 and then subtracting what number?

I I I I I I I I I I
Ill I n a poker game, a blue chip is worth 2 dollars more
What is the sum of the first 50 positive even integers? than a red chip, and a red chip is worth 2 dollars
more than a green chip. I f 5 green chips are worth m
dollars, then which of the following represents the

I I I I I value, in dollars, of 10 blue chips and 5 red chips?


A) 50 + 3m
B) 18 + 60m
ID
C) 40 + 3m
Three years ago, Nora was half as old as Mary is
now. I f Mary is four years older than Nora, how old D) 28 + 20m
is Mary now?

I I I I I
A train travels at a n average speed of 50 miles per
hour for the first 100 miles of a 200-mile trip, and at
an average of 75 miles per hour for final 100 miles.
What is the train's average speed for the entire trip?
A) 58.5 mph
I f 2/3 of the seats at a football stadium were filled
B) 60.0 mph
at the beginning of the game, and at halftime 1,000
spectators left, leaving 3/7 of the seats filled, what is C) 62.5 mph
the total number of seats i n the stadium? D) 63.5 mph

m
Which of the following is equivalent to 3m(m2 x 2 m)
for all real values of m?
A) 3m2 + 6m
I f three candy bars and two gumdrops cost $2.20, B) 3m2 x 6m
and four candy bars and two gumdrops cost $2.80,
C) 3m3 x 6m2
what is the cost, in dollars, of one gumdrop?
D) 6m4

I I I I I m
If the cost ofliving in a certain city increased by 20% in
the 10 years from 1980 to 1990, and i ncreased by 50%
DI in the 20 years from 1980 to 2000, what was the per­
x 2 - 2x + 1 cent increase in the cost of living from 1990 to 2000?
If = - 3, what is t h e value of x - 1?
2 - 2x
A) 1 5%
B) 20%

I I I I I C)
D)
25%
30%
294 McGraw-Hill Education: SAT

EXERCISE SET 2 ANSWER KEY

Reciprocate: b
No Calcu lator
a 2
1. 29 4 pounds 5 ounces = 4(16) + 5 = 69 ounces, Given equation: a+c
-- = 5
and 6 pounds 2 ounces = 6 ( 1 6) + 2 = 98 ounces. Therefore, c
bag B weighs 98 - 69 = 29 ounces more. Distribute d ivision: a
-+1=5
c
a
2. 2 / 1 5 or . 1 33 3a + b
--
7
- Subtract 1 : -=4
b 5 c
3a b 7 Multiply:
D istribute division: -+-=-
b b 5
8. D Given: (x - 9)(x - a ) = x2 - 4ax + b
Si mplify: 3a 7
-+l=-
b 5 FOIL: x2 - ax - 9x + 9a = x2 - 4ax + b
3a 2 Simplify: x2 - (a + 9)x + 9a = x2 - 4ax + b
Subtract 1 :
b 5 I f t h i s equation i s true for all x, then
the coefficients of corresponding terms
D ivide by 3 : a 2
must be equal, so a + 9 = 4a
b 15
Subtract a: 9 = 3a
3. 4 Expression to be evaluated: 2x
y+5 Divide by 3 : 3=a
Given equation: x - 2y = 1 0 Therefore b 9a = 9 (3) = 27.
=

Add 2y: X = 2y + 1 0 5 7
9. A Given equation: -+-=l
Substitute x = 2y + 1 0 : 2(2y + 1 0) x 5
y+5 Multiply by 5x: 25 + 7x = 5x
4y + 20 Subtract 7x: 25 = -2x
Simplify:
y+5 D ivide by -2: 25
-- = X
4 (y + 5) 2
Factor and simpl i fy: =4
y+5 10. C Given equation : (p + 2) 2 = (p - 5)2
4. 314 or . 75 a2 - b2 = 3 FOIL: p2 + 4p + 4 = p2 - lOp + 25
Factor: (a - b)(a + b) = 3
Subtract p2: 4p + 4 = - l Op + 25
Substitute a - b = 4: 4(a + b) = 3
3 Add l Op: l4p + 4 = 25
Divide by 4: a+h=-
4 Subtract 4 : l4p = 21

or 6 gricks 1 2 D ivide b y 1 4 : p = 1.5


5. 1 2/5 2.4 2 merts x = - gricks
5 merts 5
1 1. D Given equation: �=2
6. 64 {4} (1 - 4)2 = (-3)2 = 9
= m - nx
Multiply by m - nx: 3x = 2(m - nx)
{{4}} = ( 1 - {4})2 = (1 - 9)2 (-8)2 = 64
=

Distribute: 3x = 2 m - 2nx
7. 2 Given equation: a+b
=3 Add 2nx: 3x + 2nx = 2m
b
a Factor out x: x(3 + 2n) = 2m
D i stribute d ivision: -+1=3
b 2m
Divide by 3 + 2n: X = --
a 3 + 2n
Subtract 1 : -=2
b
CHAPTER 7 I THE SAT MATH TEST: THE HEART OF ALGEBRA 295

Calculator 17. 6 x 2 - 2x + l _
_ 3
2 - 2x
12. 1 .2 5 I ncreasing a number by 60% is equivalent
to multiplying it by 1 .60, and decreasing a number by
50% is equivalent to multiplying it by 0.50. Therefore, Factor: (x - l)(x - l)
= -3
performing both changes i n succession i s equivalent to 2(1 - x)
multiplying by 1 . 60 x 0.50 = 0.80. Multiplying by 0.80 is
Multiply by - 1 : (x - l)(x - 1)
equivalent to dividing by its reciprocal: 1 /(0.80) = 1 .25. -3
2(x - l)
1 3 . 2 , 550 The sum of the first 50 positive even inte­ x-l
gers is 2 + 4 + 6 + 8 + · + 100. As with the example is
· ·
Simplify: -- =3
2
Lesson 2, these numbers can be regrouped into 25 pairs Multiply by 2: x- 1 =6
of numbers each of which has a sum of 2 + 100 = 102.
Therefore, their sum is 25(102) = 2 , 550.
18. 1 2 We can j ust choose a number to work with,
like 10. If we subtract 3 from this number then multi­
14. 1 4 Let n = Nora's age now, and m = Mary's age ply the result by 4, we get 4(10 - 3) = 28. If we multiply
now. If 3 years ago, Nora was half as old it by 4 and then subtract a mystery number, we get 4(10) -
as M a ry is now: 1 x = 40 - x. 28 = 40 - x
n - 3 = -m
2
If Mary is 4 years older than Nora: m=4+n Subtract 40: - 1 2 = -x

Subtract 4: m-4=n Multiply by - 1 : 12 = X

Substitute n = m - 4: 1
m - 4 - 3 = -m 1 9 . A If 5 green chips are worth m dollars, then each
2 green chip is worth m/5 dollars. If a red chip is worth
Simplify: 1 2 dollars more than a green chip, then each red chip is
m - 7 = -m
2 worth m/5 + 2 dollars. If each blue chip i s wort h 2 dol­
Multiply by 2: 2m - 14 = m lars more than a red chip, then each blue chip is wort h
Subtract m and add 14: m = 14 m/5 + 4 dollars. Therefore, 1 0 blue chips a n d 5 r e d chips
are worth lO(m/5 + 4) + 5 (m/5 + 2) = 2m + 40 + m +
10 = 3m + 50 dollars.
15. 4, 200 Let x = the total number of seats i n the

s t a d iu m .
2 3 20. B The average s p e e d i s e q u a l to t h e t o t a l d i s ­
- x - 1 , 000 - x
=

3 7 tance divided by the total time. The total distance is 200


3 2 3 miles. The time for the first hundred miles i s (100 miles/
Subtract - x : - x - - x - l , 000 = a
7 3 7 50 mph) = 2 hours, and the time for the second hundred
2 3 miles is (100 miles/75 mph) = 4/3 hours. Therefore the
Add 1,000: - x - - x = l , 000 total time of the trip i s 2 + 4/3 = 10/3 hours, and the aver­
3 7
5 age speed is
Combine like terms: - X = l, 000 200 3
21
w = 200 x 1 0 = 60 mph
2 1 , 000
Multiply by 21 : X= = 4, 200 3
5 5
1 6 . 0.20 Let g = the cost, i n dollars, of one gumdrop, 21. D 3m(m2 x 2m)
and c = the cost, in dollars, of one candy bar. Parentheses: 3m(2m3)
4c + 2g = 2 . 8 0 Multiply: 6m4
3c + 2g = 2 . 2 0
22. C Assume the cost of living i n 1 980 was $ 100. If
Subtract: c = 0.60 this increased by 20% from 1980 to 1990, then the cost of
Substitute c = 0.60: 4(0.60) + 2g = 2.80 living i n 1990 was 1 .20($100) = $ 1 20. If the increase from
1980 to 2000 was 50%, then the cost of l iving i n 2000 was
Simplify: 2.40 + 2g = 2.80 1 .50($100) = $ 1 5 0 . The percent increase from 1990 to
2000 is therefore
Subtract 2 .40: 2g = 0.40
1 50 - 1 20 30
--- x 1 00% = - x 1 00% = 25%
Divide by 2 : g = 0.20 1 20 1 20
296 McGraw-Hill Education: SAT

Ski l l 2 : Worki ng with Li nea r Equations


Lesson 5 : Constructing a n d interpreting
linear equations

The Horizon Resort charges $ 150 p e r night for a sin­ answer is (B). Notice that this equation shows a linear
gle room, and a one-time valet parking fee of $35. relationship between c and n .

There is a 6 . 5% state tax on the room charges, but no


tax on the valet parking fee. Which of the fol lowing
When setting u p equations from word problems,
equations represents the total charges in dollars, c,
try to classify the relationship ( that is, linear,
for a single room, valet parking, and taxes, for a stay
quadratic, exponential) between the variables,
of n nights at The Horizon Resort?
so that you can check that the equation is of the
A) c = (150 + 0.065n) + 35 correct form. In this lesson, we will focus only on
linear relationships, that is, relationships that
c
B) = l .065(1 50n) + 35 can be expressed in the form y = mx + b.
C) c = l .065(150n + 35)
D) c = 1 .065(150 + 35)n
Which of the following represents the equation of
(Medium) This question asks us exp licitly to set up an the line with a n x-i ntercept of 6 that passes through
equation to express a mathematical relationship in a the point (4, 4)?
word problem. Usually, this is just the first step in ana­ 1
lyzing the situation more deeply, for instance, finding A) y = - - x + 6
2
particular values of the variables that satisfy certain
B) y = 2x - 4
conditions, or interpreting the meanings of terms or
coefficients i n the equation, but this problem only asks C) y = -2x + 1 2
us to set up the equation. D) y = -2x + 6
When translating verbal information into an equa­
tion, it's helpful to take small steps. First, since the room (Easy) This question asks you to construct the equation of
charge is $ 1 50 per night, the charge for n nights is $ 1 50 n . a line given some facts about its graph. Start by drawing
[f a 6.5% t a x is added t o t h i s , the room charge becomes a graph . (on the xy-plane) of the given information i n the
150n + 0.065(1 50n) = l .065(150n) . The $35 valet pa rk­ space next to the question. It also helps to know some­
ing charge is added separately, and not taxed, so the thing about the different forms of linear equations and
total charges are l .065(1 50n) + 35, and the correct what they reveal about the graph of the line.

Graph of a line in the xy-plane

y
rise y2 - y 1
slope = -
run = x 2 - x1

rise '
T__
I

run
CHAPTER 7 I THE SAT MATH TEST: THE HEART OF ALGEBRA 297

Forms of linear equations

Slope-intercept form: y = m x+b Features: slope = m, y-i ntercept = b


Standard form: ax + by = c Features: slope = -alb, y-intercept = c/b, x-i ntercept = c/a
Point- slope form: y - y1 = m (x - x1 ) Features: slope = m, point on line = (x1 1 y1 )

I n this problem, we are given two points on the l ine: This i s the equation i n ( C ) . Notice that this equation i s i n
(4, 4) and the x-i ntercept (6, 0). We can calculate the slope slope-intercept form, and reveals that this line also has
using the slope formula above: slope = (4 - 0)/(4 - 6) = a y-i ntercept of 12. Check this fact against your diagram,
(4)/(-2) = -2 . I f we use this slope and the point (6, 0), we and also check that both g iven points, (4, 4) and (6, 0),
can set up the equation i n point-slope form: satisfy this equation.
Point-slope form of equation: y - 0 = -2(x - 6)

Simpli fy and d i stribute: y = -2x + 12


298 McGraw-Hill Education: SAT

Lesson 6: Solvi ng equations with the Laws of Eguality

1 2
If - + - = 4, what is the value of x? If x2 = y2, then which of the following must be true?
x 3x
I. x = y
A) 7 y
II. X = -
2
5 x
B) 3 III. x = IYI
5 A) none
C) 5
B) I only
12
1 C) I and II only
D)
6 D) I, II and III

(Medium) At first glance, this doesn't look like (Medium-hard) This question tests your skills of deduc­
a linear equation. But one simple move reveals that it is: tive logic. Notice it is not asking which statements can b e
1 2
-+-=4 true, b u t rather which must be true. I t s e e m s t h a t if we

( 2-)
x 3x "unsquare" both sides of the original equation, we get
Multiply both sides by 3x: 3x _!_ + = 3x( 4) the equation in I. I f we divide the original equation by x
x 3x on both sides, we get the equation i n II. Does this mean
Distribute and simplify: 3 + 2 = 1 2x that statements I and II are necessarily true? No, because
Divide by 1 2 : 5/ 1 2 = x we violated rule 3 in both cases. If x2 = y2, it does not fol­
As this shows, sometimes solving equations requires a low that x = y. Notice that x could be 2 and y could b e
clever use of the Laws of Equality. - 2 . These values certainly satisfy the original equation,
but they do not satisfy the equations i n I or III. They do,
however, satisfy the equation i n II, because 2 = (-2)2/2.
The Laws of Equality However, statement II is still not necessarily true. What if
Every equation is a b alanced scale, and the Laws x and y were both O? This would satisfy the original equa­
of Equality are simply the rules for "keeping the tion, but 0 ""' (0)2/(0) because 0/0 is undefined. Therefore,
scale b alanced;' that is, deducing other true equa­ the correct a nswer is (A).
tions. In a nutshell, the Laws of Equality say that
1. You m ay make changes to any e qu ation, as This example teaches u s two lessons:
I.
2
long a s you follow rules and 3. Before taking the square root of both sides of
2. Whatever you do to one side of the equation, an equation, remember that every positive
you must do to the other. number has two square roots. For instance
3. You may not perform undefined operations 3 -3.
the square root of 9 is or
(like dividing by 0), or operations that have 2. Before dividing both sides of an equation by
more than one possible result (like taking a an unknown, make sure it can't equal O.
square root).
CHAPTER 7 / THE SAT MATH TEST: THE HEART OF ALGEBRA 299

Lesson 7: Ma king a n d a na lyzi ng g raphs of


li nea r eg uations

I f m is a constant greater thanl,which o f the D)


following could be the graph in the xy-plane of y
x + my + m = O?

A)
y

(Medium-hard) First, we should try to get the equation


into a more usefu l form. Let's try the slope-intercept
(y = mx + b) form:
x + my + m = O
B) Subtract x and m: my = -x - m
y
Divide by m:

This shows that the line has a slope of - 1 /m and a y-inter­


cept of - 1 . Si nee the problem tells us that m is greater
l,
than we know that the slope ( 1 /m ) must b e between
-

- 1 and 0. The only graph that satisfies these conditions


is (B) .

Thinking about slopes

It's helpful to think of slope as the amount a line


goes up {or down) for each step it takes to the
C) right. Lines with a positive slope slant upward

y to the r ight, lines with a negative slope slant


downward to the right, and lines with a 0 slope
are horizontal. For instance, a line with slope
- 3 moves down 3 units for every unit step to the
right.

Parallel and perpendicular l ines


• Parallel lines have equal slopes.
• Perpendicular lines have slopes that are
opposite reciprocals of each other. That is, i f
one line h a s a slope of alb, i t s p erpendicular
has a slope of -b/a.
300 McGraw-Hill Education: SAT

The points A(lO, 4) and B(-2, k) are 13 u nits apart. Looking at the diagram more closely, notice that it
Which of the following equations could describe the includes two right triangles, and we c a n find the m i s s ­
line that contains points A and B? ing s i d e of e a c h one u s i n g the Pythagorea n Theorem,
A) 13x + 1 2y = 178 or just by noticing that they are both 5 - 1 2 - 1 3 right tri­
angles (52 + 1 22 = 1 32). Putting this information i nto the
B) 5x + 1 2y = 98
diagram shows u s that B can therefore be at (-2, 9) or
C) 5x - 12y = 98 (-2, - 1).
D) 5x - 1 3y = -2
y
(Hard) Drawing a diagram will help us analyze this
problem. Although we don't know precisely where point I
13
B is, we know it is somewhere on the line x = -2 . This s :
gives us the following picture:
( - 2 4) µ
,
I
12 A( l O, 4)
y s :
I
13
13
X= -2
(-2, 4) µ 12 A( l O, 4)
Therefore, the slope (rise/run) o f the line containing
B1 is - 5 / 1 2 , and the slope of the line containing B 2 is 5/ 1 2 .
Th i s means that our answer is either (B) o r (C). How d o we
choose between them? Just remember that the line must

-2
contain the point (10, 4). If you plug x = 1 0 and y = 4 into
X= these equations, only (B) works: 5(10) + 4(1 2) = 98.

Next, notice that a l l o f the equations given i n the


Checking your work
choices are i n "standard" form, and i n standard form
the slope of the line is - a / b. Therefore, the slopes of Always check that your solutions s atisfy your
these lines are, respectively, (A) - 1 3/ 1 2, (B) - 5 / 1 2 , equations by plugging them back into the equa­
(C) 5/ 1 2 , a n d (D) 5 / 1 3 . Therefore, finding t h e slope of tions to verify.
the line should help u s choose the correct equation.
CHAPTER 7 I THE SAT MATH TEST: THE HEART OF ALGEBRA 301

• •
I f x - 2(1 - x) = 5, what is the value of x? x + 1 2x .
I f -- + - = 1, what 1s the value of x?
10 5

I I I I I I I I I I
I f f(x) = -2x + 8, and f(k) = - 10, what is the value •
of k? What is the y-intercept of the line containing the
poi nts (3, 7) and (6, 3)?

I I I I I I I I
..
What is the slope of the line that contains the points Ill
(-2, 3) and (4, 5)?
I n the xy-plane, the graph of y = h(x) is a line with
slope -2. I f h(3) = 1 and h(b) = - 9, what is the value

I I I I I
of b?


What is the slope of the line described by the
I I I I I
l 1 5
equation - + - = - ?
x 2x y ..
If a train maintains a constant speed of 60 miles per
hour, it can travel 4 miles per gallon of diesel fuel. If
this train begins a trip with a full 200 gallon tank of die­
sel fuel, and maintains a speed of 60 miles per hour,
.. which of the following equations represents the num­
ber of gallons, g, left in the tank t hours into the trip?
Line l is perpendicular to the line described by the ) 200 - 60t
equation 5x + l ly = 16. What is the slope of line /? A g=
4
1
B) g = 200 - -

I I I
1 5t
C) g = 200 - 1 5 t
1
D) g = 200 - - t
15
302 McGraw-Hill Education: SAT

1111 • -��.�--- -·-

The points A(2, 3) and B(m, 1 1), are 10 units apart.


Which of the following equations could describe the y
line that contains points A and B? (5 , 6)
A) Bx + 6y = 1 1
B) Bx - 6y = -2
C) 6x + By = 36
D) 6x - By = - 1 2 0
(k, 0)

The figure above shows a right triangle with vertices


at the origin, (5, 6) and (k, 0). What is the value of k?
A) 19
3
B) 5B
5
C) 26
3
D) 6 1
5
CHAPTER 7 / THE SAT MATH TEST: THE H EART OF ALGEBRA 303

5 g q ue tj a #&&f

IFI ..
If the points (2, 4), (5, k), and (8, 20) are on the same If � + 2- = 4 , what is the value of x?
line, what is the value of k? x 5x

I I I I I I I I I I
Ill
Line l has a slope of 3 and a y-intercept of - 4. What Ill
is its x-intercept? If the line 3x - 2y 1 2 is graphed in the xy-plane,
=

what is its x-intercept?

I I I I I I I I
m
If f(- 1) 1 and 1(3) 2 and f is a linear function,
= =
Ill
what is the slope of the graph y = f(x)? If the graphs of the equations 5x - 2y 5 and
=

6x + ky = 9 are perpendicular, what is the value of k?

I I I I I I I I I I
Bl
I f f(- 1) 1 and 1(3)
= = 2 and f is a linear fu nction,
what is f(5)? The net profit for the sales of a product is equal to the
total revenue from the sales of that product minus
the total cost for the sales of that product. If a par­
I I I I I ticular model of calculator sells for $98, and the cost
for making and selling n of these calculators is
$(35n + 1 20,000), which of the following equations
expresses the net profit i n dollars, P, for making and
selling n of these calculators?
I n the xy-plane, the graph ofline n has an x-intercept
of 2b and a n y-intercept of - 8b, where b o;e 0. What is A) P = 63n - 1 20,000
the slope of line n? B) P = 63n + 1 20,000
C) P = 63(n - 1 20,000)

I I I I I D) P = 63(n + 1 20,000)
304 McGraw-Hill Education: SAT

Which of the following represents the equation


x 2 3 4
of the line with an x-intercept of 5 and a y-intercept
of 6? f(x) a 8 b

6
A) y - 6 = - - (x - 5) The table above shows several ordered pairs corre­
5
sponding to the linear function f What is the value
5
B) y - 6 = - - (x - 5) of a + b?
6
6 A) 1 2
C) y - 6 = - - X
5 B) 1 6
5
D) y - 6 = - - x C) 20
6
D) It cannot be determined from the information
given.
CHAPTER 7 I THE SAT MATH TEST: THE HEART OF ALGEBRA 305

EXE RCISE SET 3 ANSWER KEY

Part 1: No Calculator Notice that to get from (6, 3) to (3, 7) we must go


left 3 units and up 4 units (in other words, the slope
1 . 7/3 or 2.33 x - 2(1 - x) = 5 is - 4/3). I f we simply repeat this from (3, 7), we arrive at
the y-intercept, which is (0, 1 1).
Distribute: x - 2 + 2x = 5
Simplify: 3x - 2 = 5
8. 8 This line has a slope of -2 and contains the
Add 2: 3x = 7 -9 - 1
points (3, 1) and (b, - 9). Therefore - 2 = --
Divide by 3: X = 7/3 b-3
- 10
Simplify: -2 = --

b-3
2. 9 fl.k) = -2k + 8 = - 1 0
Multiply by b - 3: -2b + 6 = - 1 0
Subtract 8: -2k = - 1 8 Subtract 6 : -2b = - 1 6
D ivide by -2: k=9 Divide b y -2: b=B

or
3 . 1 /3 .333 5-3 2 1
slope = ---

4 - ( - 2) 6 3 9. C Since the tanks starts with 200 gallons, the


amount it has left is 200 - the number of gallons used.
or
4. 1 0/3 3.33 1 1 5 The number of gallons used is
-+-=­
x 2x y
2xy 2xy lOxy 60 miles 1 gallon
Multiply by 2xy: + = t hours x x = 1 5t gallons
x 2x y 1 hour 4 miles
Simplify: 2y + y = lOx
Simplify: 3y = lOx
10 10. B Once again, a quick sketch can be very helpful.
D ivide by 3: y = -X Notice that traveling from point
3

y
5 . 1 1 /5 or 2.2 The slope of the given line
is - 5/ 1 1, so the slope of the line perpendicular to y = 11
it is 1 1 /5. (m, 1 1 )
10
or
6. 9/5 1 .8 x + l 2x
-- +-=l
10 5
Multiply by 10: (x + 1) + 4x = 10
Simplify: 5x + 1 = 10
Subtract 1: 5x = 9
Divide by 5: X = 9/5 A (2, 3) to point B(m, 1 1) requires going up 8 u n its and
right (or left) some u nknown distance b. We can find b
7. 1 1 There are a variety of ways of solving this prob­ with the
lem, but perhaps the simplest is to draw a quick Pythagorean Theorem: 32 + b2 = 102
sketch: Simplify: 64 + b2 = 100
y Subtract 64: b2 = 36
(0, 1 1 ) Take the square root: b=6
4
- (3, 7) Therefore, m is either 2 - 6 = - 4 or 2 + 6 = 8, and the
3 I
slope of this line is either 8/6 = 4/3 or 8/(- 6) = -4/3. The
4 1<"]_ -
(6, 3) only equation among the choices that is satisfied by the
3 ordered pair (2, 3) and has a slope of either 4/3 or - 4/3
---+---->.-- x is (B).
0
306 McGraw-Hill Education: SAT

1 1 . D Recall that the slopes of perpendicular lines are 2 2


opposite reciprocals. The slope of the segment from (0, 0) 17. 3/5 o r .6 -+-=4
x 5x
to (5, 6) is 6/5, so the slope of its perpendicular is - 5/6.
6-0 5 Multiply by 5x: 10 + 2 = 20x
Therefore
5-k 6 Simplify: 12 = 20x
Cross-multiply: -36 = 5 (5 - k) Divide by 20: x = 1 2/20 = 3/5
Distribute: -36 = 25 - 5k
Subtract 25: - 6 1 = -5k 18. 4 The x-intercept i s the value of x for which
Divide by - 5 : 6 1 /5 = k y = O: 3x - 2 (0) = 1 2
Simplify: 3x = 1 2
Divide by 3: x=4
Pa rt 2: Calcu lator
1 2 . 1 2 The slope of this line is 20 - 4 = 1 6 = � , 1 9 . 1 5 Recall that the slope of a line i n standard form
8-2 6 3 ax + by = c is -alb. Therefore, the slope of 5x - 2y 5 is =

k-4 8 5/2 and the slope of 6x + ky = 9 is - 6/k. I f these lines


therefore,
5-2 3 are perpendicular, then their slopes are opposite
Cross-multiply: 3k - 1 2 = 24 reciprocals:
Add 1 2 : 3k = 36 k 5
D ivide b y 3: k = 12 6 2
Multiply by 6: k = 30/2 = 1 5

1 3 . 4/3 o r 1 .3 3 Since the slope and y-inter-


cept are given, it is easy to express the linear equation in 2 0 . A 111e total revenue for selling n calculators a t $98
slope-intercept form: y = 3x - 4. each is $98n the cost for making and selling n calculators
cllie x-intercept i s the value of x on the line for is $(35n + 1 20,000). Therefore the profit is $(98n - 35n -
which y = O: 0 = 3x - 4 1 20,000) = 63n - 1 20,000 dollars.
Add 4: 4 = 3x
Divide by 3: 4/3 = X 21. C This line contains the points (5, O) and (0, 6) and
therefore has a slope of O - 6 = _ � .
14. V<i o r .25 The line contains the points (- 1 , l) and 5-0 5
(3, 2), so its slope is 2 - l 1
Since its y-intercept is 6, its slope-i ntercept form is
3 - (- 1) 4

y = - x + 6 or, subtracting 6 from both sides,
15. 5/2 or 2 . 5 Although we could solve this problem 5
by deriving the linear equation, it is perhaps easier 6
y - 6 = --x
to take advantage of the result from question 14. The 5
slope of 1 /4 means that the y-coordinate of any point
on the line increases by 1 /3 each time the x-coordinate 22. B Si nee f is a linear function, it has a slope that
increases by 1. Since the x-coordinate increases by 2 we can call m . Recall that it's often useful to think of the
betweenf(3) andj(5), the y-coordinate must therefore slope of a line as the "unit change," that is, the amount
increase by 2(1/4) = 1 /2, so J(5) = 2 + V2 = 2.5. that y changes each time x increases by 1. Since the x val­
ues increase by 1 with each step i n our table, the y values
16. 4 The line contains the points (2b, 0) and (0, - 8b); must therefore increase by m with each step. Thi s means
0 - ( - 8b) 8b that a = 8 - m and b = 8 + m . Therefore, a + b = 8 - m +
therefore, it has a slope of = - = 4.
2b - O 2b 8 + m = 16.
CHAPTER 7 / THE SAT MATH TEST: THE HEART OF ALGEBRA 307

Ski l l 3 : Worki ng with I neq ua l ities a nd


Absol ute Va l ues
Lesson 8: Understa nding i nequalities and absol ute va l ues

On the real number line, a number, b, is more than (Medium-hard) We can use this definition to translate
twice as far from -3 as it is from 3. Which of the fol­ the problem. The key is to translate the statement "b
lowing equations can be solved to find all possible is more than twice as far from -3 as it is from 3" into
values of b? a statement about distances: "The distance between b
and -3 is more than twice the distance between b and
A) l b - 3 1 > 2 lb + 3 1 3." Notice how easily this translates into a n inequality:
B) lb + 3 1 > 2 lb - 3 1 (The distance between b and -3) is more than (twice
the distance between b and 3)
C) 2l b - 3 1 > l b + 3 1
D) 2 lb + 3 1 > l b - 3 1 lb - (-3) 1 > 2 lb - 3 1

l b + 3 1 > 2l b - 3 1
Distance and absolute value
which is choice (B).
The absolute value of a number x, written as lxl,
means its distance from 0 on the number line. In
fact, we can use absolute value to represent the
distance between any two numbers.

Ix - a l means the distance between x and a on


the number line.

Notice that this works no matter which number is


greater. For instance, the distance between 2 and
7 is 12 - 7 1 = I - S I = S, which is the same as the
distance between 7 and 2: 1 7 - 21 = ISi = S.
Notice that an expression like Ix + al is equiva­
lent to Ix - (-a)I, which means that Ix + a l can
be translated as the distance between x and -a.
308 McGraw-Hill Education: SAT

Lesson 9: Solving inequalities with the Laws of lneguality


I f _ 2_ < - 2x + 1 < _ 2_ , what i s one p o s s ible value So we can solve the sandwich inequality by applying
2 3
of x? the correct laws of inequality: - -1 < - 2x + l < - -1
2 3

I I I I I
Multiply by - 6 (the common
denominator) and "switch:" 3 > 1 2x - 6 > 2

Add 6: 9 > 1 2x > 8


(Easy) This kind of inequality is called a "sandwich
inequality" because the expression in the middle is Divide by 1 2 : 0.75 > x > 0.66 . .
.

between the other two, like meat between slices of


Therefore, any value greater than 0.666 but less than
bread. Working with inequalities like this one requ ires
0.750 is correct.
understanding the Laws of Inequality. a
Which of the following must be true if -< - 3?
-

h
The L aws of lnequality A) a S -3h
Every inequality is a "tipped" scale, and the Laws B) a 2:'. -3h
o f lnequality are simply the rules for "keeping t h e
C) a ::; -3h < 0 or a 2:: -3h > 0
scale tipped the right way," that is, deducing other
true inequalities that follow from the original D) a s -3h < 0 or a 2:: -3h > 0
one. I n a nutshell, the Laws of Inequality say that
(Hard ) We might be tempted to multiply both sides of
1 . You may make changes to any inequality, as the inequality by h and get the i nequality in (A). But
long as you follow rules 2, 3 and 4 . this would be incorrect because it would ignore rule 4.
2 . Whatever y o u do to one side of the inequality, We need to consider the possibility that h might he nega­
you must do to the other. tive. Let's think about possible solutions to the original
3. You may not perform undefined operations inequality. Notice that a = J O and h = -2 gives a possible
(like d ividing by 0), or operations that h ave solution, because 1 0/(-2) = -5 s -3. But this would not
more than one possible result (like taking a satisfy the inequality in (A): 10 is not less than or equal to
square root) . (-3)(-2) = 6.
4. I f you multiply or d ivide both sides by a nega­ To solve this inequality, we will need to consider two
tive number, you must "switch" the direction distinct possible conditions:
of the inequality. This is because multiplying
or dividing by a negative number involves a Condition 1 : I f h > 0, then a s - 3h and t herefore
reflection over the origin on the number line, a S -3h < 0
and this reflection requires the "switch": Condition 2: I f h < 0, then a 2:: -3h and therefore


a 2:: -3h > 0

which is the answer in choice (D).


• I I I I I I I I I �

-4 <-2 0 2 < 4
CHAPTER 7 I THE SAT MATH TEST: THE HEART OF ALGEBRA 309

Les so n 1 0: Graphing inegua lities

O n the real number line, a number, b, is more than twice lb + 31 > 2 lb - 3 1 . How do we translate this i nto a graph?
as far from -3 as it is from 3. Which of the following The simplest way to start i s to visualize the number line,
graphs represents all possible values of b? and to think about a related, but simpler, question: What
if b is exactly twice as far from -3 as it is from 3? A little
A) guessing and checking should reveal that two points
work:
--? I I I ; I I
Exactly twice a s fa r from - 3 a s from 3
-3 0 1 3 9 /' ""
• + I I + I + I + I "
-3 0 1 3 9
B)
Notice that 9 works because 3 is the m idpoint between
...-.: I I I ? I .. -3 and 9, and 1 works because it is 2/3 of the way from -3
-3 0 1 3 9 to 3 . Also, you can confirm that both numbers satisfy the
equation lb + 3 1 = 2lb - 31. These two points now divide
the line into three parts: everything less than 1, every­
C) thing between 1 and 9, and everything greater than 9. A
little bit of checking (just pick a number from each por­
-3 0 1 3 9 tion and plug it i nto our inequality) confirms that only
the numbers i n the m iddle portion satisfy our inequality,
so the correct graph is the one in choice (D).
D)
When graphing inequalities, it often helps to start
with the graph of the corresponding equation
-3 0 1 3 9
and work from there. The graph of the equation
usually provides the boundaries for the graph
(Medium) We saw this scenario i n Lesson 8, but now of the inequality.
we are asked to graph the solution. Recall from Lesson
8 that this relationship is expressed by the inequality
310 McGraw-Hill Education: SAT

Exerc ise¥ Set 4 (No Calcu lator) MM ¥ FEW !1 flii Mi t l &12' &b i' Ht "

• Ill
What positive number is twice as fa r from 1 0 as it is Which of the following statements is equivalent to
from 1 ? the statement -4 < 2x � 2?
A) x > -2 and x � 1

I I I I I B) x < -2 or x 2'. 1
C) x 2: -2 and x < 1

n D) x � -2 or x > 1

I f the points (2, a) and (14, b) are 20 units apart, what


is la - W B
The annual profit from the sales of an item is equal

I I I I
to the annual revenue minus the annual cost for
that item. The revenue from that item is equal to the
number of units sold times the price per unit. I f n
u n its of a portable heart monitor were sold i n 201 2 at
.. a price of $65 each, and the annual cost to produce n
What is the least integer n for which O < � < � ? units was $(20,000 l On), then which of the follow­
n 9
ing statements indicates that the total profit for this
heart monitor in 2 0 1 2 was greater than $500,000?
I I I I A) 500,000 < 55n - 20,000
B) 500,000 > 55n - 20,000
II C) 500,000 < 55n + 20,000n
I f Ix + 4 1 = Ix - 51, what is the value of x?
D) 500,000 < 75n - 20,000n

I I I I I Colin can read a maximum of 25 pages an hou r. I f he


.. has been reading a 250 page book for h hours, where
h < 10, and has p pages left to read, which of the fol­
What is the greatest integer value of n such that lowing expresses the relat ionship between p and h?
n 1 25
-- > - -?
21 2 A) 250 - p � h

I I I I I 25
>p+-
B) 250 -
h
• C) 250 - p � 25h
1 3 D) 250 + 25h � p
What is the only integer b for which - > - and
3b 2: 7. 5? b 11

m
On the real number l ine, a number, x, is more than
4 times as far from 1 0 as it is from 40. Which of
the following statements describes all possible val­
ues of x?

A) x < 34 or x > 50
I f (b + 2)2 = (b - 5)2, what is the value of b?
B) x > 40
C) 34 < x < 50

I I D) 32.5 < x < 1 60


CHAPTER 7 I THE SAT MATH TEST: THE HEART OF ALGEBRA 31 1

Exerc ise Set 4 (Ca lc u lator)

1111
If a < 0 and la - 51 = 7, what is lal? If the average (arithmetic mean) of a and b is greater
than the average (arithmetic mean) of c and 2b,
which of the following must b e true?

I I I I I A) b > O
B) a > b

111 C) a > b + c
If n i s a positive i nteger and 16 > 1 6 - 3nl > 19, what D) a + c > b
is the value of n?
Ill

I I I I I
Of the statements below, which is equivalent to the
statement "The distance from x to 1 i s greater than
the distance from x to 3?
A) 1 <x<3
ID
B) x > 2
What is the only integer n such that 20 - 2n > 5 and
2n C) x < 2
- > 4?
3 D) x - 1 > 3

m
Which of the following is equivalent to the statement
4x2 2: 9?
DI
A) 2x > 3
What i s the smallest number that is as far from 9.25
as 3 i s from - 1 .5? B) x 2: 1 . 5 or x :::; - 1 .5
C) lxl > 2

I I
D) - 1 . 5 S x S 1 . 5


If l2x + I I = 2lk - 4 for all values of x, what is the 0 6
value of lkl?
The graph above indicates the complete solution set
to which of the following statements?

I I I I I A) Ix - 3 1 > 3
B) lxl < 6
C) Ix - 6 1 < 6
Ill
D) Ix - 3 1 < 3
Which of the following is equivalent to the statement
Ix - 21 < I?
m
A) x<3 Which of the following is true for all real values of x?
B) x < -l A) lxl > 0
C) l <x<3 B) x < 2 or x > 1
D) -1 < x < 3 C) x > -2 or x < -3
D ) x2 - 1 > 0
312 McGraw-Hill Education: SAT

EXERCISE SET 4 ANSW E R KEY

No Calculator 6. 3 3b � 7.5

1. 4 It is helpful to plot these values on the number Divide by 3: b � 2.5


line and think: 1 3
->­
I- 3 --+-- 6 --j b 11
Since b is greater than or equal to 2.5, it is positive, so we
-8 01 4 10 can multiply both sides by l l b without "swapping" the
� 9 -----i inequality:
>--��� i s-����
1 1 > 3b
The distance between 1 and 10 is 9, so clea rly the number
that is 9 more units to the left of 1 1 namely - 8, is twice Divide by 3: 3.67 > b
as far from 10 as it is from 1. However, this is a negative The only integer between 2 . 5 and 3.67 is 3 .
number so it ca n't be our answer. There is one other
number that is twice as fa r from 10 as it is from 1: the
number that is 1 /3 the distance from 1 to 10. Th is number 7 . 3/2 o r 1 . 5 ( b + 2)2 = ( b - 5)2
is 4, which is 3 units from 1 and 6 units from 10. FOIL: b2 + 4b + 4 = b2 - l O b + 25
Subtract b2: 4b + 4 = - lOb + 25
2 . 16 From the Distance Formula, Add lOb: 14b + 4 = 25
(2 - 14)2 + (a - b)2 = 202 Subtract 4: l4b = 2 1
Simplify: 144 + (a - b)2 = 400 Divide by 14: b = 1.5

Subtract 144: (a - b)2 = 256


8. A - 4 < 2x ::; 2
Square root: l a - bl = 16
Divide by 2 : -2 < x ::; 1
3. 8 4 5
0<-<- which is equivalent to -2 < x and x ::; 1 .
n 9
Since n must be positive for this statement to be true, we 9. A The profit is the revenue minus t h e cost:
can multiply by 9n without having to "swap" the inequal­ 65n - (20,000 + lOn) = 55n - 20,000.
ity symbols:
0 < 36 < 5n 10. C I f Colin can read a maximum of 25 pages an
hour, then in h hours he can read a maximum of 25h
Divide by 5: 0 < 7.2 < n pages. I f he has p pages left in a 250-page book, he has
Therefore, the smallest integer value of n is 8. read 250 - p pages. Since it has taken him h hours to read
these 250 - p pages, 250 - p ::; 25h.
4 . V2 o r . 5 T\.vo numbers, a and b, have the same 11. c Ix - 1 0 1 > 4 lx - 401
absolute value only if they are equal or opposites. Clearly
x + 4 and x - 5 cannot be equal, since x - 5 is 9 less than It helps to sketch the number line and divide is i nto
x + 4 . Therefore they must b e opposites. three sections: the numbers less than 10, the numbers
between 10 and 40, and the numbers greater than 40.
x + 4 = -(x - 5) CASE 1 : x < 10. It should be clear that all numbers less
Distribute: x + 4 = -x + 5 than 10 are closer to 10 than they are to 40, so this set
Add x: 2x + 4 = 5 conta ins no solutions.
Subtract 4: 2x = 1 CASE 2 : 10 < x ::; 40. I f x is between 10 and 40, x - 1 0 i s
positive and x - 4 0 is negative, so Ix - 1 0 1 = x - 10 a n d
Divide by 2: x = 1/2
Ix - 401 = - (x - 40).
Ix - 10 1 > 4lx - 401
5. 1 0 n 1
-- > - - Substitute: x - 10 > - 4 (x - 40)
21 2
Multiply by -4 2 and "swap:" 2n < 2 1 Distribute: x - 10 > - 4x + 160
Add 4x: 5x - 10 > 160
Divide by 2 : n < 10.5
Add 10: 5x > 170
Therefore, the greatest possible integer value of n is 10.
Divide by 5: x > 34
CHAPTER 7 I THE SAT MATH TEST: THE HEART OF ALGEBRA 313

So this gives us 34 < x :::; 40. like x = 1. l2x + l l = 2lk - xi


CASE 3 : x > 40. If x is greater than 40, then both x - 10 Substitute x = 1 : 12(1) + l l = 2lk - l l
and x - 40 are positive, so Ix - 1 0 1 = x - 10 and Ix - 401 = Simplify: 3 = 2lk - l l
1.5 = lk - ll
x - 40.
Divide by 2:
lx - 101 > 4 lx - 401
Therefore ± 1 .5 = k - 1
Substitute: x - 10 > 4(x - 40)
Add 1 : k = 2 . 5 or - 0 . 5
12(0) + l l = 2 l k - 0 1
Distribute: x - 10 > 4x - 1 6 0
Now try x = 0 :
Add 10: x > 4x - 150
Simplify: 1 = 2 l kl
Subtract 4x: -3x > - 1 50
Divide by 2: 0.5 = lkl
Divide by -3 and "swap: " x < 50
Therefore ±0.5 = k
So this gives us 40 < x < 50. When we combine this with
Therefore, k = - 0 . 5 and so lkl = l - 0. 5 1 = 0.5.
the solutions from CASE 2, we get 34 < x < 50.

17. C Recall that the expression Ix - 2 1 means "the


distance from x to 2," so the statement Ix - 2 1 < 1 means
Calculator "The distance from x to 2 is less than l ." Therefore, the
solution set is all of the numbers that are less than 1 u n it
12. 2 I f l a - 5 1 = 7, then either a - 5 = 7 or a - 5 = - 7, away from 2, which are all the numbers between 1 and 3 .
so either a = 1 2 or a = -2. Since a < 0, a must be -2, and
1 -2 1 =2. a + b c + 2b
18. c -- > --
2 2
1 3 . 8 CASE 1: I f 6 - 3n is positive, then Multiply by 2 : a + b > c + 2b
1 6 - 3nl = 6 - 3n, so 1 6 > 6 - 3n > 19 Subtract b: a > c+b
Subtract 6: 10 > -3n > 1 3
Divide by -3 and "swap:" - 1 0/3 < n < - 1 3/3 19. B The formal translation of this statement i s
But this contradicts the fact that n is positive. Ix - l l > Ix - 3 1 , which w e can solve algebraically b y con­
sidering three cases: (I) x :::; l , (II) 1 < x :S 3, and (II I) x >
CASE 2 : If 6 - 3n is negative, then
3, but it is probably easier to just graph the number line
1 6 - 3nl = - (6 - 3n), so 1 6 > - (6 - 3n) > 19 and notice that the midpoint between 1 and 3, that is, 2,
Distribute: 1 6 > - 6 + 3n > 1 9 is the point at which the distance to 1 and the distance
Add 6: 22 > 3n > 25 to 3 are equal. Therefore, the points that are farther from
1 than from 3 are simply the points to the right of this
Divide by 3: 7. 33 > n > 8.33
midpoint, or x > 2.
And the only integer i n this range is n = 8.
20. B 4x2 2 9
14. 7 20 - 2n > 5 Take square root: l2xl 2 3
Subtract 20: -2n > - 1 5 I f x > 0: 2x 2 3
D ivide by -2 and "swap: " n < 7. 5 Divide by 2: x 2 1.5
2n I f x < 0: 2x :::; - 3
->4
3
Divide by 2: x :S - 1 . 5
Multiply by 3: 2n > 1 2
D ivide by 2 : n>6 2 1 . D Notice that t h e midpoint o f t h e segment shown
Since n must be an i nteger between 6 and 7. 5, n = 7. is 3, and the graph shows all points that are less than 3
units in either direction. Therefore, Ix - 3 1 < 3 .
1 5 . 4.75 The distance from 3 to - 1 . 5 is 13 - (- 1 . 5) 1
= 4.5. Therefore t h e two numbers that a r e 4.5 away from 22. B (A) is untrue if x = 0 , (C) is untrue for x = -2,
9.25 are 9.25 + 4.5 = 1 3 .75 and 9.25 - 4 . 5 = 4.75. and (D) is untrue if x = 0.5. But (B) is true for any real
number.
1 6 . Y2 o r . 5 If the equation is true for all values of x,
let's choose a convenient value for x,
314 McGraw-Hill Education: SAT

Ski l l 4: Worki ng with Li nea r Systems


Lesson 1 1 : Constructi ng, graphing, a nd
interpreting l i nea r systems

y = 2x - 3
A system o f equations i s just a set o f equations
that apply simultaneously to a given problem y = 2 x + 17
-

situation. Solving for the system means finding If the solutions to the two equations above are
all sets of values for the unknowns that make graphed i n the xy-plane, what is the y-coordinate of
all of the equations true. Systems of equations the point at which the graphs intersect?
can be analyzed both algebraically (by exploring
the e quations) or geometrically (by exploring the
graphs).
I I I I I
Two high school teachers took their classes on a field (Easy) Since the equations of both lines are given i n
trip to a museum. One class spent $154 for admis­ slope-intercept form, w e could graph the two lines i n the
sion for 20 students and 3 adults, and the other xy-plane to find their point of intersection.
class spent $188 for admission for 2 4 students and 4
adults. Which of the following systems of equations
could be solved to determine the price of a single y
y = 2x - 3
student admission, s, and the price of a single adult
admission, a, in dollars?
A) a + s = 51
44s + 7a = 342
B) 20s + 3a = 154
24s + 4a = 188
20
C) + � = 1 54 y = - 2x + 1 7
s a
24 4
- + - = 188 Therefore, the point (5, 7) gi v es u s the only solution to this
s a
system, and so the answer to the original question is 7.
D) 20 + 24 = s Alternately, (as we will see in Lesson 13) we c a n
3+4=a j u s t add t h e corresponding sides of t h e t w o equations
together to get 2y = 14, which yields y = 7.
(Medium) This problem can b e described with a two -by­
two system of equations, that is, two equations with two
unknowns. The two equations come from two facts: one The solution of a two-by-two system of equations
class spent $154 for admission and the other class spent can be visualized as the intersection of their
$188 for admission. The cost of 20 student admissions graphs in the xy-plane.
and 3 adult admissions is 20s + 3a, so the first equation
i s 20s + 3a = 154. Similarly, the equation for the other If the graphs are parallel lines, or other non­
class is 24s + 4a = 188, so the correct answer is (B). intersecting graphs, then the system has no solu­
tion. If the graphs intersect multiple times, then
the system has multiple solutions.
CHAPTER 7 I THE SAT MATH TEST: THE HEART OF ALGEBRA 31 5

y - 4x = 6 (Medium) This is a two -by-two system of linear equa­


tions, and so its solution is the intersection of those two
16x = 4y + k lines. If we convert them to slope-intercept form, we get
y = 4x + 6 and y = 4x - k/4, which reveals that these
For what value of k does the system of equations
two lines have the same slope. This means that they are
above have at least one solution? either parallel lines or identical lines. Two lines with the
A) -32 same slope can i ntersect only if they are the same line,
B) -30 and therefore -k/4 = 6 and k = -24.
C) -24
D) -20
316 McGraw-Hill Education: SAT

Lesson 1 2: Solving systems by substitutio n

Let's go back t o the second linear system from Lesson 1 1 . 3x + y = 3y + 4


This system can also b e solved with a simple application
of the Law of Substitution. x + 4y = 6
Based on the system of equations above what i s the
y = 2x - 3
value of the product xy?
y = -2x + 17

I I I I I
1 . Substitute for y: 2x - 3 = -2x + 17
2. Add 2x: 4x - 3 = 17
3. Add 3: 4x = 20
4 . Divide b y 4: X=5 (Medium) This system is not quite as tidy as the previ­
5. Plug into either original ous one, but we can still solve it by using the Law of
equation to find y: y = 2 (5) - 3 or -2(5) + 17 = 7 Substitution.
3x + y = 3y + 4

When one of the equations i n a system is already x + 4y = 6


solved for one variable (or when it's relatively Subtract 4y from second equation to
easy to solve it for one variable), then substitut­ isolate x : X = - 4y + 6
ing for this variable in the other equation often
makes it easier to solve the system. Substitute for x in first
equation: 3(- 4y + 6) + y = 3y + 4
Simplify left side: - l ly + 1 8 = 3y + 4
Add l ly and subtract 4: 14 = l4y
Divide by 14: l =y
Substitute y = 1 to find x: X = - 4(1) + 6 = 2
Evaluate xy: xy = (2)(1) = 2
CHAPTER 1 I THE SAT MATH TEST: THE HEART OF ALGEBRA 317

Lesson 1 3 : Solving systems by linear combination

3x + 6y = 18 3x - y = 20
3x + 4y = 6
2x + 4y = 7
Based on the system of equations above, what is the
Based on the system of equations above, what is the
value ofy?
value of x - 5y?

I I I I I I I I I I
(Ea sy) Although this system can be solved by substitu­
(Medium) This question looks tougher than the previous
tion (try it as an exercise), the setup of these equations
one, because it's not just asking for x or y. It seems that
suggests a much easier method, known as linear combi­
the question requires us to solve the system for x and y
nation. It's based on a simple idea:
and then to plug these values into the expression x - 5y
and evaluate. We could do that, but there is a much sim­
Th e Law o f Combination pler method. Notice that a simple combination gives us
the expression the question is asking for.
If a = b and c = d, then a + c b + d,
a-c = b - d, and ac bd
=
3x - y = 20
I n other words, you should always feel free to add, - (2x + 4y = 7)
subtract, or multiply the corresponding sides of Subtract equations: x - 5y = 1 3
two equations to make a new equation.
Using Linear Combination
I f we apply this rule to our system, notice that we can When you're given a system of equations o n the
easily eliminate x from the system by j ust subtracting SAT, always notice carefully what the question
the equations: is asking you to evaluate. Even if it appears to b e
3x + 6y = 18 t h e value of a complicated expression, often you
can find it with a simple combination of the given
- (3x + 4y = 6) equations.
2y = 1 2

Divide by 2 : y=6
318 McGraw-Hill Education: SAT

Exerc ise Set S (No Ca lc u lator)

• •
I f 3x + 2y = 72, and y = 3x, what is the value of x? I f ab = - 4 and abc = 1 2, what is the value of _!__ ?
ab

I I I I I I I I I I
• •
If 2a - 7b = 10 and 2a + 7b = 2, what is the value of I f a and b are constants and the graphs of the lines
4a2 - 49b2? 2x - 3y = 8 and ax + by = 2 are perpendicular, then
what is the value of 3a ?
b
I I
..
I f the lines y = - 4x - 3 and y = -3x - b intersect at •
the point (- 1 , c), what is the value of b?
Sx - y = 1 1
2x - 2y = 9

I I Based on the system of equations above, what is the


value of 3x + y?

• A) -2
I f the lines 4x + Sy = 1 3 and 4y + kx = 2 are parallel, B) 0
what is the value of k? C) 2
D) 4

I I I
Two numbers have a difference of 4 and a sum of -7 .
.. What is their product?
I f the lines 4x + Sy = 1 3 and 6y - kx = 6 are perpen­
A) -33
dicular, what is the value of k?
B ) - 10.2S
C) 8.2S
I I I I I D) 10.2S

.. Ill
2a It costs Emma p dollars to make each of her custom
bracelets, which she sells for m dollars apiece. She
b 3
makes a profit of $60 if she makes and sells S of these
c s
-+l=­ bracelets, but she only makes a profit of $10 if she
b 3 makes S bracelets but only sells 4 of them. How much
does it cost Emma to make each bracelet?
B ased on the system of equations above, what is the
value of f!_ ? A) $36
c
B ) $38
C) $48
D) $SO
CHAPTER 7 I THE SAT MATH TEST: THE HEART OF ALGEBRA 319

Exerc ise Set 5 (Ca lcu lator)

Ill m
I f 2y = x + 1 and 4x + 6y = 0, then y = The average (arithmetic mean) of x and y is 14. I f the
value of x is doubled and the value of y is tripled,
the average (arithmetic mean) of the two numbers

I I I I I remains the same. What is the value of x?

Ill
4 6
I I I I I
I f 6x + 7y = - and 6x - 7y = -, then y =
5 5

I I I I I
7m + lOn = 7
6m + 9n = 1
Based on the system of equations above, what is the
2x - Sy = 20 value of 4m + 4n?
lOx - 25y = 4k

ID
I I I I
For what value of k does the system of equations
above have at least one solution? I n the xy-plane, perpendicular lines a and b inter­
sect at the point (2, 2). I f line a contains the point

I I I I I
(7, 1), which of the following points is on line b?
A) (0, 1)
B) (4, 5)
Ill C) (7, 3)
At the beginning of the week, the ratio of cats to D) (3, 7)
dogs at Glenna's Pet Store was 4 to 5. By the end of
the week, the number of cats had doubled, while the
number of dogs had increased by 1 2 . If the ratio of
c ats to dogs at the end of the week was 1 to 1, how Which of the following pairs of equations has no
many cats did the store have at the b eginning of the solution in common?
week? A) 2x - 3y = 1 and 6x - 9y = 3
B) y = 4x and y = - 4x

I I C) 2x - 3y = 1 and 6x - 9y = 2
D) y = 4x and 2y - 8x = 0

Jenny originally had twice as many friendship


I n the xy-plane, the line l is perpendicular to the
bracelets as Emilie. After Jenny gave Emilie 5 of her
friendship bracelets, Jenny still had 10 more than line described by the equation _!_ + � = _!_ . What is
Emilie. How many friendship bracelets did Jenny the slope of line l? x 2Y Y
have originally?
A) -2
1

I I I I
B)
2
C)
2
D) 2
320 McGraw-Hill Education: SAT

EXERCISE SET S ANSW E R KEY

No Calculator 7 . • 75 or 3fii abc = 1 2


Substitute ab = - 4: (-4)c = 1 2
1. 8 3x + 2y = 72
Divide by -4: c = -3
Substitute y = 3x: 3x + 2 (3x) = 72 c
Simplify: 9x = 72 Expression to evaluate:
ab
Divide by 9: X=8
Substitute c = -3 and ab = -4: c -3 3
2. 20 4a2 - 49b2 ab -4 4
Factor: (2a - 7b)(2a + 7b) 8. 4 . 5 or 9/2 The slope of 2x - 3y = 8 is 2/3, and
Substitute: (10) (2) = 20 the slope of ax + by = 2 is -alb. I f the two lines are
perpendicular, then the slopes are
2 b
3. 2 y = - 4x - 3 opposite reciprocals:
3 a
Substitute x = - 1 , y = c: C = -4(- 1) - 3 a 3
Reciprocate:
Simplify: C= l b 2
Other equation : y = - 3x - b Multiply by 3: 3a 9
Substitute x = - 1 , y = 1 : 1 = -3(-1) - b b 2
Simplify: 1 =3-b 9. c Sx - y = 1 1
Subtract 3: -2 = -b 2x - 2y = 9
Divide by - 1 : 2=b Subtract equations: 3x + y = 2

4. 3.2 o r 1 6/5 Parallel lines must have equal 10. c a- b=4


slopes. The slope of 4x + Sy = 13 is - 4/S, and the slope of a + b = -7
4y + kx = 2 is -k/4.
Add equations: 2a = -3
-4 -k Divide by 2 : a = -1.S
s
- -

4
Cross-multiply: -Sk = - 1 6 Substitute a = - 1 . S: - 1 .S + b = -7
Divide by - S : k = 16/S = 3.2 Add LS: b = -7 + l . S = - s . s
Evaluate product: ab = (- 1 . S) (- S . S) = 8 . 2 S
S . 7. 5 o r 1 5/2 Perpendicular line have slopes that
are opposite reciprocals. The slope of 4x + Sy = 13 is 1 1 . B L e t c = t h e cost to make each o n e of Emma's
- 4/S, and the slope of 6y - kx = 6 is k/6. bracelets. S m - Sc = 60
-4 -6 4m - Sc = 10
s
- -

k Subtract: m = SO
Cross-multiply: -4k = -30 Substitute m = SO S(SO) - Sc = 60
Divide by -4: k = 7. S Simplify: 2SO - Sc = 60
Subtract 2SO: - S c = - 190
or
6 . • 25 1.4 First equation : 2a 1 Divide by - S : C = 38
b 3
a 1
Divide by 2:
b 6 Calculator
c s
Second equation: -+l=­
b 3 1 2 . 2/7 o r .286 o r . 2 8 5 2y = x + 1
Subtract 1 : c 2 Subtract 1 : 2y - 1 = X
b 3 Given: 4x + 6y = 0
b 3 Substitute x = 2y - 1 : 4(2y - 1) + 6y = 0
Reciprocate:
c 2
Distribute: By - 4 + 6y = 0
Multiply:
CHAPTER 7 I THE SAT MATH TEST: THE HEART OF ALGEBRA 321

Simplify: 14y - 4 = 0 x+y


17. 56 = 14
Add 4: 14y = 4 2
D ivide by 14: y = 4/14= 2/7 Multiply by 2: x + y = 28
I f x is doubled and y is tripled, the average
4
1 3 . 1 /6 or . 1 66 or . 1 67 6x 7y = - remains the same: 2x + 3y
5 = l4
2
6 Multiply by 2: 2x + 3y = 28
6x - 7y = -
5 Previous equation: x + y = 28
Add equations: l 2x = 2 Multiply by 3: 3x + 3y = 84
Divide by 1 2 : x = 2/ 1 2 = 1 /6 Other equation: 2x + 3y = 28
Subtract equations: X = 56
14. 25 The slope of 2x - 5y = 20 is 2/5. The slope of
lOx - 25y = 4k is 10/25 = 2/5. Since the two lines have 18. 24 7m + lOn = 7
the same slope, they have no points of intersection 6m + 9n = 1
unless they are the same line. 2x - 5y = 20 Subtract equations: m+n=6
l Ox - 25y = 4k Multiply by 4: 4m + 4n = 24
Multiply first equation by 5: lOx - 25y = 100
Therefore, 4k = 100 and so k = 25. 19. D Line a contains the points (2, 2) and (7, l}; there-
2-l 1
fore, it has a slope of
-- = - - . I f line b is perpendicular
15. 16 I f the original ratio of cats to dogs is 4 to 2-7 5
5, then we can say there were 4n cats and 5n dogs to line a, then it must have a slope of 5 (the opposite
to start. At the end of the week, therefore, there reciprocal of - 1 /5). You might find it helpful to sketch
were 8n cats and 5n + 1 2 dogs. I f this ratio was 1 : 1 , the line with slope 5 through the point (2, 2), and confirm
then 8n = 5n + 12 that is passes through the point (3, 7), which is one unit
to the right and one 5 units up.
Subtract Sn: 3n = 1 2
Divide by 3: n=4
Therefore, there were 4n = 4( 4) = 1 6 cats at the beginning 20. C I n order for two lines in the xy-plane to have no
of the week. points in common, they must be parallel and noniden­
tical. The only two such lines among these choices are
2x - 3y = 1 and 6x - 9y = 2, which both have a slope of
1 6. 40 Let x = the number of friendship bracelets 2/3, but have different y-intercepts of - 1 /3 and - 2/9.
Emilie had to start. This means that Jenny originally had
2x bracelets. After Jenny gave 5 of them to Emilie, Jenny 1 1 1
had 2x - 5 and Emilie had x + 5 . If Jenny still had 1 0 more 21. A -+-=­
x 2y y
than Emilie, then 2x - 5 = 10 + (x + 5)
Multiply by 2xy: 2y + X = 2X
Simplify: 2x - 5 = x + 1 5
Subtract x: 2y = x
Subtract x a n d add 5 : X= 20
Divide by 2: 1
Th i s means that Jenny h a d 2 x = 2 (20) = 40 t o start. y = -X
2
This line has a slope of 1 /2, so the perpendicular must
have a slope of -2.
TH E SAT MATH TEST:
P RO BL EM SOLVI N G
AN D DATA ANALYS I S

l. Wo rki n g with Data 324

2. Wo rki n g with Rates, Ratios, Pe rc e n ta g es, a n d P ro po rti o n s 334

3. Worki n g with Ta b l es of Data 3 49

4. Wo rki n g with G ra p h s o f Data 356

322
CHAPTER 8 / THE SAT MATH TEST: PROBLEM SOLVING AND DATA ANALYSIS 323

The SAT Math: Problem Solv i ng


and Data Analys i s
Why is problem-solving and data analysis important on the SAT Math test?

About 26% (15 out of 58 points) of the SAT Math questions fall under the category of Problem Solving and D ata
Analysis. Questions i n this category test your ability to
create a representation of a problem, consider the units involved, attend to the meaning of quantities, and
[apply reasoning about} ratios, rates, and proportional relationships.
They also assess your skill in
interpreting and synthesizing data, [as well as identifying] quantitative measures of center, the overall pat­
tern, and any striking deviations from the overall pattern in different data sets.
The specific topics i nclude
using rates, ratios, and proportional relationships to solve problems
evaluating and a nalyzing data gathering methods
calculate and use statistics of "central tendency" like mean, median, and mode
basic measures of data "spread" such as standard deviation, range, and confidence intervals
solving problems concerning percentages and percent change
analyzing scatterplots, pie graphs, tables, histograms, and other graphs
exploring linear, quadratic, and exponential relationships in data

How are these skills used?

Analyzing and d rawing inferences from data are core skills not only in mathematics and the physica l sci­
ences, but also in social sciences such as psychology, sociology, and economics. Since these subjects constitute
a substantial portion of any liberal arts curriculum, colleges consider these to be essential college preparatory
skills.

Sound intimidating? It's not.

If you take the time to master the four core skills presented in these 16 lessons, you will gain the knowledge and
practice you need to master SAT Math problem-solving and data analysis questions.
324 McGraw-Hill Education: SAT

Ski l l 1 : Worki ng with Data


Lesson 1 : Worki ng with averages (a rith metic means)

Th e average (arithmetic mean) o f fo u r numbers i s sum of these numbers: 15 x 4 = 60. I f one of these num­
15. I f o n e o f t h e numbers is 18, what is t h e average bers is 1 8, then the sum of the remaining three numbers is
(arithmetic mean) of the remaining three numbers? 60 - 1 8 = 42. Using the first formula above gives us an
average of 42/3 = 14.

I I I I I Ms. Aguila's class, which has 20 students, scored a n


average o f 9 0 % on a test. M r. B owie's class, which has
30 students, scored an average of 80% on the same
test. What was the combined average score for the
The average (arithmetic mean) of a ny set of two classes? (Disregard the % symbol when grid­
numbers i s calculated with the formula ding. For instance, enter 74% as 74.)
sum
average =
I I
----­

# of numbers

But it i s helpful to notice that this for m can


take two other forms: (Medium) Can we j ust take the average of the scores
for the two classes, and say the overall average is
sum = average x # of numbers (90 + 80)/2 = 85? No, because there are more students in the
sum second class, so we can't "weigh" the two classes equally.
# of n umbers = --­

average Using the formula above we can calculate the sum of all
of the scores in both classes. In Ms. Aguila's class, the sum
of the scores is 90 x 20 = 1,800, and the sum of the scores
in M r. Bowe's class is 80 x 30 = 2,400. Therefore, the sum
(Medium) I n this problem, we are given the average of of all of the scores in the two classes combined is 1,800 +
the set and the number of numbers in the set. So we can 2,400 = 4,200. Since there are 50 students altogether in the
use the formula sum = average x # of numbers to find the two classes, the combined average is 4,200/50 = 84.
CHAPTER 8 I THE SAT MATH TEST: PROBLEM SOLVING AND DATA ANALYSIS 325

Lesson 2: Working with medians a nd modes

The median of l , 6, 8, and k is 5. What is the average


Roll Frequency
(arithmetic mean) of these four numbers?
1 10

I I I I 2
3
a
b

The median of any set of numbers is the number 4 7


that divides the ordered set into two equal sets. 5 9
In other words, half of the numbers should be
less than or equal to the median, and half the 6 9
numbers should be greater than or equal to the
median. To find a median, The table above shows the results of 50 rolls of a die,
1 . Put the numbers in increasing (or decreasing ) with two missing values labeled a and b. I f the mode
order. of these 50 rolls is 2, what is the greatest possible
2. If there are an odd number of numbers, the average (arithmetic mean) value of these rolls?
median is the middle number.
3. If there are an even number of numbers,
the median is the average of the two middle
numbers.
I I I I I
The mode of a set of numbers is the number that
(Medium) Using this definition we can find the value of
appears the most frequently. This means that not
k. The tricky part is step 1, since we don't know where
every set of numbers has a mode. For instance, in
the set 1, 1 , 2, 3, 4, the mode is 1, but the set 1, 2,
k should be when we put the numbers in order. Clearly,
however, there are only four possibilities to consider.
3, 4 does not have a mode, because every num­
bers occurs once.

(Hard) This data set has 50 numbers, each representing


If k is the least of these numbers, then the correct order­ a roll of a die. I f the mode is 2, then 2 is the most fre­
ing is k, 1, 6, 8. Since there are an even number of numbers, quent roll . Since the table above shows that the highest
the median is the average of the middle two: ( 1 + 6)/2 = 3.5. known frequency is 10 (for a roll of 1), then a (the number
But this contradicts the given fact that the median is 5, so of times a 2 was rolled ) must be at least 1 1 . We also know
that doesn't work. Putting k in the next slot gives us an order that the total number of rolls is 50, so 10 + a + b + 7 + 9 +
of 1, k, 6, 8. In this case, the median would be (k + 6)/2. 9 = 50, and therefore a + b = 1 5 . The question asks u s
k+6 t o fi n d t h e greatest possible a verage of these rolls, so we
-- = 5 want to maximize the sum of all of the rolls. This means
2
Multiply by 2: k + 6 = 10 that we want b (the number of times a 3 was rolled ) to be
Subtract 6: k=4 as great as possible. Since b = 1 5 a, then the greatest b
-

can be is 1 5 - 1 1 = 4. Therefore a = 1 1 and b = 4. Now we


Notice that this confirms our assumption that k is have to find the average of these 50 numbers: [(1)(10) +
between 1 and 6, so k must equal 4. Now we must find the (2) ( 1 1) + (3)(4) + (4)(7) + (5)(9) + (6)(9)]/50 = (10 + 22 +
average of these four numbers: ( 1 + 4 + 6 + 8)/2 = 1 9/2 = 9.5. 1 2 + 28 + 45 + 54)/50 = 3.42.
326 McGraw-Hill Education: SAT

Lesson 3: Understandin data s read

Th e SAT Math test may occasionally ask you about the "spread" o f a set o f data. You will NOT h ave t o calculate
technical statistics like variance, standard deviation, or margin of error, but you m ight be asked to answer
more basic questions about the "spread" of a set of data, as i n the questions below.

The "range" of a set of data is defined as the absolute (Medium) When a question introduces a new term, read
difference between the least value and the greatest its definition carefully- several times, if necessary. This
value in the set. I f five positive integers have an aver­ question gives us two new terms. To understand these
age (arithmetic mean) of 10, what is the greatest pos­ terms, let's apply them to a simple set of numbers, such
sible "range" of this set? as the set i n choice (A). The average of this set is 2; there­
fore, the "absolute deviation" of each of these numbers is

I I I I I
its "absolute difference" from 2, which is 0 for each num­
ber. The "average absolute deviation" of the entire set is
defined as the average of these "absolute deviations,"
(Medium-hard) I f five numbers have an average of 10, which is, of course, (0 + 0 + 0 + 0 ) /4 = 0.
then their sum must be 5 x 10 = 50. I f we want the great­ Notice that the "average absolute deviation" of a
est possible "range," then we must maximize one of these number set is a measure of "spread." Since the numbers
numbers by minimizing the sum of the other numbers. in (A) are bunched up as tightly as possible, their "aver­
Since the smal lest positive integer is l , we can minimize age absolute deviation" is 0. Now let's look at the remain­
the sum of the other four numbers by setting them all ing choices. Which seems to h ave the greatest "spread? "
equal to 1. This gives us 1 + 1 + 1 + 1 + x = 50, so x = 46. Once you've made your guess, do the calculations and
This gives us a maximum "range" of 46 - 1 = 45. see if you're right. For confirmation, you should find that
the "average absolute deviations" are (A) 0, (B) 1 , (C) 0.5,
The "absolute deviation" of a number in a set is the and (D) 0.375. Therefore, the correct answer is (B).
absolute d i fference between that number and the
average (arithmetic mean) of the set. The "average
absolute deviation" of a set is the average (arithme­
tic mean) of all of the absolute deviations in the set.
Which of the following sets has the greatest "aver­
age absolute deviation? "
A) 2, 2, 2, 2
B) 2, 3, 4, 5
C) 4, 4, 5, 5
D) 4, 4, 4, 5
CHAPTER 8 / THE SAT MATH TEST: PROBLEM SOLVING AND DATA ANALYSIS 327

Lesson 4: Variations and drawing inferences from data

The variables x and y vary directly if they have a constant ratio, that is,

r = k or y = kx (where k is a constant)
x

The variables x and y vary inversely if they have a constant product, that is,

k
xy = k or y = - (where k is a constant)
x

x y If y varies inversely as x, then the graph of their


1 5 relation in the xy-plane is a hyperbola that
approaches, but does not touch, the x- and
2 20 y-axes:

Given the ordered pairs in the table above, which of y


the following could be t rue?
A) y varies directly as x
B) y varies inversely as x
C) y varies directly as the square of x 9
3 \ y= -
D) y varies i nversely as the square of x • , ... x
\

2 • 4-
(Medium-hard) Using the definitions above, we can see y= ----
whether y and x vary directly or i nversely. Do they have -
x -
a constant ratio? No: 5/ 1 "" 20/2. Therefore, they do not x

vary directly and (A) is incorrect. Do they have a constant 2 3


product? No: 1 x 5 "" 2 x 20. Therefore, they do not vary
inversely, a nd (B) is incorrect. To check (C), we must ask:
do y and x2 have a constant ratio? Yes: 5/(1)2 = 20/(2)2 = 5, x y
therefore the correct answer is (C).
2 12

I f y varies directly as x, then the graph of their 4 96


relation in the xy-plane is a line through the
origin: The variables x and y are related by a n equation of
the form y = Axn where A and n are both positive
y real numbers. Based on the data in the table above,
y = 4x what is the value of y when x = 3?
y = 2x A) 40.5
I
I y=x B) 46.0
I
I

I
I
C) 54.0
I
I

I
I
y = O.Sx
f I
I
I'
/
/
D) 64.0
I I /
I I /
/J / /� ,,,,. .,...
(Medium) I n order to find the value of y when x = 3, we
";;. ....
x must find the specific equation relating x and y. We can
find the values of A and n in the equation y = Axn by first
328 McGraw-Hill Education: SAT

plugging in the values from the table. Plugging in the first (Notice that this means that y varies directly as x3.)
ordered pair gives us 12 = A (2) n and plugging in the sec­
ond ordered pair gives us 96 = A (4) n . Substitute n = 3 into either
equation: 1 2 = A (2)3
96 = A(4) n
1 2 = A(2) n Simplify: 12 = BA
Divide by 8: 1.5 = A
96 4"
Divide the corresponding sides:
12 2" Therefore, the equation that relates x and y is y =
Simplify: 8 = 2n l . 5x3. Finally, we find y when x = 3 by substituting into
the equation: y = 1 . 5 (3)3 = 1 . 5 (27) = 40.5, so the correct
Substitute 8 = 23: 23 = 2 n and so n =3
answer is (A).
CHAPTER 8 / THE SAT MATH TEST: PROBLEM SOLVING AND DATA ANALYSIS 329

E xer�ise Set 1 ( N o Calcu lator)

• •
The "range" of a set of data is defined as the absolute If p varies i nversely as q and p = 4 when q = 6, the
difference between the least value and the greatest which of the following is another solution for p and q?
value i n the set. Four positive integers have a n aver­
A) p = 8 and q = 1 2
age (arithmetic mean) of 7. 5.
B) p = 8 and q = 1 0
a. What is the greatest possible range of this set?
C) p = 12 and q = 1

I I I I I D) p = 1 2 and q = 2

b . What is the least possible range of this set?


g
I I I I I A set of n numbers has a n average (arithmetic mean)
of3k and a sum of 1 2m, where k and m are both posi­
tive. Which of the following is equivalent to n ?
II A)
4m
B)
4k
C)
k
I f the median of 2, 4, 6, and b is 4.2, what is the aver­ k m 4m
age (arithmetic mean) of these four numbers?
m

I I I I I
D)
4k


• I f y varies inversely as the square of x, then when x is
The average (arithmetic mean) of 2, 5, 8 and k is 0. multiplied by 4, y will be
What is the median of these numbers? A) divided by 16

I I I I I
B) divided by 2
C) multiplied by 2
D) multiplied by 1 6

A set of numbers has a sum of 48 and an average of 6 .
H o w many numbers a r e i n t h e set?
1111
Letf(x, y) = Ax2y3 where A is a constant. I ff(a, b) = 10,

I I I I I
what is the value off(2a, 2b)?
A) 100
B) 260
..
C) 320
I f the average (arithmetic mean) of 4 and x is equal
to the average (arithmetic mean) of 2, 8, and x, what D) 500
i s the value of x?


I I I I I A set of four integers has a mode of 7 and a median of
4. What is the greatest possible average (arithmetic
• mean) of this set?
The median of a set of 22 consecutive even integers A) 3.50
is 25. What is the l argest number in the set? B) 3 .75

I I I I I
C) 4.00
D) 4.25
330 McGraw-Hill Education: SAT

Exe rc ise Set 1 (Ca lc u la tor)

Four positive i ntegers have a mode of 4 and a median I f y varies i nversely as x and the graph of their rela­
of 3 . What is their sum? tion i n the xy-plane passes through the point (2, 1 5),
what i s the value of y when x = 4?

I I I I I I I I I I
Ill Ill
Five different integers have a n average (arithmetic
mean) of 10. I f none is less than 5, what i s the great­ Roll Frequency
est possible value of one of these i ntegers?
1 4

I I I I I
2 5
3 4
4 6


5 5
6 6
I f b varies i nversely as a, and b = 0.5 when a = 32,
then for how many ordered pairs (a, b) are a and b
both positive i ntegers? A six-sided die was rolled 30 times and the results
tabulated above. What i s the d i fference between

I I I I I the average (arithmetic mean) of the rolls and the


median of the rolls?
A) 0 . 1

Ill B) 0.2
The median of 1 1 consecutive integers i s 28. What is C ) 0.3
the least of these integers? D) 0.4

I I I I I
I f y varies i nversely as the square of x, and y = 4
• when x = 2, then what i s the value of y when x = 3?
I fy = Ax3 and y = 108 when x = 3, then for what value 16
A)
of x does y = 62.5? 9

I I I I I B) 8
3
C) 3
D) 9

A set of four positive i ntegers has a median of 2 and
a mode of 2 . I f the average (arithmetic mean) of this
set is 3, what is the largest p ossible number i n the
set?
CHAPTER 8 I THE SAT MATH TEST: PROBLEM SOLVING AND DATA ANALYSIS 331

• m
At a fixed temperature, the volume of a sample of If the graph of y = J(x) i n the xy-plane contains the
gas varies i nversely as the pressure of the gas. If the points (4, 3) and (16, 6), which of the fol lowing could
pressure of a sample of gas at a fixed temperature be t rue?
is increased by 50%, by what percent is the volume
A) y varies directly as the square of x
decreased?
B) y varies inversely as the square of x
A) 25%
C) y varies directly as the square root of x
1
B) 33 - % D) y varies inversely as the square root of x
3
C) 50%
2
D) 66 - %
3
332 McGraw-Hill Education: SAT

EXE RCISE SET 1 ANSWE R KEY

No Calculator 10. C f(a, b) = Aa2b3 = 10. f(2a, 2 b) = A(2a)2(2b)3 =


32(Aa 2 b3 ) = 32(10) = 320.
l a. 26 I f the average of 4 numbers is 7. 5, they must
h ave a sum of 4 x 7.5 = 30. To maximize the range, we
must maximize one of the numbers by minimizing the 1 1 . B I f this set has a mode of 7, then at least two of the
other 3 by setting them all equ a l to 1 (the smallest posi­ numbers are 7. I f the median is 4, then the two middle
l,
t ive i nteger) . The numbers therefore are l , l, and 27, numbers must have a sum of (2)(4) = 8. Therefore the two
and the range is 27 - 1 = 26. middle numbers are 1 and 7, and the sequence must b e n,
l, 7, 7. To maximize the average, we must maximize n,
but n c an't be l, because then the set would not h ave a
l b. 1 To minimize the range, we "cluster" the num­
mode of 7. It must b e the next lower integer, 0, and the
bers as closely together as possible. The tightest cluster
average is (0 + 1 + 7 + 7)/4 = 3.75.
of i ntegers with a sum of 30 is 7, 7, 8, and 8, which gives a
range of 8 - 7 = 1 .

2 . 4.1 I f the set contains fou r numbers, its median is Calculator


the average of the middle two numbers, so the middle
two numbers must have a sum of (2)(4.2) = 8.4. Thus the 1 2 . 11 The only fou r numbers that satisfy these condi­
four numbers must be 2, 4, 4.4, and 6 . (Notice that tions are 1 , 2, 4, and 4.
the question did not say that all numbers were i ntegers.)
The average of these is 16.4/4 = 4 . 1 . 1 3 . 24 I f the average of five numbers is 10, their sum
is 5 x 10 = 50. To maximize one, we must minimize the
3 . 3 . 5 I f t h e average o f these numbers is 0 , their sum of the other four. I f none is less than five, and all are
sum must b e (4)(0) = 0, and t herefore k = -15 and different i ntegers, they are 5, 6, 7, 8, and 24.
the numbers, i n increasing order, are - 1 5, 2, 5, and 8. The
median is (2 + 5)/2 = 3 . 5 . 14. 5 If the variables vary inversely, their product is
constant. (0.5)(32) = 16. The only pairs of positive inte­
4. 8 6 = 48/n, so n = 8. gers with a product of 16 are ( 1 , 1 6), (2, 8), ( 4, 4), (8, 2), and
( 1 6, 1 ) .

5. 8 4+x 2+8+x
2 3 15. 2 3 If t h e middle number is 28, there are five num­
bers less than 28, and five greater. Since they are consec­
Cross-multiply: 1 2 + 3x = 20 + 2x
utive i ntegers, the least is 28 - 5 = 23.
Subtract 2x and 1 2 : x=8

6 . 46 The median divides the set into two equal 16. 2.5 Since 1 08 = A(3)3, A = 4, so if 62.5 = 4x3, x = 2.5.
parts, so 1 1 of these numbers must b e less than 25 and
1 1 must b e greater than 25. Since they are consecutive 17. 7 At least two of the i ntegers must b e 2 and none
even i ntegers, the 1 1 numbers above the median must be can be less than 1. I f the sum must be 4 x 3 = 1 2, the set
26, 28, 30, 32, . . . 46. i ncluding the largest possible number is 1 , 2, 2, and 7.

7. D I f p and q vary i nversely, their product is a con­ 18. 7.5 The product of x and y is 2 x 1 5 = 30, so y =
stant. 4 x 6 = 24, and the only other pair with a product 30/4 = 7.5.
equal to 24 is (D) 12 and 2.
19. C Average = (1 x 4 + 2 x 5 + 3 x 4 + 4 x 6 + 5 x 5 +
8. A n = sum/average = 1 2m/3k = 4m/k 6 x 6)/30 = 3 .7. Median = average of 15th and 16th rol l :
(4 + 4)/2 = 4 . 4 - 3 .7 = 0 . 3 .
9. A The equation relating x and y is y = k/x2• I f x = l,
then y = k. I f x is multiplied by 4, then x = 4 and y = k/16, 2 0 . A y a n d x 2 must h ave a constant product o f
so y has been d ivided by 16. 4 x 22 = 1 6 . Therefore, y = 16/9.
CHAPTER 8 I THE SAT MATH TEST: PROBLEM SOLVING AND DATA ANALYSIS 333

2 1 . B Pick values for the original pressure and volume, y


such as 2 and 3. I f they vary i nversely, their product is 22. C For both ordered pairs, - is a constant:
£
the constant 2 x 3 = 6 . I f the pressure is increased by
50%, it becomes ( 1 . 5) (2) = 3, and so the volume becomes J4
3
= Jl6 = "2' so y is directly proportional to the
6 3

6/3 = 2, a change of -33 1 /3%. square root of x .


334 McGraw-Hill Education: SAT

Ski l l 2 : Working with Rates, Ratios,


Percentages, a nd Proportions
Lesson 5 : Rates and u n it rates

On a sunny day, a 50 square meter section of solar households to a p articular number of square meters of
p anel array can generate an average of 1 kilowatt­ solar panels:
hour of energy per hour over a 1 0-hour period. If an
average household consumes 30 kilowatt-hours of 30 kwh/day 1 day
1 , 000 households x x x
energy per day, how large an array would be required 1 household 1 0 sun-hours
to power 1 , 000 households on sunny days?
50 square meters
- � ---- = 1 50 , 000 square meters
A) 1,500 square meters 1 kwh/sun-hour
B) 15,000 square meters
C) 1 50,000 square meters Note very carefully how (1) all of the u n its on the left
D) 15,000,000 square meters side of the equation c ancel except for "square meters"
(which is what we want), and (2) each conversion factor
(Medium) This is clearly a "rate problem," because it
represents a n explicit fact mentioned i n the problem.
i ncludes two "per" quantities. When working with rates,
keep two important ideas i n mind:

M a ny rate problems can be easily managed with


Th e units fo r any rate c a n be translated t o give the "rate pie":
the formula for the rate. For instance, if a word
problem i ncludes the fact that "a rocket burns
fuel at a rate of 15
kilograms per second," this
fact can be translated i nto a formula as long as
we remember that per means divided by:

# kilograms of fuel
rate ( of fuel burning) = ------­
# seconds

Any "rate fact" in a problem can be inter­


preted as a "conversion factor." For instance, i f
15
" a rocket burns fuel at a rate o f kilograms per
second," then i n the context of that problem, one This is a simple graphical device to organize
second of burning is equivalent to 15
kilograms i nformation i n a rate problem. It is s imply a way
of fuel being burned. Therefore, as we d iscussed of expressing all three forms of the "rate equation"
i n Chapter 7, Lesson 4, we are entitled to use at once: distance = rate x time; rate = distance/
either of the following conversion factors i n this time; and time = distance/rate. For example, i f a
problem: word problem states that "Maria completes a n
x-mile b icycle race at an average speed of z miles
15 kilograms
--
1 second -
---=-- or ---- per hour," your "rate pie" should look like this:
1 second 15
kilogram

x miles
Just as we did in Chapter 7, Lesson 4, we can solve
this problem by j ust noticing that it is essentially a
conversion problem. The question asks " how large an
array (in square meters) would b e required to p ower
1 , 000 households on sunny d ays?" So we can treat the
problem as a conversion from a p articular number of
CHAPTER 8 I THE SAT MATH TEST: PROBLEM SOLVING AND DATA ANALYSIS 335

which still leaves 3,400 - 1,600 = 1,800 gallons left.


First, we plug the given values in: x miles goes in Notice that we have a lready taken 3 hours and 20 min­
for distance, and z miles per hour goes i n for rate. utes, and as yet have not finished pumping. This means
Then, as soon as two of the spaces are filled, we that choices (A) and (B) are certainly i ncorrect. So how
simply perform the operation between them long will it take to pump the remaining 1 , 800 gallons?
(i n this case division) and put the result in the Now that we can bring the stronger pump online, it will
final space. In this case, the time Maria took to only take 1,800 gallons x ( 1 m inute/ 180 gallons) = 10
complete the race was x/z hours. more minutes; therefore, the correct answer is (D).
Although you don't need to construct a graph of this
situation to solve the problem, graphing helps show the
overal l picture:
A water pump for a dredging project can remove 180
gallons of water per minute, but can work only for 3
consecutive hours, at which time it requires 20 min­
utes of maintenance before it can be brought back
36,000 - ��·�99 - - - - - - - - - - - - - - - - - - - - .
online. While it is offl i ne, a smaller pump is used I

in its place, which can pump 80 gallons per minute. <Jl 27,000 \so: gal/ min
\ :
Using this system, what is the least amount of time it c::
.s
would take to pump 35,800 gallons of water? (ii 1 8,000
CJ
1 80 gal/mm I

A) 3 hour 10 minutes
9,000
B) 3 hours 15 minutes
C) 3 hours 25 minutes 0
0 60 1 20 180 240
D) 3 hours 30 m inutes
Minutes
(Hard) I f we want to pump out the water as quickly as
possible, we want to use the stronger pump for the max­
imum three hours. To find the total amount of water Notice that the l i ne has a slope of 180 for the first 180
pumped in that time, we do the conversion: minutes, then 80 for the next 20 minutes, and then 180
for the next 180 mi nutes, and crosses the l ine y = 35,800
1 80 gallons
60 minutes -�-- at 2 1 0 minutes.
3 h ours x x 32, 400 gallons
1 hour 1 minute

So after 3 hours, there are still 35,800 - 32,400 = 3,400


I n the graph of a ny linear function, y i n terms
gallons left to pump. At that point, the smaller pump must
be used for a minimum of 20 minutes, which can pump of x , the slope of the line is equivalent to the
unit rate of the function, that is, the rate at
which y increases or decreases for every unit
80 gallons
20 minutes x = 1 , 600 gallons increase in x.
l minute
336 McGraw-Hill Education: SAT

Lesson 6: Ratios: pa rt-to-pa rt a nd pa rt-to-whole

A marathon offers $5,000 in p r i z e money to t h e top


three finishers. If the first-, second-, and third-place part by the sum. For instance, if a p aint mixture
prizes are d istributed in a ratio of 5:4 : 1 , how much is a 2:5 combination of red and yellow, respec­
money, in doll ars, does the second-place finisher tively, the "whole" is 2 + 5 = 7, which means that
receive? the mixture is 2/7 red and 5/7 yellow.

I I I I I
Bronze is an a lloy (a metallic mixture) consisting of
copper and tin. If 50 kg of a bronze a lloy of 20% tin
and 80% copper is mixed with 70 kg of a bronze a lloy
(Easy) When given a "part-to-part" ratio, such as 5:4: 1 of 5% tin and 95% copper, what fraction, by weight,
(which is of course, really a part-to-part-to-part ratio), it of the combined bronze a lloy is tin?
often helps to add up the parts and consider the whole. A) 5/48
This prize is divided into 5 + 4 + 1 = 10 equal parts, so
B) 9/80
the winner gets 5/ 10 of the prize money, the second-place
finisher gets 4/10 of the prize money, and the third-place C) 1 /8
finisher gets 1 / 10 of the prize money. The second-place fin­ D) 1 /4
isher therefore takes home (4/ 10) x $5,000 = $2,000.
(Medium) The combined alloy will weigh 50 kg + 70 kg =
1 20 kg. The total weight of the tin comes from the two
If you are given a part-to-part ratio, it is often separate a lloys: (0.20) (50) + (0.05)(70) = 10 + 3.5 = 1 3 . 5 kg.
helpful to add up the parts and then divide each Therefore the fraction of the combined a lloy that is tin is
1 3 . 5 / 1 20, which simplifies to 9/80.
CHAPTER 8 / THE SAT MATH TEST: PROBLEM SOLVING AND DATA ANALYSIS 337

Exercise Set 2 (Ca lcu lator)

• ..
I f a train travels at a constant rate of 50 miles per A motorcycle has a fuel efficiency of 60 miles per gal­
hour, how many m inutes will it take to travel 90 lon when it is cruising at a speed of 50 miles per hour.
miles? How many hours can it travel at 50 miles per hour on
a full tank of gas, if its tank can hold 10 gallons?

I I I I I I I I I I

Two cars leave the same point simultaneously, going •
in the same direction along a straight, flat road, one I f the ratio of a to b i s 3 to 4, and the ratio of a to c i s
at 35 miles per hour and the other at 50 miles per 5 to 2, what is t h e ratio of b to c?
hour. After how many minutes will the cars be 5
A) 3 to 10
miles apart?
B) 3 to 5

I I I I I C) 5 to 3
D) 10 to 3

..
I f a $6,000 contribution is divided among chari­ ..
ties A, B, and C i n a ratio of 8:5:2, respectively, how A paint mixture consists of a 3:2: 1 1 ratio ofred, violet,
much more, i n dollars, does charity A receive than and white, respectively. How m a ny ounces of violet
charity C? are needed to m a ke 256 ounces of this mixture?

I I I I I
A ) 32
B) 36
C) 46
• D) 48
I f a car traveling at 60 mph is chasing a car travelling
at 50 mph and is 114 mile behind, how many minutes
will it take the first car to catch the second? •
A pool that holds 20,000 gallons is 114 full. A pump
I I I I I can deliver g gallons of water every m m i nutes. If
the pumping company charges d dollars per min­
ute, how much will it cost, i n dollars, to fil l the pool?
.. 5, 000 md
A)
A truck's gas tank can hold 1 8 gallons. I f the tank is g
2/3 full and the truck travels for 4 hours at 60 miles 5, 000 gd
per hour until it runs out of gas, what is the efficiency B)
m
of the truck, i n miles per gallon?
1 5 , 000 md
C)

I I I I I
g
5, 000 gd
D)
m
338 McGraw-Hill Education: SAT

Ill
Yael travels to work at an average speed of 40 miles 2a 1 c 1 . a
I f - = - and - - -, what is -?
per hour and returns home by the same route at 24 3b 5 2b 2 c

miles per hour. I f the total time for the round trip is
2 hours, how many miles is her trip to work?
A) 25 I I I
B) 30
C) 45
D) 60 I f a cyclist races at 30 miles per hour for 1/2 of the
distance of a race, and 45 miles per hour for the fi nal
1/2 of the distance, what is her average speed, i n
ID miles p e r hour, for t h e entire race?

I I I
A hare runs at a constant rate of a miles per hou r,
and a tortoise runs at a constant rate of b miles per
hour, where 0 < b < a. How many more hours will
it take the tortoise to finish a race of d miles than
the hare? ..
Anne can paint a room in 2 hours, and Barbara can
a+b paint the same room in 3 hours. I f they each work
A) --

2d the same rate when they work together as they do


ad - bd alone, how many hours should it take them to paint
B)
ab the same room if they work together?
b-a
C) --

d
ab - bd
D)
ad

What is the average speed, in miles per hour, of a


sprinter who runs 1/.i mile i n 45 seconds? (1 hour =
Janice can edit 700 words per minute and Edward
60 seconds)
can edit 500 words per minute. If each page of text
contains 800 words, how many pages can they edit, A) 1 1 . 25
working together, in 20 minutes? B) 13.5

I I I I
C) 20
D) 22

Ill
I f a printer can print 5 pages in 20 seconds, how
Ill
many pages can it print i n 5 minutes? A car t ravels d miles i n t hours and arrives at its des­
tination 3 hours late. At what average speed, i n miles

I I I I I per hour, should the car h ave gone i n order to arrive


on t ime?

t-3
Ill A) --

d
Traveling at 40 miles per hour, Diego can complete d
his daily commute in 45 minutes. How many min­ B)
t-3
utes would he save if he traveled at 50 miles per d
hour? C) - - 3
t

I I I I
d-3
D)
CHAPTER 8 I THE SAT MATH TEST: PROBLEM SOLVING AND DATA ANALYSIS 339

Ill
In three separate I-mile races, Ellen finished with Sylvia drove 3 1 5 miles and arrived at her destination
times of x minutes, y minutes, and z minutes, in 9 hours. I f she had driven 10 miles per hour faster,
respectively. What was her average speed, in miles how many hours would she have saved on the trip?
per hour, for all three races?
A} 1 .75 hours
x+y+z B) 2.00 hours
A)
3 C} 2 . 2 5 hours
3
B) D} 2.50 hours
x+y+z
x+y+z
C}
180
1 80
D)
x+y+z
340 McGraw-Hill Education: SAT

EXE RC ISE SET 2 ANSWE R KEY

1. 108 time = distance/rate = 90 miles/50 mph = 1 . 8 _:_ + _:_ = 2


hours = 1 .8 hour x 60 min/hour = 1 0 8 m i nutes. 40 24
Simplify: _:_ = 2
2 . 20 The fast car is moving ahead of the slow car at a 15
Multiply by 15: x = 3 0 miles
rate of 50 - 35 = 1 5 mph, and so it will be 5 miles ahead
after 5 -;- 1 5 = 1 /3 hour = 20 minutes.
1 1 . B The tortoise would take d/b hours to com­
plete the race, and the hare would take d/a hours
3. 2,400 Since 8 + 5 + 2 = 15, charity A receives 8/ 1 5 to complete the race, so the tortoise would take
o f t h e contribution, a n d charity C receives 2 / 1 5 . Th e dif­ d d ad bd ad - bd
ference is 6/ 15, or 2/5, of the total, which is (2/5)($6,000) = --- ---= hours longer.
a a ab ab ab
$2,400.
12. 30 Together they can edit 700 + 500 = 1 , 200 words
4. l.5 Since the faster car is catching up to the slower per minute, so i n 20 minutes they can edit
car at 60 - 50 = 10 mph, it will take (1/4 mile)/(10 mph) = 1 , 200 words 1 page
20 minutes x x = 30 pages
1 /40 hours = 60/40 minutes = 1 . 5 minutes. 1 minute 800 words

5. 20 The tank contains (2/3)(18) = 1 2 gallons, and 1 3 . 75 I f the printer can print 5 pages in 20 seconds,
t ravels (4 hours)(60 mph) = 240 miles, so its efficiency is it can print 1 5 pages in 1 minute, and therefore 1 5 x 5 =
240/ 1 2 = 20 miles per gallon. 75 pages in 5 minutes.

6. 1 2 With 10 gallons of gas and an efficiency of 14. 9 Since 45 minutes is % hour, Diego's daily com­
60 miles per gallon, the car can t ravel 10 x 60 = mute is 40 x % = 30 miles. If he traveled at 50 mph it
600 miles. At 50 miles an hour this would take 600/50 = would take him 30/50 = 3/5 hours = 36 minutes, so
1 2 hours. he would save 45 - 36 = 9 minutes.

2a 2 b 1 2
7. D
b -
- b a 4 5 10
= X-=-X-=- 15. 3/ 10 or 0.3 -X-=-X-
C a C 3 2 3 3b c 5 1
4a 2
8 . A According to the ratio, the mixture is 2/(3 + Simplify:
2 + 1 1) = 2 / 1 6 = 1 /8 violet. Therefore 256 ounces of the 3c 5
m ixture would contain (1 /8)(256) = 32 ounces of violet a 2 3 3
Multiply by % : -=-X-=-
paint. C 5 4 10

9. C I f the pool is V4 full, it requires (3/4)(20,000) = 1 6 . 3 6 Pick a convenient length for the race, such a s
15,000 more gallons. 1 8 0 miles (which i s a multiple o f both 3 0 a n d 45). The first
half of the race would therefore b e 90 m iles, which woul d
m minutes d dollars take 9 0 miles -;- 30 m p h 3 hours, and the second half
=

1 5 , 000 gallons x x --. --


g gallons 1 minute would take 90 miles -;- 45 mph = 2 hours. Therefore, the
1 5 , 000 md entire race would take 3 + 2 = 5 hours, and the cyclist's
= d o II ars. average speed wou ld therefore b e 180 miles -;- 5 hours =
g
36 miles per hour.

10. B Let x = the distance, in miles, from home to


17. l .2 or 6/5 Anne's rate is 1 /2 room per hour, and
work. Since time = distance/rate, it takes Yael x/40 hours
Barbara's rate is 1 /3 room per hour, so together their rate
to get to work and x/24 hours to get home.
is 1 /3 + 1/2 = 5/6 room per hour. Therefore, painting
one room should take (1 room)/(5/6 room per hour) =
6/5 hours.
CHAPTER 8 I THE SAT MATH TEST: PROBLEM SOLVING AND DATA ANALYSIS 341

0.25 mile 3, 600 seconds 3 miles 60 minutes 1 80


18 . c ---- x = 20 mp h 20. D ------- x ---- --- mp h
45 seconds 1 hour x + y + z minutes 1 hour x+y+z

1 9 . 8 In order to a rrive on time, it would have to travel 2 1 . 8 Sylvia traveled at 3 1 5/9 = 35 miles per hour. If she
the d miles i n t - 3 hours, which would require a speed had traveled at 35 + 10 = 45 miles per hour, she would
of d/ ( t - 3) mph. have arrived in 3 1 5/ 45 = 7 hours, thereby saving 2 hours.
342 McGraw-Hill Education: SAT

Lesson 7: I nterpreti ng percent problems

What number is 5 percent o f 36? Again, let's use the glossary to translate and then solve:
28 � what percent of 70?

I I I I I Simplify:
28 = (x -;- 100) x 70
x
28 = - x 70 = 0.7x
1 00
Divide by 0.7: 40 = X
When interpreting word problems, remember
that statements about quantities can usually What number is 1 20% greater than 50?
be translated into equations or inequalities.
H ere's a simple translation key:
I I
is/is equal to means
of
(l+�).
means x To increase a number by x%, just multiply it by
what/wha t n umber means x
To decrease a number by xo/o, just

(1-�).
per means 1 00
percen t means -;- 100 multiply it by For instance, to increase
100
a number by 20%, just multiply by 1 .20 (because
the final quantity i s 120% of the original quantity,
(Easy) Notice that this enables us to translate the question and to decrease a number by 20%, just multiply
i nto an equation, which can be solved to get the answer: by 0.80 (because the final quantity is 80% of the
What number � 5 percent Qf 36? original quantity).
x = (5 -;- 100) x 36 so x = 1.8

+
2 8 is what percent o f 70? (Easy) I f we increase a number by 1 20%, the resulting
number is 100% 1 20% = 220% of the original number.

I I I I Therefore, the number that is 1 20% greater than 50 is


llO.
2 . 2 0 x 50 =
CHAPTER 8 I THE SAT MATH TEST: PROBLEM SOLVING AND DATA ANALYSIS 343

Lesson 8 : Percent change

A shirt has a marked retail price of $80, but is on If a population of bacteria increases from 100 to 250,
sale at a 20% discount. If a customer has a coupon what is the percent increase in this population?
for 10% off of the sale price, and if the sales tax is
A) 60%
5%, what is the final price of this shirt, including all
discounts and tax? B) 67%
A) $58.80 C) 1 50%
B) $60.00 D) 250%
C) $60.48
D) $ 6 1 .60 To find the percent change i n a quantity, just use
(Medium) To find the final price, we must perform three the formula
changes: decrease by 20%, decrease by 10%, and increase
final amount - initial amount
by 5%. This gives us ( 1 .05)(0.90)(0.80)($80) = $60.48, so the percent ch ange = . . . x 1 oocrr
10

answer is (C). Notice that, since multiplication is com­ mztial amount


mutative, it doesn't matter in what order we perform the
three changes; the result will still be the same. Notice that any "percent change" i s a "per­
cent of the initial amount," which explains why
the initial amount s the value in the denominator.

(Easy) I f we know this formula, this question is straight­


forward: the percent change is (250 - 100)/100 x 100% =
1 50%, choice (C). I f you mistakenly use 250 as the
denominator, you would get an a nswer of (A) 60%, which
is incorrect.
344 McGraw-Hill Education: SAT

Lesson 9: Working with proportions and sca les

On a scale blueprint, the drawing of a rectangular


patio has dimensions 5 cm by 7.5 cm. I f the longer The Law of Cross-Multiplication
side of the actual patio measures 21 feet, what is the
If two ratios are equal, then their "cross­
area, in square feet, of the actual p atio?
products" must also be equal.
A) 1 57. 5 square feet
I f � = £ then ad = be.
B) 294.0 square feet b d
C ) 356.5 square feet The Law of Cross-Swapping
D) 442.0 square feet
If two ratios are equal, then their "cross­
(Medium) In a scale drawi ng, all lengths are propor­ swapped" ratios must also be equal.
tional to the corresponding lengths in real l i fe . That a c d c a b
I f - = -, then - = - and - = -.
is, the lengths in the drawing and the corresponding b d b a c d
lengths i n real l i fe shou ld a l l b e i n the same ratio. We
c a n set up a proportion here to find the shorter side of
7.5 cm 5 cm
the patio, x.
2 1 feet x feet
7.5 cm 5 cm Cross-multiply: 7. 5x = 105
2 1 feet x feet Divide by 7. 5: X = 14

Therefore, the patio has dimensions 21 feet by 14 feet,


When working with proportions, remember the two and so it has a n area of (2 1)(14) = 294 square feet. The
laws of proportions. correct answer is (B).
CHAPTER 8 / THE SAT MATH TEST: PROBLEM SOLVING AND DATA ANALYSIS 345

Exe rc ise Set 3 Ca lculator

• •
What number is 1 50% of 30? I f the price of a house i ncreased from $40,000 to
$ 1 20,000, what is the percent increase in price?

I I I I I A) 67%
B) 80%

.. C) 200%
D) 300%
If the areas of two circles are i n the ratio of 4:9, the
circumference of the larger circle is how many times
the circumference of the smaller circle?

I I I I I At a student meeting, the ratio of athletes to non
athletes is 3:2, and among the athletes the ratio of
males to females is 3:5. What percent of the students
.. at this meeting are female athletes?

What number is 30% less than 70? A) 22.5%


B) 25%

I I I I I C) 27.5%
D) 37.5%


What number is the same percent of 36 as 5 is of 24? B

I I I I I
To make a certain purple dye, red dye and blue dye
are m ixed in a ratio of 3:4. To make a certain orange
dye, red dye and yellow dye are mixed in a ratio of 3:2.
If equal a mounts of the p urple and orange dye are
• mixed, what fraction of the new mixture is red dye?

A)
David's motorcycle uses 2/5 of a gallon of gasoline 9
to t ravel 8 miles. At this rate, how many miles can it 20
travel on 5 gallons of gasoline?
B)
2

I I I I C)
18
35
D)
27
.. 40
The retail price of a shirt is $60, but it is on sale at
a 20% discount and you h ave a n additional 20% off
coupon. l f t here is also a 5% sales tax, is the final cost
IJil
of the shirt? If the price of a stock declined by 30% in one yea r
a n d increased b y 8 0 % the next year, b y what percent
A) $34.20 did the price increase over the two-year period?
A) 24%
B) $36.48
C) $37.80
B) 26%
D ) $40.32
C) 50%
D) 500
346 McGraw-Hill Education: SAT

• •
A farmer has a n annual budget of $ 1,200 for barley I f the width of a rectangle decreases by 20%, by what
seed, with which he can plant 30 acres of barley. If percent must the length increase i n order for the
next year the cost per pound of the seed is proj ected total area of the rectangle to double? (Ignore the %
to decrease by 20%, how many acres will he b e able symbol when gridding.)
to afford to plant next year on the same budget?
A) 24
B) 25
I I
C) 36
Ill
D) 37. 5
Two middle school classes take a vote on the desti­
nation for a class trip. Class A has 2 5 students, 56%
of whom voted to go to St. Louis. Class B has n stu­
• dents, 60% of whom voted to go to St. Louis. I f 57. 5%
I f x is 3_ % of 90, what is the value of 3_ x?
_
of the two classes combined voted to go to St. Louis,
3 3 what is the value of n?

I I I I I I
Ill
I f n is 300% less than � , what is the value of l nl?
Ill
2 I f 1 2 ounces of a 30% salt solution are mixed with

I I I
24 ounces of a 60% salt solution, what i s the percent
concentration of salt i n the mixture?
A) 45%
ID B) 48%
The cost of a pack of b atteries, after a 5% sales tax, C) 50%
i s $8.40. What was the price before tax, in dollars? D) 54%

I I I I
I f the length of a rectangle i s doubled but its width
Ill is decreased by 10%, by what percent does its area
I f the price of a sweater is marked down from $80 increase?
to $68, what i s the percent discount? (Ignore the % A) 80%
symbol when gridding.)
B) 90%

I I I I I C) 180%
D) 190%

Ill
Three numbers, a, b, and c, are all positive. I f b is
30% greater than a , and c i s 40% greater than b, what
is the value of .£ ?
a

I I I I I
CHAPTER 8 / THE SAT MATH TEST: PROBLEM SOLVING AND DATA ANALYSIS 347

m
The freshman class at H illside H igh School has 45 I f the population of town B is 50% greater than the
more girls than boys. If the class has n boys, what population of town A, and the population of town C
p ercent of the freshman class are girls? is 20% greater than the population of town A, then
what percent greater is the population of town B
n + 45
A) --- % than the population of town C?
2n + 45
l OOn A) 20%
B) --- %
2n + 45 B) 25%
l OO(n + 45) C) 30%
C) %
2n + 45 D) 40
l OOn
D) -- %
n + 45
348 McGraw-Hill Education: SAT

EXE RCISE SET 3 ANSWE R KEY

1. 45 1 . 50 x 30 = 45

2. 1.5 Imagine that the areas are 411 and 911. Since the
area of a circle is 11r2, their radii are 2 and 3, and their circum­ 1 4 . 8.00 Let x b e the price before tax:
ferences are 2(2)11 = 411 and 2(3)11 = 611, and 611 -:- 411 = 1.5. l .05x = $8.40
Divide by 1 .05: x = $8.00
3 . 49 70 - 0.30(70) = 0.70 (70) = 49.
1 5 . 1 5 ( 6 8 - 80)/80 = - 0 . 1 5
x 5
4 . 7. 5
36 24 1 6 . 1 . 82 b = l . 30a and c = l .40b, so c = l .40(1 . 30 a) =
Cross multiply: 24x = 180 l . 82a. Therefore c/a = l . 82a/a = 1 .82.
D ivide by 24: X = 7. 5
17. 150 For convenience, pick the dimensions of the
s2 gallon rectangle to be 1 0 and 10. ( This is of course a square, but
5 gallons
5 . 100 remember that a square is a rectangle! ) This means that
8 miles x miles the original area is 10 x 1 0 = 100. I f the width decreases
Cross multiply:
2
- x = 40 by 20%, the new width is (0.80)(10) = 8. Let the new
5 length be x. Since the new rectangle has double the area,
Multiply by 5/2: X = 100 Bx = 200, and so x = 25. This is a n increase of (25 - 10)/ 10 x
100% = 150%.
6. D 1 .05 x 0.80 x 0.80 x $60 = $40.32
1 8 . 1 5 The total number of "St. Louis votes" can b e
7. c ( 1 20,000 - 40,000)/40,000 x 100% = 200% expressed in two ways, so w e c a n s e t up a n equation to
solve for n: (0.56) (25) + (0.60) n = 0 . 575(25 + n)
8. D The fraction of students who are athletes is Simplify: 14 + 0.6n = 14.375 + 0 . 575n
3/(2 + 3) = 3/5, and the fraction of these who are females Subtract 14 and .575n: 0.025n = 0. 375
is 5/(3 + 5) = 5/8. Therefore the portion who are female D ivide by .025: n = 15
athletes is 3/5 x 5/8 = 3/8 = 37. 5% .
19. C Th e total amount o f salt in the mixture i s (.30)(12) +
9. C Th e purple dye i s 3/(3 + 4) = 3/7 red, and the orange (.60)(24) = 18, and the total weight of the mixture is 1 2 + 24 =
dye is 3/(3 + 2) = 3/5 red. Therefore, a half-purple, half­ 36 ounces, so the percent salt is 18/36 = 50%.
orange dye is (1/2)(3/7) + (1/2)(3/5) = 3/14 + 3/10 = 18/35 red.
20. A If the origin a l dimensions are w and l, the origi­
10. B If the price of the stock were originally, say, nal area is wl. I f the length is doubled and the width
$ 100, then after this two -year period its price would be decreased by 10%, the new area i s (0.9/)(2 w) = l . Bwl,
(0.70)(1 . 80)($ 100) = $ 1 26, which is a 26% increase. which is an increase of 80%.

1 1 . D The quantity of barley seed is proportiona l to 21. C The number of girls i n the class i s n + 45, and the
the acreage it can cover. The cost of seed for each acre of total number of students is n + n + 45, so the percent
barley was origin a l ly $ 1 , 200/30 = $40 per acre. The n + 45
of girls is --- x 1 00% .
next yea r, after the 20% decrease, the price would be 2n + 45
(0.80)($40) = $32 per acre. With the same budget, the
farmer can t herefore plant 1 , 200/32 = 37.5 acres of barley. 2 2 . B B is 50% greater than A : B = l . 5A
C is 2 0 % greater t h a n A : C = 1 . 2A
1 2 . 1 / 1 5 or 0.067 or 0.066 Divide b y 1 . 2 : 0.83C = A
Substitute: B = l .5(0.83C)
2 2 1 80 3 Simplify: B = l . 25C
- % of 90 = - -:- 1 00 x 90 = - = -
3 3 300 5
2 2 3 1
--X=---=-
3 3 5 15
CHAPTER 8 I THE SAT MATH TEST: PROBLEM SOLVING AND DATA ANALYSIS 349

Ski l l 3 : Worki ng with Ta bles of Data


Lesson 1 0: Using ta bles as problem-solving tools

A table c a n be useful for organizing information that fal l s into categories. Even if a problem does not include a
table, ask yourself: does the information in this problem fall into non- overlapping categories? I f so, consider
setting up a table with the categories as row or column labels.

A comm ittee determines that it will meet on the and only $ 1 ,000 in trade-in credit from the Model
Thursday after the third Monday of every month. P dea ler. Even after the trade-in credits are applied
What is the latest date of the month on which this to both cars, Model N is still $2,000 more expensive
meeting could fall? than Model P (before taxes and fees). What is the
st icker price of Model N?
A) the 17th
B) the 18nd A) $10,000
C) the 24rd B) $ 1 2,000

D) the 25th C) $1 2,500


D) $ 1 3,000
(Medium) A calendar, which of course is a kind of table,
can b e handy here. But how do we fil l i n the numbers? (Medium) Again, this problem does not give us a table,
Since we want the latest date of the month possible, we but the fact that we must keep track of two car models
need to find the latest date on which the third Monday and two prices per car model (st icker price and post
could fal l . A little trial-and-error should revea l that the trade-in price) suggests that a table might be usefu l .
latest the first Monday could fal l is the 7th.
Sticker Price After Trade-In
s M T w Th F s
Model N l . 25x l . 25x - 1 , 500
1 Model P x x - 1,000

7 8
Since Model N has a sticker price that is 25% higher
14 than that of Model P, if the sticker price of Model P is x,
then the sticker price for Model N is l .25x. The respective
21 22 23 24 costs after trade-in, then, are l . 25x - 1, 500 and x - 1,000.
Now we must solve for x by setting up an equation, which
comes from the fact that, even after the trade-in, Model N
Since there are seven days per week, the second
is still $2,000 more expensive. Therefore l .25x - 1500 =
Monday must be the 14th and the third must be the
(x - 1,000) + 2,000.
2 1 st. Therefore the meeting will be three days later, on
Thu rsday the 24th. We don't h ave to complete the entire Simplify: l . 25x - 1,500 = x + 1,000
calendar page to solve the problem. Add 1,500 and subtract x: 0. 25x = 2,500
Divide by 0.25: x = 10,000
Sam is considering buying a car, and has two mod­
els to choose from. Model N has a sticker price that is Now, keep in mind that the question is asking for the
2 5% higher than that of Model P, but he will receive sticker price of Model N, not Model P. Therefore the cor­
$ 1 , 500 in trade-in credit from the Model N dealer rect an swer is ( 1 . 25)(10,000) = 1 2 , 500.
350 McGraw-Hill Education: SAT

Lesson 1 1 : Finding conditional proba bil ities and


popu lation f ractions

OPINION ON PROPOSAL 547 question, namely: ifa poll subject who expressed an opin­
Approve Disapprove No Opinion Total ion on Proposal 54 7 is chosen at random, what is the prob­
ability that the person is female?
Female 1 20 42 38 200
Male 98 40 62 200
Total 218 82 100 400
The probability of an event is a part-to-whole
ratio, and therefore can have only a value from
According to the results of the poll indicated i n the
0 to 1. An event with probability 0 is impossible,
table above, what p ercent of those who expressed a n
and an event with probability l is certain.
opinion on the proposal were female?
A) 52%
B) 54% The table indicates that the total number of respon­
C) 68% dents who expressed a n opinion is 218 + 82 = 300. Of
these, 120 + 42 = 162 are women, and therefore the per­
D) 81%
cent of these that are women i s 1 62/300 x 100% = 54% .
(Medium) This question asks us to find a population Alternately, we could say that if a respondent expressing
fraction, that is, a part-to -whole fraction. This question a n opinion were chosen at random, that person would
i s almost identical to asking a conditional probability have a 0.54 probability of being female.
CHAPTER 8 I THE SAT MATH TEST: PROBLEM SOLVING AND DATA ANALYSIS 351

Lesson 1 2: Analxzins variable relations with tables

In Lesson 4 we used tables to ident ify direct and which we can then use to plot points on a graph.
i nverse variations between variables, but ordered
p a i r s of variables can h ave m a ny other relationships x y
as wel l .
2 10
5 b
Tables-along with graphs and equations-are
important tools for analyzing functional relation­ 10 34
ships b etween variables. When given an equa­
tion expressing y in terms of x, you should know If the variables x and y in the table above h ave a lin­
how to generate a table of ordered pairs and use ear relationship, what is the value of b?
that table to graph the relationship. Alternately,
A) 1 9
you should learn to analyze the properties of a
function from that table of values. B) 20
C) 21
D) 22
Even if our equation i s complicated, l i ke
(Medium) One way t o approach this question is t o use
x3 - x
f ( x ) = 2x + 4 the fact that, in a linear relation, the y i ncreases i n pro ­
portion to the change i n x. This rate of increase is the
we can just pick values for x, l i ke -3, -2, - 1 , 0, 1 , 2, and slope of the line. Taking the first and third ordered pairs,
3, use these to calculate the corresponding values of it seems that when x increases by 10 - 2 = 8, y in turn
y (orf(x)) and create a table of ordered pairs increases by 34 - 10 = 24. Therefore, the slope of the line
is 24/8 = 3. That is, every time the x coordinate i ncreases
x y by 1, the y coordinate increases by 3 . Now looking at the
first and second ordered pairs, since x is increasing by
-3 12 5 - 2 = 3, y must increase by 3 (3) = 9, and so b is equal to
10 + 9 = 19, and the correct answer is (A).
-2 no sol.
-1 0 Another way to look at it is that we have solved the
. 34 - 1 0 b - 1 0
0 0 proportion --- = -- :
10 - 2 5-2
1 0 24
- b - 10
--
Simplify:
8 3
2 3/4 Cross multiply: 72 = 8b - 80
3 1 2/5 Add 80: 152 = 8b
Divide by 8: 19 = b
352 McGraw-Hill Education: SAT

Exercise Set 4 (Ca lc u lator)

• ..
BACTERIA CULTURE POPULATION By what percent did the population of c u lture B
i ncrease over the first 3 m inutes?
M i nutes 0 1 2 3
A) 36.4%
Culture A 520 720 920 1 , 1 20
B) 4 2 . 1 %
Culture B 500 600 720 864 C) 72.8%
D) 172.8%
Which of the fol lowing equations best expresses the
population, P, of bacteria culture A, as a function
of t, in minutes?
A) P = 200t + 520
Questions 6-8 refer t o t h e followi ng i nformation
B ) p = 520(1 .4) '
C ) p = 2,000(t - 0.5)2 TALENT SHOW TICKETS
D) p = 520t + 200
Adult Child Senior Student
Tickets Sold 84 40 16 1 10
.. Total Revenue $630 $200 $96 $495
Which of the fol lowing equations best expresses the
popul ation, P, of bacteria culture B, as a function of
t, i n m i nutes?

A) P = lOOt + 500 According to the table above, how much i s the price
B ) p = 500(1 . 2) ' of one senior ticket?
C ) p = 2,000(t - 0.5)2 A) $4.00
D) p = 500t + 100 B ) $6.00
C) $ 1 2 . 0 0

.. D) $16.00
After 2 minutes, the population of culture A is what
percent greater than the popul ation of culture B?
..
A) 16.7% H o w much more is the c o s t of o n e adult t icket than
B ) 20.0% the cost of one student t icket?
C ) 27.8% A) $0.50
D) 1 2 7.8% B ) $ 1 .50
C ) $2.50

• D) $3.00
I f culture A continues to grow at a constant rate, at
what time should its population reach 2,000?
A) 7 minutes 4 seconds
B) 7 minutes 24 seconds
C ) 7 minutes 40 seconds
D) 8 m i nutes 20 seconds
CHAPTER 8 I THE SAT MATH TEST: PROBLEM SOLVING AND DATA ANALYSIS 353


Which is closest to the average (arithmetic mean)
price of the 250 tickets sold? Questions 1 2-2 1 refer to the fol lowing
information
A) $5.54
B) $5.59 U.S. ENERGY CONSUMPTION
C) $5.68 (Quadrillion BTU (QBTU))

D) $5.72 Fossil Nuclear Non-nuclea r Total


Fuels Renewables

1950 3 1 .63 0.00 2.98 34.61


1970 63.52 0.24 4.07 67.84
1990 72.33 6.10 6.04 84.47
What is the median price of the 250 tickets sold?
2010 81 . 1 1 8.43 8.09 97.63
A) $5.00
B) $5.50
C) $5.75
D) $6.00 From 1970 to 1990, the percent i ncrease in the U. S.
consumption of nuclear energy was closest to
A) 96%
B) 240%
I f a meeting must take place on the third Tuesday of
C) 2,400%
the month, what is the earliest date of the month on
which it could take place? D) 3,400%
A) the 14th
B) the 15th Ill
C) the 22nd I n a pie graph representing total U. S . energy con­
DJ the 27th sumption in 2010, the sector representing non
nuclear renewables would have a central angle
measuring approximately
Ill A) go
What is the latest date of the month on which the B) 1 2°
meeting could take place?
C) 24°
A) the 13th D) 30°
B) the 14th
C) the 2 1 st
ID
D) the 26th
Nuclear energy and renewable energy are often
grouped together in the category "non-greenhouse"
energy. I n 1 970, approximately what percent of non­
greenhouse energy was nuclear?
A) 0.4%
B) 5.6%
C) 5.9%
D) 6 .4%
354 McGraw-Hill Education: SAT

Ill Ill
I n 2010 what percent of non-greenhouse energy For how m any of the years shown above did fossil
consumption was nuclear? fuels account for less than 90% of the a n nual U.S.
energy consumption?
A) 9%
B) 29% A) One

C) 49% B) Two

D) 51% C) Three
D) Four

In the four years shown, what percent of the total


energy consumed was due to non-nuclear renewables? Between 1950 and 2010, the average a n nual rate of
A) 5.6% i ncrease in the consumption of non-nuclear renew­
able energy was closest to
B) 6.8%
C) 7.4% A) 0.085 QBTU/yr
D) 7.9% B) 0. 1 28 QBTU/yr
C) 1 .70 QBTU/yr
D) 2.27 QBTU/yr
Ill
What was the percent increase in fossil fuel energy
consumption between 1 950 and 20107
A) 28% Between 1 9 70 and 2010, the a n nual consumption of
B) 6 1% fossil fuels in the U. S . i ncreased nearly linearly. I f
t h i s l i near trend were t o continue, which of t h e fol­
C) 1 24%
lowing is closest to the level of U . S . fossil fuel con­
D) 1 56% sumption we would expect for 2035 (in quadrillion
BTUs)?
A) 90
DI
B) 9 1
The "renewabil ity index" is defined as the fraction
of total U.S. energy consu mption that comes from C ) 92
non-nuclear renewable energy. What was the per­ D) 93
cent increase in the renewability index from 1 9 70 to
2010?
A) 17%
B) 37%
C) 47%
D) 99%
CHAPTER 8 I THE SAT MATH TEST: PROBLEM SOLVING AND DATA ANALYSIS 355

EXERCISE SET 4 ANSWE R KEY

1 . A According to the table, the population of culture


10. B The earliest the first Tuesday could b e is the 1 st,
A increases by 200 bacteria every minute, indicating a
so the earliest the third Tuesday could b e is the 1 5t h .
linear relationship with a slope of 200. (Remember that
the slope of a function is equivalent to its unit rate of
change.) Choice (A) is the only option that indicates a 1 1 . C The latest t h e fi r s t Tuesday could b e is the 7 t h , s o
line with slope 200. t h e latest t h e t h i r d Tuesday could b e is the 2 1 st .

2 . B The table i ndicates that culture B is not increas­ 1 2 . C I n 1 9 70, nuclear energy consumption was 0. 24,
ing linearly, since the population difference from min­ and in 1990 it was 6. 10. This represents a n increase of
ute to minute is not constant, but increasing. This rules (6. 1 0 - 0. 24)/0.24 x 100% = 2,442%.
out choices (A) and (D). By substituting t = 0, t = 1 and
t = 2, we can see that only the function i n (B) gives the 13. D I n 2010, non nuclear renewables accounted for
correct populations. Notice that the base of the expo ­ 8.09/97.63 x 100% = 8.3% of consumption, which would
nential, 1 . 2, indicates t h a t the population grows b y 20% correspond to a 0.083 x 360° = 29.88° central angle.
each m i nute.
14. B I n 1 9 70, the total "non-greenhouse" energy was
3 . C At the 2 m i nute mark, the populations are 920 0.24 + 4.07 = 4 . 3 1 . Therefore the percent that was nuclea r
and 720, respectively, so culture A has a population that is 0. 24/4.31 x 1 0 0 % = 5.6%.
is (920 - 720)/720 x 100% = 27.8% greater.
1 5 . D I n 2010, this percent was 8.43/(8.43 + 8.09) x

4 . B I f we use the population equation (see ques­ 100% = 51%.


tion 1), we can solve for t. Plugging in 2,000 for P gives
us 2,000 = 200t + 520, which gives a solution of t = 7.4 1 6 . C The total non nuclear renewable energy con­
minutes. Since 0.4 minutes equals 0.4 x 60 = 24 seconds, sumption for the four years is 2.98 + 4.07 + 6.04 + 8.09 =
the time elapsed i s 7 minutes 24 seconds. 2 1 . 1 8, and the total energy consumption is 34. 6 1 +
67.84 + 84.47 + 97.63 = 284.55. Therefore the percent is
5 . C I n the first 3 m inutes, culture B grew from 500 to 2 1 . 1 8/284.55 x 1 00% = 7.4%.
864 bacteria, which is a n increase of (864 - 500)/500 x
100% = 72.8%. 17. D I n 1 950, fossil fuel consumption was 31 .63,
and i n 2010 it was 8 1 . 1 1 . This is a n increase of (81 . 1 1 -
6. B The total revenue for each ticket type equals 3 1 .63)/31 .63 x 100% = 156%.
the price per ticket times the number of tickets sold.
Therefore, the price for each senior t icket is $96 -:- 16 = $6. 1 8 . B I n 1 970, the renewability i ndex was 4.07/67.84 =
0.060, and i n 2010 it was 8.09/97.63 = .082. Thi s i s a per­
7. D One adult ticket costs $630 -:- 84 = $7.50, and cent increase of (0.082 - 0.060)/0.060 x 100% = 37%.
one student t icket costs $495 -:- llO
= $4. 50, so each adult

t icket costs $3 more. 1 9 . B I n 1990, the percent of consumption from fossil


fuels was 72.33/84.47 x 100% = 85.6%, and i n 2010 it was
8 . C The average price per ticket equals the total 8 1 . 1 1 /97.62 x 100% = 83.1%.
revenue for all t ickets d ivided by the number of tickets:
($630 + $200 + $96 + $495)/250 = $5.684. 20. A The annual rate of increase is the total increase
divided by the time span in years. The total increase is
9. A The median price of 250 tickets is the average of 8.09 - 2.98 = 5. 1 1 . Over a 60-year span, this gives a rate of
the prices of the 1 25th and 1 26th t ickets, if the price for 5. 1 1 /60 = 0.085.
each ticket is listed in increasing order. The t icket prices,
i n increasing order, are $4.50 for students (llOt ickets), 2 1 . C I n the 40 year span from 1 970 to 2010, fossil fuel
$5.00 for children (40 tickets), $6.00 for seniors (16 t ick­ consumption increased at a rate of (81 . 1 1 - 63. 52)/40 =
ets), and $7.50 for adults (84 tickets) . With this ordering, 0.44 QBTU/Yr. I n 25 more years at this rate, the con­
the 1 25th and 1 26th price are both $5.00. sumption should be 8 1 . 1 1 + 25(0.44) = 92 . 1 1 QBTU.
356 McGraw-Hill Education: SAT

Ski l l 4: Worki ng with Gra p h s of Data


Lesson 1 3: Working with scatterplots

S catterplots are graphs of ordered pairs that represent (Medium) I n this problem, the l i ne of best fit shows a
data points. They are very usefu l for showing relation­ general downward trend in the data, even though the
ships between variables that do not vary in a highly pre­ data are fa irly widely scattered . To find the "average
dictable way. yea rly decrease" i n particulate matter concentration, we
simply have to find the slope of this line. To get the most
PARTICULATE MATTER, 2004-20 1 2 accu rate estimate for slope, we should take points on the
25
line that are far apart. The leftmost endpoint seems to be


at (2003, 1 2), and the rightmost endpoint seems to be at
-......_ (2013, 9). This gives u s a slope of (9 - 1 2)/(2013 - 2003) =
CJ)
s 20 -3/ 10 = - 0. 3 micrograms/m3 per year, so the correct
(lj
So
an swer is (A).
0 II 0
0 0
...
u
15
s
According to the line of best fit to the data above,
•• 0
· I
- -
•• - - - which of the fol lowing is closest to the percent
c: �� - - - - - - - -- - - - - -
....__,

0 '
o � •• decrease in average particu late matter concentra­
• :--.t----- -
·o
(lj
0 0 •• 0 l�r--- --
10
b •• •
tion from 2007 to 2012?
c:

Q) ••
u II 0 0 •• •• A) 9%
c:
0 5 ••
u B) 1 8%
C) 36%
0
2004 2006 2008 20 1 0 20 1 2 D) 60%
(Medium-hard) This question is similar to the previous
Year
one, but notice the two important differences: first, it is
asking us to compare two specifi c years, and it is asking
Particulate matter is a class of air pollutants. The us to calculate the percent decrease rather than the rate
scatterplot above shows 40 readings for particu­ of decrease, so we will need the percent change formula
late matter concent ration, in micrograms per cubic from Lesson 8. The line of best fit gives a value of about
meter, for a particular metropol itan area over 11 in 2007 and about 9 in 2 0 1 2 . Therefore the percent
9 years. Based on the l i ne of best fit shown, which cha nge is (9 - 1 1 )/ 1 1 x 100% = - 18%, and the correct
of the fol lowing is closest to the average yearly answer is (B).
decrease in particulate matter concentration?
A) 0.32 micrograms/m3 per yea r If the Environmental P rotect ion Agency's a i r
q u a l ity standard i s 1 2 micrograms of p a r t ic u l ate
B ) 0.64 m icrograms/m3 p e r year
matter per cubic meter, as shown with the dot­
C) 3 . 2 microgram s/m3 per year ted l i ne, what p ercent of these data fa II above t h i s
D) 6.4 m icrograms/m3 per year standard?
A) 1 1%
A line of best fit i s simply a line that "hugs" the B) 14.5%
data in a scatterplot better than any other line. In C) 27.5%
a statistics course, you will learn how to calculate
a line of best fit precisely (or your c alculator can D) 35%
do it for you), but the SAT will only expect you to (Easy) This question is simply asking for a part-to-whole
use lines of best fit to make inferences about gen­ ratio expressed as a percentage. There are 40 total data
eral trends in data or to find "expected values." points (as the problem states), and 14 of them lie above
the line: 1 4/40 x 1 00% = 35%, so the correct answer is (D).
CHAPTER 8 I THE SAT MATH TEST: PROBLEM SOLVING AND DATA ANALYSIS 357

Lesson 1 4: Linea r a nd non l i near relationsh ips

Sometimes the SAT Math test will ask you to draw inferences from graphs that are non-linear. Even if a graph
is not linear, you should still be able to draw inferences based on the points on that graph. For i nstance, you
should be able to find the average rate of change between two points by finding the slope of the l i ne segment
connecting them, or compare the rate of growth of one curve to that of another curve.

CELL POPULATION (Medium) The time when the two popul ations are equal
is easy to find: it's where the two curves intersect, at the
2,000 4-minute mark. The four answer choices tell us where

� to look next on the graph . Since choice (A ) is 2 m inutes,
CtUtm e A y we should find the populations at the 4 + 2 = 6-minute
1 ,600
/ mark. At 6 minutes, culture B has a population of 700

I
and culture A has a popu lation of 1 ,400, which of course
1 ,200 is twice as great, and therefore the answer is (A ) .
CJ)
:=l
Q) I
I How much longer did it take culture B to double its
u
800 I �� original population than it took culture A?
/ __�
�--
Cilltu e B A) 2 minutes

400 --- --- � 7 B) 3 minutes


___....
/ C ) 4 minutes
D) 5 mi nutes
0
1 2 3 4 5 6 7 8 9 (Medium) Culture A has a starting population of 200
Minutes and doubles its population to 400 at the 3 - m i nute mark.
Culture B has a starting population of 400 and doubles
its population to 800 at the 8-minute mark. Therefore,
The graph above shows the number of cells in two
culture B took 8 - 3 = 5 more minutes to double its pop­
separate bacterial cultures as a function of time.
ulation, and the correct answer is (D).
How much time elapsed b etween the point when
the two cultures had equal population and the point
when the population of culture A was twice that of
culture B?
A) 2 minutes
B) 3 minutes
C ) 4 m i nutes
D) 5 minutes
358 McGraw-Hill Education: SAT

Lesson 1 5 : Drawi ng i nferences from graphs

C ELL POPULATION (Medium) By taking a ny two points on the line for culture
B, for i nstance (0,400 ) and (4,600) , we c a n calculate the
2, 000 slope of the l i ne, which equals the unit rate of growth:
,/"'--- (600 - 400) / (4 - 0 ) = 50 cells per m i nute. Since c u lture
CtUtrn e A v B contains 900 cells at the 1 0 m i nute m a rk, it woul d take

v
1 , 600 600/50 = 12 more m i nutes to reach 1 , 500 cells, and the
correct answer i s ( B ) .
1 , 200 I
VJ
::=l
<l.) �
I A particular experiment requires that both culture A
and culture B contain between 400 and 800 cell s each.
u
I --i.-- The time period in which the cell population for both
/
800
i.--i--c W.tu eB
__---
cultures is within this range is called the "experimen­

7
i.--
tal window." Which of the following is closest to the
--1-- "experimental window" for the two cultures shown
400
i__.v
...-- in the graph above? ( 1 m i nute = 60 seconds)
A) 95 seconds
0
1 2 3 4 5 6 7 8 9 B) 1 2 0 seconds

Minutes C) 165 seconds


D) 240 seconds
If culture B were to continue its linear growth, how ( Medium-hard) The "experimental window" is the period
m a ny more m inutes ( beyond the 10 m i nutes shown when both populations are between 400 and 800. The
in the graph ) would be required for culture B to population of culture A hits 400 at the 3 -minute m ark and
reach 1 , 500 cells? exceeds 800 cells after roughly the 4.6-minute m ark. The
A) 9 m i nutes population of culture B starts off ( at the 0-rninute m ark) at
B ) 1 2 m i nutes 400 cells, and exceeds 800 cells after the 8-minute m ark. The
overlapping period is between 3 minutes and 4.6 m inutes,
C) 1 3 . 5 m inutes for a period of roughly 1 .6 minutes or 1 . 6 x 60 = 96 seconds;
D) 15 m inutes therefore, the correct answer is (A).
CHAPTER 8 / THE SAT MATH TEST: PROBLEM SOLVING AND DATA ANALYSIS 359

Lesson 1 6: Working with pie graphs

We've a l l seen pie graphs. They are convenient ways of


representing part-to-part and part-to -whole relation­ When ana ly z i n g pie graphs, remember this help­
ships. O n the SAT Math test, you m ay be asked to analyze ful formula:
the features of pie graphs in some detail, or to discuss part degrees in sector
the features of a hypothetical pie graph.
whole 360°

MINORITY REPRESENTATION IN BROADCAST TELEVISION

U.S. Population U.S. Media Ownership


2007 2007

Population Media Ownership

African American II 1 3.0% 0.6%


Hispanic American D 1 5.0% 1 .3%
Asian American D 4.5% 0.9%
Other Minority D 1 .5% 0.4%
White Non-Minority D 66.0% 96.8%
Source: Freepress

In the diagram above, which of the following is clos­ from $30,000 to $48,000. Based on this information,
est to the measure of the central angle of the sec­ which of the following could b e the measure of the
tor representing total m inority ownership of U.S. central angle of the sector representing marketing
broadcast television media i n 2007? expenses?
A) 3° A) 36°
B) 8° B) 54°
C) 1 2 ° C) 62°
D ) 1 5° D) 70°
(Easy) According to the graphic, the total minority own­ (Medium-hard) The measure of the central a ngle of
ership of television media in 2007 was 0.6% + 1 .3% + the sector depends on the part-to -whole ratio, so we
0.9% + 0.4% = 3 . 2 % . Therefore the central of the sector need to calculate the maximum and minimum part­
representing this portion measures 0.032 x 360° = 1 1 .52°, to -whole ratio for the marketing expenses, the market­
so the correct answer is (C). ing expenses are fixed at $ 1 2,000, but the total expenses
could range from $ 1 2,000 + $30,000 + $30,000 = $72,000
Maria i s constructing a pie graph to represent to $ 1 2,000 + $30,000 + $48,000 = $90,000. This means
the expenses for her project, consisting of three that the part-to -whole ratio for marketing could range
expense c ategories: marketing, design, and devel­ from 1 2,000/90,000 = 0 . 1 33 to 1 2,000/72,000 = 0 . 1 67.
opment. She knows that the marketing expenses are Therefore the central angle for the marketing sector can
$ 1 2,000 and the design expenses are $30,000, but measure a nywhere from 0. 133 x 360° = 48° to 0 . 167 x
the development expense could range anywhere 360° = 60°. The only choice i n this range is (B) 54°.
360 McGraw-Hill Education: SAT

' **'


From 2009 to 2010, the total combined revenue for
Questions 1 -4 refer to the fol lowi ng information a l l stores increased by approximately
A) $50,000
ANNUAL REVENUE PER STORE
B) $200,000
,-..,
0
1 , 000 C) $400,000
V't
'-
0

"' 0
D) $600,000
"d 0
c:: 800
"'
ct:J

:i
0 0 /
Vt

E 600 ,. v Between 2006 and 2 0 1 2 , what was the percent
CJ.)
,_, ·�• ••
....0.. '.,./ increase in the total number of retail stores for this
C/) 11 / company?
,_,
v ,. ,
t/,
400
CJ.)
0... •• t '
CJ.)
A) 45%
:i v ..
0
c::
CJ.) 200
!/ 0
Ii B) 50%
>
CJ.)
0::: C) 100%
0 D) 200%
2004 2006 2008 20 1 0 20 1 2
Year

T -------


The scatterplot above shows the annual revenue for Questions 5-9 refer to the fol l owi ng information
a l l of the individual retai l stores operated by a cloth­
U N I T E D STATES F E D E RA L B U D G E T�FISCAL YEAR 20 1 0
ing company for each year from 2004 th rough 2 0 1 2 .
Based on t h e l i n e o f best fit t o t h e data shown, which
All o t h e r exp e n s e s c o m b i n e d 1 3%
of the fol lowing is closest to the percent increase in
revenue per store from 2005 to 201 1? Vet e r a n s B e n efits 3. 1 %

3.5%
D e fe n s e
20%
Federal Pensions
A) 50%
Education 3.7%
B) 100% U n e m p l oym e n t
C) 1 20% Compensation 4.6%

D) 300% Wel fa r e / F o o d Stamps 5.3%

I nterest o n N a t i o n a l D e b t 5.7%

8%
II Medicaid/Child H e a l t h I nsurance

I n 2006, the total combined revenue for all stores


was closest to
..
A) $350,000 The chart above shows the allocation of $ 3 . 5 trillion
B) $480,000 in U. S . federal expenses for 2010. What were the total
C) $700,000 2010 expenditures on Defense?
D) $950,000 A) $700 billion
B) $70 billion
C) $7 billion
D) $700 million
CHAPTER 8 I THE SAT MATH TEST: PROBLEM SOLVING AND DATA ANALYSIS 361

.. •
What is the measure of the central angle for the sec­ How much more did the United States spend i n 2010
tor representing Med icare expenses? on I nterest on National Debt than on E ducation?
A) 1 3 .0° A) $2 billion
B) 45.5° B) $7.0 billion
C) 46.8° C) $20 billion
D) 48.2° D) $70 billion

..
• If 50% of the budget for Federal Pensions were to be
I f I nterest on National Debt expenses were to reallocated as Social Security expenses, the size of
decrease by $20 billion from their 2010 levels, this the Social Security budget would increase by what
would represent a percent decrease of approximately percent?
A) 6% A) 1 .75%
B) 10% B) 8.75%
C) 1 2% C) 17.75%
D) 15% D) 2 1 .75%

Questions 1 0- 1 7 refer to the followi ng i nformation

PRESCHOOL BLOOD LEAD LEVELS VS. VIOLENT CRIME RATES IN THE UNITED STATES

(23-Year Lag)
::a
-- ' 30 900
�I $
0
E' ro-

_)/ 'f\
�1 -
25 750 :i
"U
"'
...,

QJ n
.:; 0 ......
J- -�� §"
20 600
J ,

o en \ <.o ro
o en \ I en
' w
co 0
7 15
[/>

I "U
\

...,
�v
'
450 ro

...... ......
0 LI, \ N
0
0 � 0
\
c

_//
-5 _o
"' 10 ' 300 � 0
QJ \
0
\ 0
ct \
QJ (")
5 - 1 50 Cl)
tl.() ' -
I
ttl "U
�I s
> ·
<t:: 0 0
1 940 1 950 1 960 1 970 1 980 1 990
1 963 1 973 1 983 1 993 2003 20 1 3
Year
Source: Rick Nevin, Lead Poisoning and the Bell Curve, 201 2


According to the graph above, in 1 970 the number of
violent crimes per 1 00, 000 capita in the United States
was closest to
A) 25
B) 375
C) 700
D) 750
362 McGraw-Hill Education: SAT

Ill Ill
In 1 970 the average preschool blood lead level, in Which of the following five-yea r spans saw the
mcg/dL, was closest to greatest p ercent increase i n violent crime?
A) 10 A) 1 9 63 -1 9 68
B) 12 B) 1 968-1973
C) 23 C) 1 973-1978
D) 25 D) 1978-1983

The percent decline i n violent crime from 1993 to Approximately how many years did it take for aver­
2 0 1 3 is closest to age preschool blood lead levels to return to their
1950 levels?
A ) 1 1%
B) 35% A) 25
C) 47% B) 30
D) 88% C) 35
D) 40

Ill
From 1970 to 1990, the average annual rate of •
decline in preschool blood lead levels, in mcg/dL per For approximately how many years between 1 9 6 3
yea r, was approximately and 2 0 1 3 was the violent c r i m e rate in the United
States greater than 375 crimes per 100,000 c apita?
A) 1
B) 5 A) 25
C) 1 5 B) 30
D ) 17 C) 37
D) 42

ID
Which of the following 1 0 -year spans saw the great­
est percent increase in preschool blood lead levels?
A) 1945-1 955
B) 1 955-1 965
C) 1 9 6 5 - 1 975
D) 1 9 75 - 1 985
CHAPTER 8 / THE SAT MATH TEST: PROBLEM SOLVING AND DATA ANALYSIS 363

EXERCISE SET S ANSWER KEY

1 . C I n 2005, the revenue per store, according to the


10. B The vertical axis label on the left shows that the
line of best fit, was about $300,000, and i n 2012 it was
violent crime trend is indicated by the solid curve and
about $650,000, so the percent change is (650,000 -
the bottom time series (1 963-201 3) . For this curve, 1970
300,000)/300,000 x 100% = 1 16.67%, which is closest to
is slightly to the left of the vertical line at 1973, which
(C) 1 20%.
shows values clearly between 300 and 450.

2 . D I n 2006, the data points show that there were 3


1 1 . C The vertical axis label on the left shows that the
stores, with revenue of roughly $200,000, $330,000, and
preschool blood lead trend is indicated by the dashed
$420,000, for a total of $950,000.
curve and the top time series (1940-1990).

3 . D I n 2009, the combined revenue for the three


12. C I n 1 993, the violent crime rate was 750, and in
stores was approximately $420,000 + $450,000 +
2013 it was about 400. The percent decrease is therefore
$550,000 = $ 1 ,420,000. I n 2010, the combined revenue
(400 - 750)/750 x 100% = 46.7%
for four stores was approximately $220,000 + $520,000 +
$600,000 + $675,000 = $2,01 5,000, for an increase of
about $595,000. 1 3 . A In 1970, the blood lead levels were about 23 and
i n 1 990, they were about 3. The rate of decline is therefore
(23 - 3)/(1 990 - 1970) = 1 mcg/dL per year.
4. C I n 2006 there were 3 stores and in 2012 there were
6 stores, which is an increase of (6 - 3)/3 x 100% = 100%.
14. A From 1945-1955 preschool blood lead levels
increased from about 5 to about 17, a percent increase of
5. A The chart shows that 20% of the expense budget
(17 - 5)/5 x 100% = 240%.
went to defense, which equals 0.2 x $3,500,000,000,000 =
$700 billion.
15. A The question asks for the greatest percent
increase, not the greatest net increase i n violent crime.
6. C Medicare accounts for 13% of expenses, so the
Notice that the net increase from 1963-1968 (from
sector angle is 0 . 1 3 x 360° = 46. 8°.
roughly 150 to 250) seems to be slightly less than net
i ncrease from 1 968-1973 (from roughly 250 to 375),
7. B The I nterest on National Debt in 2010 was the percent i ncrease from 1 963-1968 (+67%) i s clearly
0.057 x $3.5 trillion = $ 1 99.5 billion, so a decrease of greater than that from 1968-1 973 (+50%).
$20 billion would be 20/ 1 99 . 5 x 100% = 10%.
16. B I n 1 950, blood lead levels were about 12 mcg/dL,
8. D The difference b etween I nterest on National and they did not return to this level until 1980.
Debt and Education i s 5.7% - 3.7% = 2%, and 0.02 x
$3.5 trillion = $70 billion.
17. D The graph indic ates that from about 1 9 70 to
2013, the violent crime rate was above 375 crimes per
9. B "The Social Security budget i n 2010 was 0.20 x 100,000 capita.
$3.5 trillion = $700 billion. 50% of the Federal Pensions
budget is 0.5 x 0.035 x $3.5 billion = $ 6 1 . 2 5 billion. This
would b e a n increase of 6 1 . 25/700 x 100% = 8.75%.
TH E SAT MATH :
AD VAN C ED MATH EMATIC S

l. U nd e rsta n d i n g Fu nctions 366

2. Wo rki n g with Q u a d ratics a n d Oth e r Polyn o m i a l s 3 76

3. Wo rki n g with Expo n e n ti a l s a n d Rad ica l s 392

4. Worki n g w i t h Rati o n a l Express ions 400

364
CHAPTER 9 I THE SAT MATH: ADVANCED MATHEMATICS 365

The SAT Math:


Advanced Mathematics
Why are the Advanced Mathematics topics important o n the SAT Math test?

About 27% (16 out of 58 points) of the SAT Math questions are Advanced Mathematics questions. Questions i n
this category are about
understanding of the structure of expressions and the ability to analyze, manipulate, and rewrite these
expressions. This includes an understanding of the key parts of expressions, such as terms, factors, and
coefficien ts, and the ability to interpret complica ted expressions made up of these components.
It will also assess you r skill in
rewriting expressions, identifying equivalent forms of expressions, and understanding the purpose of
different forms.
The specific topics include
solving, graphing, and analyzing quadratic equations
solving equations with radicals that may include extraneous solutions
solving systems including linear and quadratic equations
creating exponential or quadratic functions from their properties
calculating with and simplifying rational expressions
analyzing radicals and exponentials with rational exponents
creating equ ivalent forms of expressions to reveal their properties
working with compositions and transformations of functions
a nalyzing higher-order polynomial functions, particularly in terms of their factors and zeros

How is it used?

Fluency i n these topics i n advanced math is essential to success in postsecondary mathematics, science,
engineering, and technology. Since these subj ects constitute a portion of any liberal arts curriculum, and a sub­
stantial portion of any STEM (science, tech nology, engineering, or mathematics) program, colleges consider
these to be essential college preparatory skills for potential STEM majors.

Sound intimidating? It's not.

If you take the time to master the fou r core skills presented in these 14 lessons, you will gain the knowledge and
practice you need to master SAT Advanced Math questions.
366 McGraw-Hill Education: SAT

Ski l l 1 : U ndersta nd i ng Fu nctions


Lesson 1 : What i s a fu nction?

A function is just a "recipe" for turning any "input" number i nto another number, called the "output" number.
The input number is usually called x, and the output number is f(x) or y. For instance, the function f(x) = 3x2 + 2
is a three-step recipe for turning any input number, x, into another number,f(x), by the following steps: (1) square x,
(2) multiply this result by 3, and (3) add 2 to this result. The final result is calledf(x) or y.

I ff(2x) = x + 2 for all values of x, which of the follow­ of the situation in which the tank starts with twice as
ing equalsf(x)? much water as the original tank had, and the water
drains at three times the original rate?
A) x+2
--

2 A)

B) .:_ + 2
2
Depth
C) x - 2
2
D) 2x - 2
(Medium) Let's use the "function-as-recipe" idea. The Time
equation tells us that f is a function that turns an input
of 2x into x + 2. What steps would we need to take to B)
accomplish t his?
I nput value: 2x
Depth
1. Divide by 2: x
2 . Add 2: x+2
Therefore, f is a two-step function that takes an input,
divides it by 2, and then adds 2. Therefore, f(x) equals Time
the result when an input of x is put through the same
steps, which yields (B) f(x) = .:_ + 2. C)
2
Another way to think about this problem is to pick
a value for x, like x = 1. Substituting this into the given
equation gives us !(2(1)) = 1 + 2, or !(2) = 3 . Therefore, Depth
the correct function must take a n input of 2 and turn it
into 3. If we substitute x = 2 into all of the choices, we get
(A) !(2) = 2, (B) !(2) = 3, (C) !(2) = 0, and (D) !(2) = - 1 .
Clearly, the only function that gives the correct output Time
is (B).
D)

Depth Depth

Time Time
The graph above shows the depth of water in a right
cylindrical tank as a function of time as the tank (Medium) Although no increments are shown on the
drains. Which of the following represents the graph axes (so, for instance, the tick marks on the time axis
CHAPTER 9 I THE SAT MATH: ADVANCED MATHEMATICS 367

could indicate minutes, or hours, or days, or any other depth unit per time u n it. (Remember from Chapter 8
time u n it, and the tick marks on the depth axis could rep­ that the slope equals the unit rate of change .) I n
resent meters, or centimeters, or any other depth unit), the new graph, then, t h e tank should start a t a depth of
we do know that the point at which the axes cross is the 2 x 2 = 4 depth units, and it should drain at 3 x 1 /3 = 1
origin, or the point (0, 0). The given graph shows that the depth unit per time unit. I n other words, it should take
tank starts at 2 depth u n its and drains completely after 4 time units for the tanks to drain completely. The only
6 time units. I n other words, the tank drains at 1 /3 of a graph that shows this correctly i s (A).
368 McGraw-Hill Education: SAT

Lesson 2: Functions as graphs, eguations, or tables

Make sure you're fluent i n expressing functions in three ways: as graphs in the xy-plane, equations in func­
tional notation, or tables of ordered pairs. Also, m ake sure you can go from one format to another. Every
input-output (x - y) pair can be represented in any of these three ways . For instance, if the function g turns
an input value of -2 into an output value of 4, we can translate this in three ways:
The graph of y = g(x) in the xy-plane contains the point (-2, 4) .
• g ( - 2) = 4
I n a table of ordered pairs for the function, x = -2 is paired with y = 4 .

y Given t h e table o f values for functions g a n d h above,


for what value of x must g(h(x)) = 6?
y =f(x) •
A) 2
B) 5
y = g (x) C) 6
D) 12
--+--+--+--f--1--+-+-+--- x
(Medium-hard) Th e notation g(h(x)) = 6 means that
when the input number, x, is put i nto the function h, and
this result is then placed into function g, the result is 6.
Working backward, we should ask: what input to g would
The graphs of functions ! and g are shown above for
yield a n output of6? According to the table, only an input
- 3 :::::: x :::::: 3 . Which of the following describes the set
of 3 i nto g would yield an output of 6. This means that
of all x for which.f(x) :::::: g(x)?
h(x) = 3 . So what i nput into h would yield an output of 3?
A) x 2'. -3 Consulting the table again, we can see that g ( 5) = 3, and
therefore x = 5 and the correct answer is (B).
B) -3 :::::: x :::::: - 1 or 2 :::::: x :::::: 3
C) - 1 :::::: x :::::: 2
D) 3 :::::: x :::::: 5
(Easy) The key to this problem is understanding what the
statement J(x) :::::: g(x) means. Since J(x) and g(x) are the
y-values of the respective functions, f(x) is less than or
equal to g(x) wherever the graphs cross or the graph of
g(x) is above the graph ofj(x). The two graphs cross at the
points ( - 1 , 4) and (2, 3), and g(x) is above J(x) at every
point in b etween, so the correct answer is ( C) - 1 :::::: x :::::: 2.

g(x) h(x)
1 2 -9
2 4 -6
3 6 -3
4 8 0
5 10 3
6 12 6
7 14 9
8 16 12
9 18 15
CHAPTER 9 I THE SAT MATH: ADVANCED MATHEMATICS 369

. Lesson 3: Compositions a nd tra nsformations of fu nctions

The notationf(g(x)) indicates t h e composition o f two functions, g andf The number x is put into t h e function
g and this result is put into the function! and the result is calledf(g(x)).

I ff(x) = x + 2 andf(g(l)) = 6, which of the following When the function y = g(x) is graphed in the
could b e g(x)? xy-plane, it has a minimum value at the point
(1, -2). What is the maximum value of the function
A) g(x) = 3x h(x) = -3g(x) - l?
B) g(x) = x + 3
A) 4
C) g(x) = x - 3
B) 5
D) g(x) = 2x + 1
C) 6
(Medium-hard) The notation f(g(l)) = 6 indicates that D) 7
the number 1 is placed i nto function g, then the result
is placed into functionf, and the result is an output of 6. (Medium) The graph of y = h(x) = - 3g(x) - 1 is the
Given equation: f(g(l)) = 6 graph of g after it has been stretched vertically by a fac­
tor of 3, reflected over the x-axis, and then shifted down
Use the given definition off: g(l) + 2 = 6 1 unit. This would transform the minimum value point of
( 1 , -2) to a maximum value point on the new graph at
Subtract 2: g(l) = 4 ( 1 , 3 ( - 2 ) - 1 ) or ( 1 , 5), so the correct answer is (B).
-

I n other words, g is function that gives an output of 4


when its input is 1 . The only function among the choices
that has this property is (B) g(x) = x + 3.

l ff(x) = x2 + 1 and g(f(x)) = 2x2 + 4 for all values of


x, which of the following expresses g(x)?

A) g(x) = 2x + 1
B) g(x) = 2x + 2
C) g(x) = 2x + 3
D) g(x) = 2x2 + 1

(Medium-hard) As with the previous question, it helps


to use the law of substitution to simplify the problem. By
the definition off, g(f(x)) = g(x2 + 1) = 2x2 + 4. Therefore,
the function g turns a n input of x2 + 1 into an output of
2x2 + 4. What series of steps would accomplish this?
Starting expression: x2 + 1

Multiply by 2:

Add 2: 2x2 + 4
Therefore, g is a two- step function that takes an input,
multiplies it by 2, and adds 2, which is the function in
choice (B).
370 McGraw-Hill Education: SAT

Function Transformations The graph of y = J{x) + k, where k is a positive


number, is the graph of y = j{x) shifted up
y
k units.

..I

Y = f(x)
'
'I k , , '
' �6- -

If the function y = f(x) is graphed in the xy-plane


(as in the example above), then the following rep­
resent transformations of function f
The graph of y = kj{x) is the graph of y = f(x)
The graph ofy = J{x + k), where k is a positive num­
stretched vertically by a factor of k (if k > 1 ) or
ber, is the graph ofy = f{x) shifted left k units.
shrunk vertically by a factor of k (if k < 1).
y
y

' ...
- 6- ,,,. ,,

The graph of y = f(x - k), where k is a p ositive


number, is the graph of y = j{x) shifted right k The graph of y = -J{x) i s the graph of y = f(x)
units. reflected over the x-axis.

y y

y = - f(x)
'
'
..I

' I ,
' �6- -

k
CHAPTER 9 I THE SAT MATH: ADVANCED MATHEMATICS 371

Exercise Set 1 (Ca lc u lator)

• ------- ..- -------

I f J(x) = x2 + x + k, where k is a constant, and


Questions 5-9 refer to the table below.
J(2) = 10, what is the value off(-2)?

I I I I I
x J(x) k(x)
1 3 5
2 4 6
3 5 1
The minimum value of the function y = h(x) corre­
sponds to the point (-3, 2) on the xy-plane. What is 4 6 2
the maximum value of g(x) = 6 - h (x + 2)?
5 1 3

I I I I I 6 2 4

.. ..
The function g is defined by the equation g(x) = According to the table above, J(3) =
ax + b, where a and b are constants. I f g(l) = 7 and
g(3) = 6, what is the value of g(-5)?
I I I I I
I I I I I •
According to the table above, J(k(6)) =

Let the function h b e defined by the equation
h (x) = J(g(x)) where J(x) x2 - 1 and g(x) = x + 5.
=

What is the value of h(2)?


I I According to the table above, k(k(6)) =

According to the table above, if k(J(x)) = 5, then


what is the value of x?

I I
372 McGraw-Hill Education: SAT

• Ill
Which of the following is true for all values of x x+l
indicated in the earlier table? I f h(x) = --and f(x) = (x 1 ) 2 , then which of the
-

2
following is equal tof(h(x)) for all x?
A) f(k(x)) - k(f(x)) = O
B) f(k(x)) + k(f(x)) = x A) x 2 - 2x + 2
C) f(k(x)) - k(f(x)) = x 2
B) x 2 - 2x + 2
D) f(k(x)) + k(f(x)) = O
4
C) x 2 - 2x + l
------ · ------
2
2
x - 2x + l
D)
4
I f g(x - 1) = x2 + 1 , which of the following is equal
to g(x)?

A) x2 + 2
B) x2 + 2x
C) x2 + 2x + 1
D) x2 + 2x + 2
CHAPTER 9 / THE SAT MATH: ADVANCED MATHEMATICS 373

Exerc ise Set 1 (No C a l c u lator)

Ill
I f g(f(x)) = - 1 , what is the value of x + 10?
Questions 1 2- 1 9 a re based o n the graph below.

y I I I
y = f(x)


I ff(k) + g(k) = 0, what is the value of k?

I I I I
What is the value of g(- 1)?
DI
I f f(a) = g(a), where a < 0, and f(b) = g(b), where

I I I I I
b > 0, what is the value of a + b?

I I I I
Ill
What is the value of g(f(3))?

I I I I I
Let h(x) = f(x) x g(x). What is the maximum value of
h(x) if -3 :::; x :::; 3?


What is the value off(g(3))?
374 McGraw-Hill Education: SAT

Ill
Which of the following graphs represents the
y
function = f(x) + g(x)?
A) y

.-+-�--+--+--+-� X
1

B) y

1
.-+-�--+--+--+-� X

C) y

D) y
CHAPTER 9 / THE SAT MATH: ADVANCED MATHEMATICS 375

EXE RCISE SET 1 ANSWE R KEY

Calculator
1. 6 !(2) = 22 + 2 + k = 10, so 6 + k = 10 and k = 4. 11. D f(h(x)) = f ( ;) ( ; r
x 1
=
x 1
-1

( ; �r ( ; r
Therefore, f(-2) = (-2)2 + (-2) + 4 = 6.
x l x l
= - =
2 . 4 The graph of the function g(x) = 6 - h (x + 2)
is the graph of h after (1) a shift 2 units to the left, (2) a x 2 - 2x + l
reflection over the x- axis, and (3) a shift 6 units up. I f we 4
perform these transformations on the point (-3, 2), we
get the point (- 5, 4), and so the maximum value of g is 4
when x = - 5 .
No Calculator
3. 1 0 g(3) = a (3) + b = 6
g(l) = a(l) + b = 7 1 2 . 2 The graph of g contains the point (- 1 , 2), there­
Subtract the equations: 2a = - 1 fore g(- 1) = 2.
Divide by 2: a = -0.5
Substitute to find b: -0.5 + b = 7 13. 3 The graph offcontains the point (3, l); therefore,
Add 0.5: b = 7. 5 !(3) = l, and so g(f(3)) = g(l). Since the graph of g contains
Therefore g(x) = - 0.5x + 7.5 the point (1, 3), g(l) = 3 .
g(- 5) = - 0.5(-5) + 7. 5 = 10
1 4 . 2 The graph of g contains the point (3, - 1);
4. 48 h (2) = f(g(2)) = !(2 + 5) = !(7) = (7)2 - 1 = 48 therefore, g(3) = -1, and so f(g(3)) = f(- 1). Since the
graph off contains the point (- 1 , 2), f(-1) = 2.
5. s !(3) = 5
1 5 . 8 The only input to function g that yields an
6. 6 f(k(6)) = !(4) = 6 output of - 1 is 3 . Therefore, if g(f(x)) = - 1 , f(x) must
equal 3. The only input to f that yields an output of 3 is
7. 2 k(k(6)) = k(4) = 2 -2, therefore x = -2 and x + 1 0 = 8 .

8. S According to the table, the only input into k 16. 3 The only input for which f and g give outputs
that yields a n output of 5 is 1 . Therefore, f(x) must b e 1 , that are opposites is 3, becausef(3) = 1 and g(x) = - 1 .
a n d t h e only input intofthat yields an output of 1 is x = 5.
17. 1 Th e two points a t which the graphs o f g and
9 . A Examination of the table reveals that, for all f cross are (- 1, 2) and (2, 1). Therefore, a = -1 and b = 2
given values of x, f(g(x)) = x and g(f(x)) = x. (This means and so a + b = 1 .
that f and k are inverse functions, that is, they "undo"
each other.) This implies thatf(k(x)) - k(f(x)) = x - x = 0. 1 8 . 4 h(x) = f(x) x g(x) has a maximum value when
x = - 1 , wheref(l) x g(l) = 2 x 2 = 4.
10. D One way to approach this question is to pick
a new variable, z, such that z = x - 1 and therefore 19. A To graph y = f(x) + g(x), we must simply
x = z + 1. "plot points" by choosing values of x and finding
Original equation: g(x - 1) = x2 + 1 the corresponding y-values. For instance, if x = -3,
Substitute z = x - 1 : g(z) = (z + 1)2 + 1 y = !(3) + g(3) = 4 + 0 = 4, so the new graph must contain
FOIL: g(z) = z2 + 2z + 1 + 1 the point (-3, 4). Continuing i n this manner for x = -2,
Simplify: g(z) = z2 + 2z + 2 x = - 1, and so on yields the graph in (A).
Therefore g(x) = x2 + 2x + 2
376 McGraw-Hill Education: SAT

Ski l l 2 : Worki ng with Quadratics a nd


Other Polynomia ls
Lesson 4 : Adding, multiplyi ng, a nd factoring polynomials

A quadratic expression i s a second-degree polynomial, of 12 and a sum of 7? A little guessing and check­
that is, an expression of the form ax2 + bx + c. The SAT ing should reveal that the numbers are 3 and 4.
M ath test may ask you to analyze quadratic expressions Step 3: Rewrite the original quadratic, but expand the
and equations, as well as h igher- order polynomials. middle term i n terms of the sum you just found:
6x2 + 7x + 2 = 6x2 + (3x + 4x) + 2
Step 4: Use the associative Jaw of addition to group the
To factor a simple quadratic expression, first
first two terms together a nd the last two terms
see if it fits any of the b a sic factoring formulas
together: 6x2 + (3x + 4x) + 2 = (6x2 + 3x) + (4x + 2)
b elow.
Step 5: Factor out the greatest common factor from each
Difference of Squares: pair.
(6x2 + 3x) + (4x + 2) = 3x(2x + 1) + 2 (2x + l)
x2 - a2 = (x + a)(x - a)
If we do this correctly, the binomial factors will
Perfect Square Trinomials: be the same.
Step 6: Factor out the common binomial factor.
x2 + 2 ax + a2 = (x + a)(x + a) = (x + a)2
(3x + 2)(2x + 1)
x2 - 2 ax + a2 = (x - a)(x - a) = (x - a)2 Step 7: FOIL this result to confirm that it is e quivalent to
the original quadratic.

Which of the fol lowing is a factor of x2 + Bx + 16? Therefore, the correct answer is (B).
A) x - 4 Alternately, we could "test" each choice as a potential
B) x - B factor of 6x2 + 7x + 2 u ntil we find one that works. For
instance, we can test choice (A) by trying to find another
C) x + 4 binomial factor that when multiplied by (3x - 2) gives a
D) x + B product of 6x2 + 7x + 2 . -The best guess wou ld be (2x - 1),
because the product of the two first terms (3x x 2x) gives
(Easy) Notice that this quadratic fits the pattern us the correct first term, 6x2, and the product of the two
x2 + 2 ax + a2 and therefore can be factored using the last terms (-2 x - 1) gives us the correct last term, 2. Now
second formula above: x2 + Bx + 16 = (x + 4)(x + 4). we FOIL the two binomials completely to see if we get the
Therefore, the correct answer is (C). correct middle term: (3x - 2)(2x - 1) = 6x2 - 3x - 4x + 2 =
6x2 - 7x + 2, which has an incorrect middle term
To factor a more complex quadratic expression, (-7x instead of 7x). The fact that this is the opposite sign
use the Product-Sum Method illustrated b elow. of the correct middle term suggests that we need only
change the binomial from subtraction to addition, which
gives u s an answer of (B) 3x + 2.
Which of the following is a factor of 6x2 + 7x + 2?
A) 3x - 2
To add or subtract polynomials, simply combine
B) 3x + 2
like terms.
C) 3x - 1
D) 3x + 1
Expression to b e simplified:
(Medium) First notice that this is a quadratic expression
(3x4 + 5x3 - 2x + 2) - (x4 - 5x3 + 2x2 + 6)
i n which a = 6, b = 7, and c = 2. Now we can factor this
expression using the Product- Sum Method . Distribute to eliminate parentheses:
3x4 + 5x3 - 2x + 2 - x4 + 5x3 - 2x2 - 6
Step 1 : Call ac the product number (6 x 2 = 1 2), and b Combine like terms:
the sum number (7).
(3x4 - x4) + (5x3 + sx3) - 2x2 - 2x + (2 - 6)
Step 2: Find the two numbers with a product equal
to the product number and a sum equal to the Simplify:
sum number. What two numbers have a product 2x4 + l 0x3 - 2x2 - 2x - 4
CHAPTER 9 I THE SAT MATH: ADVANCED MATHEMATICS 377

Which of the following is equivalent to 2x(x + 1) - Which of the following is equivalent to (2x - 7)(3x + 1)
x2(x + 1) for all values of x? for all values of x?
A) x2 + x A) 6x2 - 7
B) x3 - x2 + 2x B) 6x2 + Sx - 7
C) -x3 + x2 + 2x C) 6x2 - 2lx - 7
D) -x3 + x2 + 2x + 1 D) 6x2 - 1 9x - 7

(Easy) Original expression: 2x(x + 1) - x2(x + 1) (Easy) Original expression: (2x - 7)(3x + 1)
Distribute: 2x2 + 2x - x3 - x2 F OIL : (2x)(3x) + (2x)(l) +
Combine like terms: -x3 + x2 + 2x (-7)(3x) + (-7)(1)
Simplify: 6x2 + 2x - 2lx - 7
Therefore, the correct answer is (C).
Combine like terms: 6x2 - 19x - 7

When multiplying binomials, remember to FOIL. Therefore, the correct answer is (D).

Expression to be multiplied: (ax + b)(ex + d)


To multiply two polynomials, remember to
F (product of the two "first" terms) : ax x ex = (ae)x2 distribute each term i n the first p olynomial to
0 (product of the two "outside" terms): ax x d = (ad)x each term i n the second p olynomial, then sim­
plify. FO!Ling is just a special example of this
I ( product of the two "inside" terms) : b x ex = (be)x kind of distribution.
L (product of the two "last" terms): b x d = bd
F + O + I + L: (ae)x2 + (ad)x + (be)x + bd
Expression to be simplified: (2x2 - x + 2) x (x3 + x - 1)
Distribute: (2x2)(x3) + (2x2)(x) - (2x2)(1) - (x)(x3) -
(x)(x) + (x)(l) + (2)(x3) + (2)(x) - (2)(1)
Simplify: 2x5 + 2x3 - 2x2 - x4 - x2 + x + 2x3 + 2x - 2
Combine like terms: 2x5 - x4 + 4x3 - 3x2 + 3x - 2
378 McGraw-Hill Education: SAT

Lesson 5 : Solving quadratic equations

To solve factorable quadratic equations, first u s e t h e Laws o f Equality t o s e t o n e side o f t h e equation to 0, then
factor and use the Zero Product Property.
Zero Product Property: I f the product of a ny set of numbers is 0, then at least one of those numbers
must b e 0 .

Which of the following is a solution to the equation (Medium) Although we could just plug the numbers
8 - x2 = -2x? i n the choices back i nto the equation to see which one
A) - 4 works, it's a bit of a pain to do that with such obnoxious
numbers. The ugliness of these numbers also tells us
B) -3
that this quadratic is not easily factorable. Therefore, it's
C) -2 probably best to use the Quadratic Formula.
D) -1
Equation to be solved: 3x2 = 4x + 2
We could just plug in each number i n the choices t o the
equation until we find one that works. But it's good to Subtract 4x and 2 to set right side to 0: 3x2 - 4x - 2 = O
know the general method for finding both solutions. I n Use Quadratic Formula:
this case, t h e fact that t h e numbers in t h e choices a r e all
integers suggests that this quadratic is factorable. 4 ± J(-4)2 - 4(3)(- 2) 4 ± ..)16 + 24
X= =
2(3) 6
-----

Equation to b e solved: 8 - x2 = -2x 4 ± J40 4 ± 2Ji0 2 ± Ji0


Subtract 8 and add x2: O = x2 - 2x - 8 6 6 3
Factor using the Product- Sum O = (x - 4)(x + 2) Therefore, the correct answer is (D).
Method:
Use Zero P roduct Property: x - 4 = O or x + 2 = 0, I f x > 0 and x2 - 5x = 61 what is the value of x?
so x = 4 or -2

To solve tougher quadratic e qu atio n s first use,


I I I
the Laws of Equality to set one side of the equa­ (Medium) Equation to be solved: x2 - 5x = 6
tion to 0, then use the Quadratic Formula.
Subtract 6 to set one
Quadratic Formula: If ax2 + bx + c = 0, side to 0: x2 - 5x - 6 = 0
Factor using the
b ± .Jb 2 - 4ac (x - 6)(x + 1) = 0
then x = Product-Sum Method:
-

2a
----

Use the Zero Product


The equation ax2 + bx + c = O has no real solutions Property: x = 6 or x = - 1
if b2 - 4ac < 0. This is because the square root of
a negative number is not a real number. Since the problem states that x > 0, the correct
answer is 6.
Alternately, we could h ave used the Quadratic
Which of the following is a solution to the equation Formula on the equation x2 - 5x - 6 = 0:
3x2 = 4x + 2?
A) 4 - JiO -b ± .Jb2 - 4ac 5 ± JC- 5)2 - 4(1)(- 6)
6 2a 2(1)
B) 2 - JiO 5 ± -149 5 ± 7
---

6 -- = 6 or - l
2 2
C) 2 - JiO
4
D) 2 - JiO
---

3
CHAPTER 9 / THE SAT MATH: ADVANCED MATHEMATICS 379

2x2 - 7x + k = O
If one of the solutions to the equation
x2 bx+ c =
If a quadratic equation has the form + 0, x
is = 5, what is the other possible value of x?
-b
the zeros of the quadratic must h ave a sum of
and a product of c. A) 3
2
This is because ifa = l, the quadratic formula gives B) 2
solutions of -b + � and -b + � .
3
2 2 C) 2
3
Sum of zeros:
D) 3
-b+)b2 -4c + -b-)b2 -4c 2
����-

2 2
-���-

(Medium-hard) We can start by substituting x = 5 i nto


-b+�+-b-� the original equation and solving:
2
-2b
=-=-b Original equation: 2x2 - 7x + k = 0
2 Substitute x = 5: =
2(5)2 - 7(5) + k 0
Product of zeros: Simplify: 15 + k = 0
Subtract 1 5 : k = -15
-b+)b2 -4c -b-)b2 -4c
-���- x -����
Rewrite origin a l equation 2x2 - 7x - 1 5 = 0
2 2 with k = - 1 5:
(-b) 2 -(b2 -4c) b2 -b2 +4c
=
(2x + 3)(x - 5) 0
4 4 Factor with Product-Sum Method:
x = -3/2 or 5
4c
=-=c Use Zero Product Property:
4
Therefore, the correct answer is (A) .
Alternately, we can save a bit of time and effort by
using the theorem above.

Original equation: 2x2 - 7x + k = 0


Divide by 2: 7 k
x- --x+-=0
?

2 2
Since the quadratic is now in the form x2 + bx + c = 0,
we know that the sum of the solutions must be 7/2, or 3.5.
Therefore, if one of the solutions is 5, the other must be
3. 5 =
- 5 - 1 . 5, or -3/2.
380 McGraw-Hill Education: SAT

Lesson 6: Analyzing the sraphs of quadratic functions

Th e graph o f any quadratic function in t h e xy-plane, that i s , a fu nction of t h e form y = J(x) = ax2 + bx + c , has
the following important features:

_ 2a
It is a parabola with a vertical axis of symmetry at x = __!!_ ,

[ )2h�
The y-i ntercept is c, since j(O) = a(0)2 + b(O) + c = c.
I f it crosses the x-axis, it does so at the points -b + - 4ac ) [ ):�
, 0 and
-b - - 4ac
, 0 }
I f a is positive, the parabola is "open up," and if a is negative, it is "open down."
I f the quadratic is i n the form y = a(x - h)2 + k, then the vertex of the parabola is (h, k).

-2).
The graph of the quadratic function y = g(x) i n the x = 3. The zeros of the parabola (the points where y = 0, or
xy-plane is a parabola with vertex at (3, If this where the graph crosses the x-axis) must be symmetric to
graph also passes through the origin, which of the this line. Since the origin is 3 units to the le.ft of this axis, the
following must equal O? other zero must be three units to the right of the axis, or at
A) g(4) the point (6, 0). This means that g(6) must equal 0, and the
correct answer is (C).
B) g(5) Notice that we don't need to do a nything compli­
C) g(6) cated, like find the specific quadratic equation (which
D) g(7) would b e a pain in the neck) .

(Medium) It's helpful to draw a sketch of this parabola so When t h e quadratic function f is graphed i n the
that we can see its shape. xy-plane, its graph has a positive y-intercept and
two di stinct negative x-intercepts. Which of the
y following could b e f?
A) J(x) = -2(x + 3)(x + 1)
B) J(x) = 3 (x + 2)2

\ C)
D)
J(x) = -4(x - 2)(x - 3)
J(x) = (x + l)(x + 3)

(Easy) Since the functions are all given i n factored


form, it is easy to see where their zeros lie by using the
Zero Product Property. The function i n (A) has zeros
ex-intercepts) at x = - 3 and x = - 1 , which are both nega­
x=3 tive, but its y-intercept isf(O) = 2 (3)(1) = - 6, which i s of
-

course not positive. The only choice that gives two dis­

-2),
tinct x-intercepts and a positive value for f(x) i s choice
For this question, the axis of symmetry is key. Since the (D) J(x) = (x + l)(x + 3), which has x-intercepts at x = - 1
parabola has a vertex of (3, its axis of symmetry is and x = -3, and a y-intercept at y = 3 .
CHAPTER 9 / THE SAT MATH: ADVANCED MATHEMATICS 381

The quadratic function h is defined by the equation


h(x) = ax2 + bx + c , where a is a negative constant
and c is a positive constant. Which of the following
could b e the graph of h in the xy-plane?

A) y

B) y

C) y

D) y

(Easy) The graph of y = ax2 + bx + c is an "open down"


parabola if a is negative, and has a y-intercept of c. The
only "open down" parabola with a positive y-intercept is
choice (B).
382 McGraw-Hill Education: SAT

Exercise Set 2 (No Ca lc u l ator)

• •
I f (x - 2)(x + 2) = 0, then x2 + 1 0 = I f x = - 5 is one of the solutions of the equation
0 = x2 - ax - 12, what is the other solution?

I I I I I I I
II
I f (a - 3)(a + k) = a2 + 3a - 1 8 for all values of a, •
what is the value of k?
Which of the following is equivalent to

I I
2a(a - 5) + 3a2(a + 1) for all values of a?

A) 6a4 - 24a3 - 6
B) 5a 5 + 3a2 - lOa
.. C) 3a3 + 5a2 - lOa
When the quadratic function y = l O (x + 4)(x + 6)
D) 3a3 + 2a2 - lOa - 6
is graphed in the xy-plane, the result is a parabola
with vertex at (a, b). What is the value of ab?
Ill
Which of the following functions, when graphed in
the xy-plane, has exactly one negative x-intercept
• and one negative y-intercept?
I f the function y = 3x2 - kx - 1 2 has a zero at x = 3, A) y = -x2 - 6x - 9
what is the value of k? B) y = -x2 + 6x - 9

I I I I I
C) y = x2 + 6x + 9
D) y = x2 - 6x + 9


I f the graph of a quadratic function in the xy-plane is
m
a parabola that intersects the x-axis at x = - 1 . 2 and I f 2x2 + Bx = 42 and x < 0, what is the value
x = 4.8, what is the x- coordinate of its vertex? of x2?

I I I I I
A) 4
B) 9
C) 49
• D) 64
I f the graph of y = a (x - b)(x - 4) has a vertex at
(5, -3), what is the value of ab?

I I I I I When the function y = h(x) = ax2 + bx + c is graphed
in the xy-plane, the result is a parabola with vertex
at (4, 7). I f h(2) = 0, which of the following must also
• equal O?
What is the sum of the zeros of the function A) h(5)
h(x) = 2x2 - 5x - 1 2?
B) h (6)

I I I I I
C) h(8)
D) h(9)
CHAPTER 9 / THE SAT MATH: ADVANCED MATHEMATICS 383

Exercise Set 2 (Ca lc u lator) 'U5@1¥1fW§ &£ w · w ei t if ¥ #M& ¥ & 1 1

Ill Ill
I f x > O and 2x2 - 4x = 30, what is the value of x? The graph of y = f(x) in the xy-plane is a parabola
with vertex at (3, 7). Which of the following must be

I I I I I
equal to f(- 1)?

A) /(2)
B) f{4)
ID C) /(7)
I f x2 + bx + 9 = 0 has only one solution, and b > 0, D) /(15)
what is the value of b?

I I I I I Which of the following fu nctions, when graphed


in the xy-plane, has two positive x-intercepts and a
negative y-intercept?

Ill A) y = -2(x - l)(x + 5)


When y = 5(x - 3 . 2) (x - 4.6) is graphed in the B) y = -2(x + 3)2
xy-plane, what is the value of the y-intercept? C) y = -2(x - 5)2
D) y = -2(x - l)(x - 5)
I I I I I
Which of the following equations has no real
Ill solutions?
When y = 5(x - 3 . 2)(x - 4.6) is graphed in the A) x2 - 3x + 2 = 0
xy-plane, what is the x-coordinate of the vertex?
B) x2 - 3x - 2 = 0
C) x2 + 2x - 3 = 0
D) x2 + 2x + 3 = 0

Ill The graph of the fu nction y = a(x + 6)(x + 8) has a n


I f (2x - l)(x + 3) + 2x = 2x2 + kx - 3 for all values of axis o f symmetry a t x = k . What is the value of k?
x, what is the value of k?
A) -7
-6
I I I B)
C) 7
D) 8

1111 ID
I f b2 + 20b = 96 and b > 0, what is the value of The graph of the quadratic function y = f(x) i n the
b + 10? xy-plane is a parabola with vertex at (6, - 1) . Which
of the following must have the same value as the

I I I y-intercept of this graph?

A) f(-2)
B) /(3 . 5)
C) /( 1 2)
D) /(13.5)
384 McGraw-Hill Education: SAT

EXE RCISE SET 2 ANSWE R KEY

No Calculator 9. c 2a(a - 5) + 3a2(a + 1)


Distribute: 2a2 - lOa + 3a3 + 3a2
1 . 14 (x - 2)(x + 2) = 0
Collect like terms: 3a3 + 5a2 - lOa
FOIL: x2 - 4 = 0
Add 14: x2 + 10 = 14
10. A Substitute x = 0 to find the y-intercept of each
graph. Only (A) and (B) yield negative y-intercepts, so (C)
2. 6 (a - 3)(a + k) = a2 + 3a - 18
and (D) can be eliminated. Factoring the function in (A)
FOIL: a2 + (k - 3)a - 3k = a2 + 3a - 18
yields y = - (x + 3), which has only a single x-intercept at
Equate coefficients: k - 3 = 3; -3k = - 1 8
X = -3.
Therefore k = 6.
11. c 2x2 + Bx = 42
3 . 50 By the Factor Theorem, the parabola has
D ivide by 2: x2 + 4x = 2 1
x-intercepts at x = - 4 and x = -6. The x-coordinate of
Subtract 2 1 : x2 + 4 x - 2 1 = 0
the vertex is the average of these zeros, or -5. To get the
Factor: (x + 7)(x - 3) = 0
y- coordinate of the vertex, we just plug x = - 5 back into
Therefore, x = -7 or 3, but since x < 0, x = -7 and
the equation: y = 10(-5 + 4)(-5 + 6) = 10(-1)(1) = - 10.
therefore, x2 = (-7)2 = 49.
Therefore a = - 5 and b = - 10 and so ab = 50.
12. B D raw a quick sketch of the parabola. Since it
4. 5 When x = 3, y = 0 : 0 = 3 (3)2 - k(3) - 1 2
has a vertex at (4, 7), it must h ave a n axis of symmetry of
Simplify: 0 = 27 - 3k - 1 2
x = 4. The two zeros of the function must b e symmetric
Simplify: 0 = 1 5 - 3k
to the line x = 4, and since the zero x = 2 is two units to
Add 3k: 3k = 15
the left of the axis, the other must by 2 units to the right,
D ivide by 3: k=5
at x = 6.
5. 1 . 8 The x-coordinate of the vertex is the average of
the x-intercepts (if they exist): (- 1 . 2 + 4.8)/2 = 3.6/2 = 1 .8.
Calcu lator
6. 1 8 The x-coordinate of the vertex is the average of
the x-intercepts (if they exist):
5 = (b + 4)/2 13. 5 2x2 - 4x = 3 0
Multiply by 2: 10 = b + 4 Divide by 2: x 2 - 2x = 1 5
Subtract 4: 6=b Subtract 1 5 : x2 - 2x - 1 5 = 0
Substitute x = 5 and y = -3 into equation to find the Factor: (x - 5)(x + 3) = 0
value of a: -3 = a(5 - 6)(5 - 4) = -a Therefore, x = 5 or -3. But since x > 0, x = 5 .
Multiply by - 1 : 3=a
Therefore, ab = (3)(6) = 1 8 14. 6 Let's c a l l t h e o n e solution a . I f it is t h e only solu­
tion, the two factors must b e the same:
7. 2.5 0 = 2x2 - 5 x - 1 2 x2 + bx + 9 = (x - a)(x - a)
Factor: 0 = (2x + 3)(x - 4) FOIL: x2 + bx + 9 = x2 - 2 ax + a2
Therefore, the zeros are x = -3/2 and x = 4, which have Therefore, b = -2a and a2 = 9. This means that x = 3
a sum of 2 . 5 . Alternately, you can divide the original or -3 and so b = -2(3) = - 6 or -2(-3) = 6. Since b must
equation by 2: be positive, b = 6.
0 = x2 - 2 . 5x - 12
and recall that any quadratic in the form x2 + bx + c = O 15. 73 .6 The y-intercept is simply the value of the
must have zeros that have a sum of -b and a product of c. fu nction when x = 0: y = 5 (0 - 3 . 2) (0 - 4.6) = 73.6.
Therefore, without having to calculate the zeros, we can
see that they have a sum of - (-2 . 5) = 2 . 5 . 16. 3 .9 The x- coordinate of the vertex is simply the
average of the zeros: (3. 2 + 4.6)/2 = 3 . 9 .
8 . 2 . 4 We know that o n e o f t h e zeros is x = - 5, and
we want to find the other, x = b. We can use the Factor 17. 7 (2x - l)(x + 3) + 2x = 2x2 + kx - 3
Theorem: FOIL: 2x2 + 5x - 3 + 2x = 2x2 + kx - 3
x2 - ax - 1 2 = (x + 5)(x - b) Simplify: 2x2 + 7x - 3 = 2x2 + kx - 3
FOIL: x2 - ax - 1 2 = x2 + (5 - b)x - 5b Subtract 2x2 and add 3: 7x = kx
Since the constant terms must be equal, 1 2 = 5b and Divide by x: 7=k
therefore, b = 1 2/5 = 2.4.
CHAPTER 9 I THE SAT MATH: ADVANCED MATHEMATICS 385

18. 1 4 b2 + 20 b = 96 2 1 . D This one is tough. Since this question allows a


Subtract 96: b2 + 20b - 96 = 0 calculator, you could solve this by graphing or with the
Factor: (b - 4)(b + 24) = 0 Quad ratic Formula. Remember that a quadratic equa­
Therefore, b = 4 or -24, but if b > 0, then b must tion has no real solution if b2 - 4ac < 0. The only choice
equal 4, and therefore, b + 1 0 = 14. Alternately, you for which b2 - 4ac is negative is (D). Alternately, if you
might notice that adding 100 to both sides of the origi nal graph the left side of each equation as a function in the
equation gives a "perfect square trinomial" on the left xy-plane (which I only advise if you have a good graph­
side: b2 + 20 b + 100 = 196 ing calculator), you will see that the function in (D) never
Factor: (b + 10)2 = 196 crosses the x-axis, implying that it cannot equal 0.
Take square root: b + 10 = ± 14
I f b > 0: b + 1 0 = 14 22. A This quadratic has zeros at x = -6 and
x = - 8, so its axis of symmetry is at the midpoint of
1 9 . C Since the vertex of the parabola is at (3, 7), the zeros, at x = 7.
-

the axis of symmetry is x = 3. Since x = -1 is 4 units


to the left of this axis, and x = 7 is 4 units to the right 23. C If the vertex of the parabola is at (6, - 1), its
of this axis, f(- 1) must equalf(7). axis of symmetry must be x = 6. The y-intercept of the
function isf(O), which is the value of y when x = 0. Since
20. D y = -2(x - l)(x - 5) has x-intercepts at x = 1 this point is 6 units to the left of the axis of symmetry,
and x = 5 and a y-intercept of y = - 10. (Notice that the its reflection over the axis of symmetry is 6 units to the
function in (C) has only one positive x-i ntercept at x = 5.) rights of the axis, atf(l2).
386 McGraw-Hill Education: SAT

Lesson 7: Analyzing polynomia l equations

The Factor Theorem

If a polynomial expression has a zero (a value of x for which the polynomial equals 0) at x = a, it must
have a factor of (x a) .
-

Conversely, if a polynom ial has a factor of (x - a), it must have a zero at x = a.

The function f is defined by the equation f(x) = x3 - Which range of values defines all of the values of x
ax2 - bx + 20 where a and b are constants. I n the for which the function f i n the previous question i s
xy-plane, the graph of y = f(x) intersects the x-axis positive?
at the points (-2, 0), (2, 0), and (p, 0). What is the
A) x < -2 or x > 2
value of p?
B) -2 < x < 5
A) 4
C) -2 < x < 2 or x > 5
B) 5
D) 2 < x < 5
C) 1 0
D ) 20
When analyzing a p olynomial function, you
(Medium-hard) Since x = -2 and x = 2 and x = p are may find it very helpful to draw its graph in the
zeros of the fu nction (that is, they are inputs that yield xy = plane. Sometimes the x-and y-intercepts are
an output of O), the polynom ial must have (x + 2), (x - 2), all you need to get a good picture by hand. You
and (x - p) as factors. should also know how to use the graphing func­
tion on you r calculator, when it i s permitted.

f(x) = x3 - ax2 - bx + 20 = (x + 2)(x - 2)(x - p)


(Hard) This question is easier to solve if we have a graph
FOIL (x + 2) (x - 2) : = (x2 - 4)(x - p)
of the fu nction. Since we know that the equation of the
FOIL (x2 - 4) (x - p) : = x3 - px2 - 4x + 4p fu nction is y = (x + 2) (x - 2)(x 5), we know that it has
-

x-intercepts at x = -2, x = 2, and x = 5, and a y-intercept


Since x3 px2 - 4x + 4p must be equivalent to x3 - ax2 -
-
at y = (0 + 2)(0 - 2)(0 - 5) = 20. Therefore, the graph
bx + 20, a l l of the corresponding coefficients must be looks like this:
equal. That is, -p = -a, - 4 = -b, and 4p = 20. Therefore, y
p = 5, a = 5, and b = 4, and the correct answer is (B).

On this graph, the points where f is positive are the


points above the x-axis. Thi s corresponds to the points
where x is between -2 and 2, and where x is greater
than 5. Therefore, the correct answer is (C) .
CHAPTER 9 I THE SAT MATH: ADVANCED MATHEMATICS 387

Lesson 8: Systems involving quad ratics


y y= l
x2 + y2 = 4
y = x2

How many distinct ordered pairs (x, y) satisfy the


three-equation system above?
A) Zero B) One C) Two D) Three

The figure above shows the graph of a system of two


(Medium) To find the solutions of a system means to
find the ordered pairs (x, y) that satisfy all of the equa­
equations in the xy-plane. How many solutions does
tions simultaneously. Although graphing this system is
this system h ave?
not too hard, it is probably simpler to solve this system
A) Zero B) One C) Two D) Three algebraically.

(Easy) Finding the solutions to a system of equations means Substitute the first equation,
finding the ordered pairs that satisfy all of the equations y = 1 , into the other two: x2 + (1)2 = 4
simultaneously. (Ifyou need to review how to solve systems, 1 = x2
see Chapter 7.) If the equations are graphed, the solutions
Use x2 = 1 to substitute into
correspond to any points where all of the graphs meet. In
other equation: (1) + (1)2 = 4
this case, the two graphs cross in two distinct points, so the
system has two solutions and the answer is (C). Simplify: 2=4

y + 2x = 6 Since this yields an equation that can never b e true,


regardless of the values of the u nknowns, there is no real
y = x2 + 3x
solution to this system, and the correct answer is (A).
I f you graph this system, it will show a horizontal
Given the system above, which of the following
line, a circle, and a parabola. You will see that no point
could b e the value of y?
exists where all three graphs meet, indicating that the
A) 1 or - 6 system has no solution.
B) 0 or - 5
C) 0 or 1 0
D) 4 or 1 8

(Medium) Perhaps t h e simplest way t o solve t h i s system


is with the process of substitution, which we applied to
7,
linear systems in Chapter Lesson 1 2 .

First equation: y + 2x = 6
Substitute y = x2 + 3x: x2 + 3x + 2x = 6
Subtract 6: x2 + 5x - 6 = 0
Factor with P roduct-Sum Method: (x + 6)(x - 1) = 0
Apply Zero-Product P roperty: x = - 6 or 1
But b e careful. You may b e tempted to choose (A) 1 or
- 6, but the question asks for the value of y, not x. To find
the corresponding values of y, we must plug our x-values
back into one of the equations: y = (- 6)2 + 3(- 6) = 1 8 or
y = (1)2 + 3(1) = 4; therefore, the correct answer is (D).
388 McGraw-Hill Education: SAT

Exerc ise Set 3 ( No Ca lculator)

• • ---

If x3 - 7x2 + 1 6x - 1 2 = (x - a)(x - b)(x - c) for all xz + yz = 9


values of x, what is the value of abc?
Which of the following equations, if graphed in the

I I I I I
xy-plane, would intersect the graph of the equation
above in exactly one point?
A) y = - 4
B) y = - 3

C) y = -1
I f x3 - 7x2 + 1 6x - 1 2 = (x - a)(x - b)(x - c) for all
values of x, what is the value of a + b + c? E) y = O

I I I I •
I f g(x) = a(x + l)(x - 2)(x - 3) where a is a negative
constant, which of the fol lowing is greatest?
.. A) g(0. 5)
1 . I f x3 - 7x2 + 1 6x - 1 2 = (x - a)(x - b)(x - c) for all B) g(l . 5)
values of x, what is the value of ab + be + ac? C) g(2 . 5)
D) g(3 . 5)
I I
..
• I f 2x2 + ax + b has zeros at x = 5 and x = - 1 ,
what is the value o f a + b?
I f x2 - ax + 1 2 has a zero at x = 3, what is the value
of a? A) -18
B) -9

I I I I I C)
D)
-2
-1

.. Ill
I f x2 - ax + 1 2 has a zero at x = 3, at what other I f the graph of the equation y = ax4 + bx i n the
value of x does it h ave a zero? xy-plane passes through the points (2, 1 2) and
(-2, 4), what is the value of a + b?

I I I I A) 0.5 B) 1 . 5 C) 2 0.

D) 2 . 5


Ill
---

y = 4x2 + 2
I f the function y = 3 (x2 + l)(x3 - l)(x + 2) is graphed
x + y = 16
in the xy-plane, in how many distinct points will it
When the two equations in the system above are intersect the x-axis?
graphed in the xy-plane, they intersect in the point
A) Two B) Three C) Four
(a, b). I f a > 0, what is the value of a?
D) Five
CHAPTER 9 / THE SAT MATH: ADVANCED MATHEMATICS 389

Exercise Set 3 (Ca lcu lator}


I f x2 + y = lOx and y = 25, what is the value of x? For how many distinct positive integer values of n is
l
(n - ) ( n - 9)(n - 17) less than O?

I I I I A) Six
B) Seven
C) Eight
Ill D) ine
I f 2x3 - 5x - a has a zero at x = 4, what is the value
of a? Ill
x2 + 2y2 = 44

I I I I I y2 = x - 2
When the two equations above are graphed i n the
xy-plane, they intersect in the point (h, k). What is
• the value of h?
I f x > 0 and x4 - 9x3 - 22x2 = 0, what is the value of x? A) - 8
B) - 6

I I I I I C)
D)
6
8

m2 + 2 n = 10
I f d is a positive constant and the graph in the
xy-plane of y = (x2)(x2 + x - 72) (x - d) has only one 2m2 + 2 n = 14
positive zero, what i s the value of d? Given the system of equations above, which of the
following could be the value of m + n?

I I I I I A) -7
B) -2
C)
D) 2
y = 2x2 + 1 8
y = ax Ill
I n the system above, a is a positive constant. When For how many distinct values of x does (x2 - 4)
the two equations are graphed in the xy-plane, they (x - 4)2(x2 + 4) equal O?
intersect i n exactly one point. What is the value of a? A) Three
B) Four

I I I I I C) Five
D) Six

• m
4a2 - 5b = 16 The function fl.x) is defined by the equation
3a2 - 5b = 7 fl.x) = a(x + 2)(x - a)(x - 8) where a is a constant.
Iffl.2 .5) is negative, which of the following could be
Given the system of equations above, what is the the value of a?
value of a2b2?
A) -2
B)
I I I I I
0
C) 2
D) 4
390 McGraw-Hill Education: SAT

EXE RCISE SET 3 ANSWE R KEY

No Calculator Divide by 6: a = -8
Substitute a = - 8: -8 - b = 2
1. 12 When the expression (x - a)(x - b) (x - c) is Add 8: -b = 10
fu l ly distributed and simplified, it yields the expression Multiply by - 1 : b = - 10
x3 - (a + b + c)x2 + (ab + be + ac)x - abc. If this is equiv­ Therefore, a + b = - 8 + - 1 0 = - 1 8.
alent to x3 - 7x2 + 1 6x - 12 for all values of x, then all of
the corresponding coefficients must be equal. 10. D
Substitute (2, 1 2) : 1 2 = a(2)4 + b(2)
2. 7 See question 1 . Simplify: l6a + 2 b = 1 2
Substitute (-2, 4) : 4 = a(-2)4 + b(-2)
3 . 1 6 See question 1 . Simplify: l6a - 2b = 4
Add two equations: 32a = 1 6
4. 7 If x2 - ax + 1 2 = O when x = 3, then Divide by 32: a = V2
(3)2 - 3a + 1 2 = 0 Substitute: 16(1 /2) + 2 b = 1 2
Simplify: 2 1 - 3a = O Subtract 8: 2b = 4
Add 3a: 2 1 = 3a Divide by 2 : b=2
Divide by 3: 7=a Therefore, a + b = 2 . 5 .

5. 4 A s we saw in question 4, a = 7. 1 1 . A Use the Zero Product Property. The factor


x2 - 7x + 1 2 (x2 + 1) can not b e zero for a ny value of x, (x3 - 1) is zero
Factor: (x - 3)(x - 4) when x = 1 , and (x + 2) is zero when x = -2. Therefore,
Therefore, the zeros are 3 and 4. there are only two distinct points i n which this graph
touches the x- axis.
6. 7/4 o r 1 . 75 x + y = 16
Subtract x: y = 16 - x
Substitute: 16 - x = 4x2 + 2
Subtract 16, add x: 0 = 4x2 + x - 14 Calculator
Factor: 0 = (4x - 7)(x + 2) 12. 5 Substitute y = 25: x2 + 25 = lOx
Therefore, x = -2 or 7/4, but if x must be positive, it Subtract lOx: x2 - l Ox + 25 = 0
equals 7/4 . Factor: (x - 5)(x - 5) = 0
Use Zero Product Property: X=5
7. B The graph of the given equation is a circle
centered at the origin with a radius of 3. Therefore, the 13. 1 08 I f x = 4 is a zero: 2(4)3 - 5(4) - a = 0
horizontal line at y = -3 j ust intersects it at (0, -3). You Simplify: 108 - a = 0
can also substitute y = -3 into the origi nal equation and Add a: 108 = a
verify that it gives exactly one solution.
14. 1 1 x4 - 9x3 - 22x2 = 0
8. C Just notice the sign of each factor for each input: Divide by x2: x2 - 9x - 22 = 0
g(0.5) = (-)(+) (-)(-) = negative Factor: (x - l l) (x + 2) = 0
g(l . 5) = (-) (+) (-) (-) = negative Use Zero Product Property: x = 1 1 or -2
g(2 . 5) = (-)(+) (+) (-) = positive
g(3 .5) = (-)(+)(+)(+) = negative 15. 8 y = (x2)(x2 + x - 72)(x - d)
Si nee (C) is the only option that yields a positive value, it Factor: y = (x2)(x + 9)(x - 8)(x - d)
is the greatest. By the Zero Property, the zeros are x = 0, - 9, 8, or d.
Since d is positive, but there can only b e one positive
9. A 2x2 + ax + b zero, d = 8.
I f x = 5 is a zero: 2(5)2 + 5a + b = 0
Subtract 50: 5a + b = - 50 16. 1 2 y = 2x2 + 1 8
I f x = -1 is a zero:
Subtract 2:
2(-1)2 + a(- 1) + b = O
-a + b = -2
y
Substitute = ax: ax = 2x2 + 1 8
Subtract ax: O = 2x2 - ax + 1 8
Multiply by - 1 : a-b=2 a
Add equations: 6a = - 48 Divide by 2: O = x2 - - x + 9
2
CHAPTER 9 I THE SAT MATH: ADVANCED MATHEMATICS 391

If the graphs intersect in only one point, the system Distribute: x2 + 2x - 4 = 44


must h ave only one solution, so this quadratic must be a Subtract 44: x2 + 2x - 48 = O
"perfect square trinomial" as discussed in Lesson 4 . Factor: (x - 6)(x + 8) = O
a This seems to imply that the x-coordinate of the point of
x 2 - - x + 9 = x 2 - 2bx + b 2
2 intersection could be either 6 or - 8, both of which are
choices. Can they both be correct? No: if we substitute
Equate coefficients: b2 = 9
x = - 8 into either equation, we get no solution, because
2b = a/2
y2 cannot equal - 8 . Therefore, the correct a nswer is (C)
The only positive solution to this system is b = 3 and
6, and the points of intersection are (6, 2) and (6, -2).
a = 12.

17. 1 44 4a2 - Sb = 1 6 20. c 2m2 + 2 n = 14


3 a 2 - Sb = 7 m2 + 2n = 1 0
Subtract equations: a2 = 9 Subtract equations: m2 = 4
Substitute a2 = 9: 3 (9) - Sb = 7 Take square root: m = ±2
Subtract 27: -Sb = -20 Substitute m2 = 4: 4 + 2n = 10
Divide by -S: b=4 Subtract 4: 2n = 6
Therefore, a2b2 = 9 (4)2 =144. Divide by 2: n=3
Therefore, m + n = -2 + 3 = 1 or 2 + 3 = S .
1 8 . B In order for the product of three numbers to be
negative, either all three numbers must be negative or 2 1 . A U s e t h e Zero Product Property. (x2 - 4 ) equals
exactly one must be negative and the others positive. Since 0 if x is 2 or -2, (x - 4) equals O if x is 4, and (x2 + 4)
n must be a positive integer, n - 1 cannot be negative, and cannot equal 0. Therefore, there are exactly three dis­
so there must be two positive factors and one negative. The tinct zeros.
only integers that yield this result are the integers from 1 0
t o 1 6, inclusive, which i s a total o f seven integers. 22. C j(2 . S) = a (2 . S + 2)(2 . S - a)(2 . S - 8)
Simplify: (-24.7S)(a)(2. S - a)
19. c x2 + 2y2 = 44 This product can only be negative if a and (2. S - a) h ave
Substitute y2 = x - 2: x2 + 2 (x - 2) = 44 the same sign, which is only true for (C) a = 2.
392 McGraw-Hill Education: SAT

Skill 3 : Working with Exponentials and Radicals


Lesson 9: The Laws of Exponentials
1 1
When working with exponentials you must understand e.g., 2 - 3 = t ...:.. .. 2 ...:.. .. 2 ...:.. .. 2 = - = -
the Laws of Exponentials. 23 8

Law #4: xm x xn xm + n (When multiplying expo­


=

Law #1: I f n is a positive integer, then xn means nentials with equal bases, add the exponents.)
the result when I is multiplied by x repeatedly
n times.
e.g., x3 x x2 = ( 1 x x x x x x) x (1 x x x x)
= l x x x x x x x x x x = x5
e.g., 3 5 = 1 x 3 x 3 x 3 x 3 x 3 = 243

Law #5: xn X y" (xy)n (When multiplying


You might think that it's unnecessary to include the 1 in =

this product, but including it will help clarify what zero, exponentials with equal exponents, multiply the
negative, and fractional exponents mean. For instance, bases.)
think about the following sequence:
243, 81, 27, 9, 3, ___ , ___ , _ __ This law follows from the Commutative and Associative
Laws of Addition.
What are the missing three terms in this sequence?
With a little trial and error, you will see that the rule e.g., 43 x 72 = (1 x 4 x 4 x 4) x (1 x 7 x 7 x 7)
for getting each term is "divide the previous term by 3," = 1 x (4 x 7) x (4 x 7) x (4 x 7)
and therefore the missing terms are 1, 1 /3, and 1 /9. But = (4 x 7)3
notice, also, that these terms are just the descending
xm
integer powers of 3:
Law #6: xm - n (When d ividing exponentials
x"
=

35 = 1 x 3 x 3 x 3 x 3 x 3 = 243
with equal bases, subtract the exponents.)
34 = 1 x 3 x 3 x 3 x 3 = 81
33 = 1 x 3 x 3 x 3 = 27
32 = I x 3 x 3 =9 X5 lxxxxxxxxxx
e.g., l x x x x = x2
31 = 1 x 3 =3 x3 lxxxxxx
3° = 1 =l

(�)n
3 - I = 1 -:- 3 = l /3
xn
3 -2 = 1 -:- 3 -:- 3 = 1 /9 Law #7: = (When dividing exponentials
y" y
And so on. I f you explore this pattern, and patterns for with equal exponents, divide the bases.)
the powers of other numbers, you will notice that some

[ )
other laws clearly emerge.
503 50 3
e.g., = = 23 = 8
Law #2: As long as x does not equal 0, x6 = I. 253 25

You can think of :(! as meaning "l multiplied by x zero


times, or not at all." Therefore, the result is 1 .

e.g., (x3)2 = (1 x x3 x x3) = ( 1 x (1 x x x x x x)


Law #3: I f n i s a positive integer, then x-n means x (1 x x x x x x)) = 1 x x x x x x x
the result when I is divided by x repeatedly x x x x x = x6
n times.
1
In other words, x-• I

xn Law #9: x;;- efX


= -.

=
CHAPTER 9 I THE SAT MATH: ADVANCED MATHEMATICS 393

"
Proof: This follows directly from Law #8. If we raise X " to Which of the following expressions is equivalent to
the nth power, by Law #8 we must get x 1 or x. The number 3 X 32n
gn
-- for all values of n?
that we must raise to the nth power in order to get x is, by
definition, the "nth root of x."

Law #IO: Ifx > I and x" = x" , then a = b.


A)
B) 3
(�f
C) 3 n
Which of the following expressions is equivalent to D) 92 n
(n + n + n)(n + n + n )
7.
3- 1 32n 32n
(Medium) Express all terms in 3X
--
31
X
---
gn
exponential form: 9"
A)
3 32n + I
gn
Apply Law #4:
B) 3n2
C) 3n6 32n + I
Substitute 9 = 32:
D) 27n2 (3 2 ) "
Apply Law #8 to the 32 n T I
32n
(Medium) (n + n + n)(n + n + n )
denominator:
3- I
Apply Law #6: 32n + I - Zn = 3 1 = 3
Simplify numerator: (3n)(3n)
3- 1
Therefore, the correct answer is (B).
Simplify numerator 9n 2 Alternately, we can plug in various values for
and apply Law #3 to n and find that the expression gives a value of n no
denominator: 3 matter what.
To divide by a number is to
multiply by its reciprocal: 9n2 x 3 = 27n2

Therefore, the correct answer is (D).


394 McGraw-Hill Education: SAT

Lesson 1 0: The Laws of Radicals

Th e radical symbol (../) is used t o indicate roots, which are the inverse o f exponentials. For instance, because
23 = 8, we can say that 2 is the "third root " or "cube root" of 8 (2 = if8 ).
Law #9 of exponentials shows us that radicals (or "roots" ) can be expressed as exponentials. For instance,
if8 = at. Therefore, we can use the Laws of Exponentials to simplify radical expressions.

( 2Fx r = ( 2c2x)� r = (2) 6 ( c2x)� r Which of the following is equivalent to 2


e.g.,
J2 + 4,/IB ?
(No calculator)
J2
= (2) 6 (2x )3 = (2)6 (2)3 ( x )3 = 29 x 3
A) 6M B) 7 C) 14 D) 19

(Medium) Notice that each answer choice is much sim­


I

Law #1: ifX x;; ( This is just the "reflected"


=
pler than the original expression. This suggests that the
version of Law of Exponentials #9.) original expression can be simplified. Let's begin by look­
ing at the radical expressions . I f you know your p erfect
Law #2: ifX x efY efXJ ( Thi s follows directly
= squares you will see that neither radicand (the expres­
from Law of Exponentials #5.) sion inside the radical) is a perfect square, but one of

n� (
the radicands-1 8-is a multiple of a p erfect square:
Law #3: � = v-Y This follows directly from Law 18 2
= x 9.
�y
o f Exponentials #7.) 2J2 +4 ,/IB
Original expression:
J2
Substitute 1 8 = 9 x 2: 2J2+4�
Working with square roots is much easier if you
J2
memorize the first 10 or so "perfect square Apply Law #2: 2J2+4J9xJ2
integers": J2
32 = 42 = 52 = 62 = 72 = J9 = 3: 2J2 + I2J2 = I4J2 = 14
8222== 92 = 102 = 112 = 122 =
4, 9, 16, 25, 36, 49, Simplify
64, 81, 100, 121, 144 . . . J2 J2
This will help you both simplify and estimate
radical expressions. Therefore, the correct answer is (C).
• If the radicand has a perfect square factor, the If x =
2 4, y2 = 9, and (x - 2)(y + 3) 0, what is the
""'
radical can be simplified by factoring. value of x + y?
e.g., m J36 = x J2 = 6J2 C) D) 5
A) -5 B) -1
• If a fraction has a radical in the denominator,
eliminate it by multiplying numerator and
Every positive number has two distinct square
denominator by the radical.
roots. For instance, both and -5 are the square
5
e.g., I+ J2 = 1 + J2 x J3 = J3 +J6 root of 25,
because (5)2 = 25 (-5)2 = 25.
and
J
J3 J3 J3 3 However, the symbol means the principal, or
non-negative square root, so J25 = 5
and not -5.
• To estimate the value of square roots, notice
which two consecutive perfect squares the
radicand lies between. (Easy) If x2 = 4, then x = ±2, and if y2 = 9, then y = ±2.
But if (x - 2)(y + ""' 0, the x cannot equal 2 and y cannot
3)
e.g., J64 < m < JITT. and therefore < m < 8 9 equal -3. Therefore, x = 2 and y = and x + y = l, so
- 3,
the correct answer is (C).
CHAPTER 9 / THE SAT MATH: ADVANCED MATHEMATICS 395

Lesson 1 1 : Solvin g radica l and exponential eguation s

1
x+2 = J2, what is the value of x?
I f -- If 3y = t· what is the value of y3?
A)
1+2J2
-- 2
2 A)
9

B)
l-2J2 B)
4
2 9
l-2J2 C)
2
C)
� 3
1+2J2 D )
4
D)

3
(Hard)
_x+2l_ =J2
(Hard ) 3y= H
Multiply by (x + 2): l=J2(x+2)
Distribute: l = Ex+2J2 Use Radical Law #3: 3y= JYJ2
Subtract 2J2: l-2J2=J2x Multiply by JY: 3yJY=J2
1+2J2 #1: l
3/y' = 2' l
Divide by J2:
�=X Use Radical Law

Use Exponential Law #4: 3y 2 = 2 2


1 l

Therefore, the correct answer is ( D ) . l

.
Divide by 3: y' =-2'3
1

1 {";;
Ifk
2 = 4v2, what is t h e value o f k? Square both sides: y = -92
3

A) -3
5 Therefore, the correct answer is (A).
B)
2
3
C)
2
7
D)
2
(Medium-hard ) �=4J2
2k
Use Exponential Law #3: 2- k = 4J2
Use Radical Law #1:
Substitute 4 = 22: 2- k = 22 2t
x

Use Exponential Law #4: 2- k = 21


I f 2a = 2b, then a = b: - k =-25
Multiply by - 1 : k=--25
396 McGraw-Hill Education: SAT

Exerc ise Set 4 (No Calculator)

• •
If 2a2 + 3a - 5a2 = 9, what is the value of a - a 2 ? a 3 - vr;:� -3
If __ = -- , what is the value of (ab )2 ?
3 + ./5 b

I I I I I I I I I I

Ill
I f (200)(4,000) = 8 x 10 m , what is the value of m?
I f g x = 25, what is the value of 3 x- 1 ?

I I I A)
3
25
B)
5
-
3
C)
25
3
D) 24

• •
2x . .
--
8w 2 If g(x, y) = 3 and a and b are pos1t 1ve numbers,
I f w = -1030, what is the value of ? y
(8w) 2 . (4a 2b) ?
what 1s the value of g '
g(a, b)

I I A)
4
B)
1
2
C) 2 D) 4

• 1111 __

I f 2 x = 10, what is the value of 5(22 x) + 2 x? 2" x 2"


Which of the following is equivalent to
2" x 2
I I I I for all positive values of n?

A) 2 B) 2n C) 2 n - 1
D) 22n

I f (x + 2)(x + 4)(x + 6) = 0, what is the greatest •
possible value of _.!:.__ ? Which of the following is equivalent to 3 m + 3 m + 3 m
2x for all positive values of m?

I I I I I A) 3m + 1
D) 33 m + I
B) 32 m C) 33 m

• ..
( r
I f 4 + 4..Jz = a + b ..J2 , where a and b are integers, I f x is a positive number and 5 x = y, which of the
what is the value of a + b? following expresses 5y2 i n terms of x?

52x B) 52 x + 1 C) 53x
I I I I I A)
D) 252x
CHAPTER 9 / THE SAT MATH: ADVANCED MATHEMATICS 397

Exercise Set 4 (Ca lcu lator)

Ill Ill
I f n 2 = M and n > 0, what is the value of n? What is one possible value for x such that
O < i x < JX < X?.
5

I I I I I I I I I I

What is the smallest integer value of m such that
Which of the following is equivalent to
1
-- < 0.000025? 4
lQm for all positive values of x?
2 - 2 (x + x)(x + x)

I I I I I
4 1 16
A) C) D)
4x 2
B)
x' x2 x•

Ill
DI The square root of a certain positive number is twice
If _2_k = 9 m, what is the value of k?
r
the number itself. What is the number?

1 1 1 1
I I I I I A) B) - C) - D)
8 4 2 J2

m
Ill Which of the following is equivalent to
I f (xm )3(xm + 1 ) 2 = x3 7 for all values of x, what is the 2m En + m mn . .
for all pos1t1ve values of m and n?
value of m? m J2
A) 3m J3;i

I I I B)
C)
sm J2
3 .j3;,,
D) s Fr'z
..
I f 9 .Jl2 - 4Jz::i = nJ3, what is the value of n? m

I I I I I
x
Ill
( 2-fi f", what is the value of n?
In the figure above, if n > 1 , which of the following
expresses x in terms of n?
If si =

A) .Jn 2 - 1
I I I I I B) .Jn - 1
C) .Jn + l
.Jn - 1
D)
2
398 McGraw-Hill Education: SAT

EXE RCISE SET 4 ANSWER KEY

No Calculator 2( 4a) b3
Simplify: = -- x -
(2b)3 2a
1. 3 2a2 + 3a - 5a2 = 9
8ab3 1
Simplify: 3a - 3a2 = 9 Simplify: --
-
Divide by 3: a - a2 = 3 1 6ab3 2

2. (200)(4,000) = 800,000 = 8 x 105 2" x 2"


5 10. c
2" x 2
8w 2 2"
3. 1 /S o r . 1 25 Cancel common factor:
z1
(8w) 2
8w 2 Exponential Law #6: 2n - J
Exponential Law #5:
64w 2
1 11. A 3m + 3m + 3m
Cancel common factors:
8 Combine like terms: 3 (3 m)
Exponential Law #4: 3m+ l
4. 510 5(22x) + 2 x
Exponential Law #8: 5(2 x)2 + 2 x 12. B 5y2
Substitute 2 x 10:
= 5(10)2 + 10 Substitute y = sx: x
5(5 ) 2
Simplify: 5(10)2 + 10 = 510 Exponential Law #8: 5(52 x)
Exponential Law #4: 52 x+ l
5. 6 4 I f (x + 2)(x + 4)(x + 6) = 0, then x = -2, - 4, or
- 6. Therefore rx could equal 22, 24, or 26• The greatest of
these is 26 = 64.
Calculator

6. 80 ( 4 + 4.J2 ) 2 13. 64 n2 = M
Radical Law # l n2 = (64 4) 11 2
FOIL: ( 4) 2 + 2( 4) ( 4.fi ) + ( 4.fi r Exponential Law #8: n2 = 642
Simplify: 16 + 32.fi + 32
1
48 + 32.fi 14. 5 - < 0.000025
Simplify: 10m
Scientific Notation: 1 X 10- m < 2.5 X 10-5
Therefore a = 48 and b = 32 and a + b = 80.
Substitution and checking makes it clear that m = 5 is
the smallest integer that satisfies the inequality.
7. a 3 - .JS
8 =
3 + .J5 - b- 15. = 9 J27
2.5 �
3- k
Cross-multiply: ab = ( 3 + .JS )( 3 - .JS ) 3 1 - (-k) = g J27
Exponential L a w #6:
Simplify: ab = 9 - 5 = 4 Simplify: 3 k + I = 9 x 3.f3
Therefore ab3 1 2 = 4 3 1 2 = 8 Express as exponentials: 3 k + I = 3 2 X 3 X 31
Exponential Law #4: 3k + l = 33 5.

8. 5/3 or 1 .66 or 1 .67 gx = 25 Exponential Law #10: k + 1 = 3.5


Substitute 9 = 32: (32Y = 25 Subtract 1: k = 2.5
Exponential Law #8: 32 x = 25
Take square root: 3x = 5 16. (xm) 3 (xm + l ) 2 = x3 7
3x 5
7
Divide by 3: Exponential Law #8: (x3m)(x2 m + 2 ) = x3 7
31 3 Exponential Law #4: x5 m + 2 = x3 7
Exponential Law #10: Sm + 2 = 37
Exponential Law #6: 3x - I = �
3 Subtract 2: 5m = 35
Divide b y 5: m=7
2(4a)
9. B g (4a, 2b) (2b)3 17. 6 gJU 4J27 = n .f3
g (a, b) 2a
-

b3 Factor: gJ4 x .f3 - 4J9 x .f3 = n .f3


CHAPTER 9 I THE SAT MATH: ADVANCED MATHEMATICS 399

Divide by Ji 9 14 - 4 ../9 = n 4
20. B
Simplify: 18 - 12 = 6 = n Z-2(x + x)(x + x)

Simplify:
4 x 22
18. 6 (2x)2
Substitute 8 = 23: Simplify: 16
1 4x2
2' = 2"
2L

Exponential Law #8:


1 n 4
Exponential Law #10: Cancel common factor:
2 12
Multiply by 12: 6=n
21. BTranslate: fx = 2x
Square both sides: X = 4X2
19. 1 < x '.S 1 . 56 1
Divide by 4x: -=x
4
M iddle inequality: i x < fx
5
16 22. 2mEn + mJIBn
Square both sides: - x2 < x D
25 m.J2
16
Divide by x: -x < l 2m.J2fn + mJ9.J2fn
25 Factor terms:
m.J2
( Since x > 0, we do not "swap" the inequal ity. )
25 Cancel common factors: 2.Jn + J9fn
Multiply by 25/16: x < - = 1 .563 2.Jn + 3.Jn = 5.Jn
16 Combine like terms:
Last inequality: fx < x
Square both sides: x < x2 23. B Pythagorean Theorem : 12 + x2 = ( .rn r
Divide by x: 1 <x
Therefore, x must b e both greater than 1 and less than or Simplify: l + x2 = n
equal to 1 .56. Subtract 1 : x2 = n - l
Take square root: x = .Jn - 1
400 McGraw-Hill Education: SAT

Skill 4:
Working with Rational Expressions
Lesson 1 2: I nterpreting and computing with
rationa l expressions

Which o f t h e following is equivalent t o � - __


x x+1
l
If � is equivalent to
3x + 1
-1-
3x + l
+ B for all x, which
for all x greater than O?
of the following is equivalent to B?
2
3x - 1
A)
x+l A)
1 B) 3x + 1
B)
x(x + l) C) 9x2
x+2 D) 9x2 - 1
C)
x2 + x
(Hard) It helps to notice that the given rational expres­
3 sion is "improper," but that the transformed expression
D)
x+l is not. Recall that an "improper fraction," like 5/3, is one
in which the numerator is larger than the denominator.
When adding, subtracting, multiplying, or divid­ Such fractions can also b e expressed as "mixed num­
ing rational expressions, just fol low the rules for bers," which include an integer a nd a "proper fraction: "
working with fractions. 5/3 = 1 % . Similarly, an "improper rational expression"
is one in which the degree of the numerator is greater
• When adding or subtracting fractions, first than the degree of the denominator. In the expression
get a common denominator, then combine
numerators.
--9x 2
3x + l
, the numerator has a degree of 2 and the
denominator has a degree of 1 . Just as with improper
--
x + l --
-
x (x + l)(x - 1) x(x) fractions, we can convert this to a "mixed" expression by
x x-1 (x)(x - 1) (x)(x - 1) just doing the division:
(x2 - l) - x2 -- --
-1 1
)
3x - l
(x)(x - l) x2 - x x - x2 3x + 1 9x 2 + ox + O
1
9x 2 + 3x
x(l - x)
- 3x + O
• When multiplying fractions, just multiply
- 3x - l
straight across. 1

--
x + l --
x
X = --
(x + l)(x) = x + l 9x 2
which means that -- equals 3x - 1 +
1
--
. Therefore,
x-1 (x)(x - 1) x - 1 3x + l 3x + l
X
the correct answer is (A).
• To divide by a fraction, just multiply by its
reciprocal.

--
x+1
X
x
-'- -- = -- X
• x-1 x x
--
x + 1 x -1 = --
x2 - 1
x2

(Medium) To simplify this difference of fractions, we


must find a common denominator.

2 1 2(x + l) lx 2x + 2 - x x+2
x x+l x(x + l) x(x + l) x(x + 1) x2 + x

So the correct answer is (C) .


CHAPTER 9 I THE SAT MATH: ADVANCED MATHEMATICS 401

1 1 (Medium-hard) You may find it helpful to review


-+b=­ Chapter 8, Lesson 5, "Rates and unit rates" before tack­
x y
ling this problem. We are told that x represents the
Let x represent the time, in hours, it takes pump A to number of " hours per tank" for pump A, that is, the num­
fill a standard tank, and let y represent the time, in ber of hours it takes pump A to fill one standard tank.
hours, it takes pump A and pump B, working together, Therefore, its reciprocal, l /x, must represent the number
to fill the same standard tank. If the equation above of "tanks per hour" for pump A, that is, the number of
represents this situation, then b must represent tanks (or fraction of a tank) that pump A can fill i n one
hour. Likewise, since y represents the number of " hours
A) the time, i n hours, it takes pump B, working per tank" when the two pumps work together, l /y must
alone, to fill the standard tank represent the number of "tanks per hour" that the two
B) the portion of the standard tank that pump B pumps can fill when working together.
fills when the pumps work together to fill the The essential fact i n this situation is that "the rate
entire standard tank (in tanks per hour) at which the two pumps work
C) the rate, in standard tanks per hour, of pump B together must equal the sum of the rates (in tanks
D) the difference between the rates, in standard per hour) of the two pumps working separately." (For
tanks per hour, of pump B and pump A instance, if pump A can fill 2 tanks per hour and pump
B can fill 3 tanks per hour, then working together they
Rational expressions are often used to express can fill 5 tanks per hour.)
rates. (Remember: rate, rational, and ratio all Since the given equation essentially says, "the
derive from the same Latin root.) When work­ rate of pump A plus b = the rate of pump A and pump
ing with rational expressions that represent real B working together," b must represent the rate (in
quantities, it often helps to think in terms of the
tanks per hour) of pump B. Therefore, the correct
rate-units that they represent.
answer is (C).

For instance, if t represents the amount of time, in


hours, it takes someone to paint n rooms, then tin
represents the number of "hours per room" and
nit represents the number of "rooms per hour."
402 McGraw-Hill Education: SAT

Lesson 1 3: Sim pl ifying rational expressions

I f x = 3a a n d a ""' 2, which o f t h e following i s Substitute x = 3a:


3a + 6
x 2 - 36 3a - 6
equivalent t o ?
(x - 6) 2 Divide numerator and a+2
a+2 denominator by 3: a-2
A)
a-2
Therefore the answer is (A). Bonus question: Why did the
3a + 2
B) question have to mention that a ""' 2?
3a - 2
3a + 2 2x 2 - 18 a(x + b) for all
C) If x > 3, where a
3a
_

5x 2 - 1 0x - 1 5 - x + l
D)
9a 2 - 36 and b are constants, what is the value of ab?
9a 2 + 36 2
A)
5
3
Since rational expressions are j ust fractions B)
(although perhaps complicated ones), we sim­ 5
plify them exactly the same way we simplify
6
C)
any fraction, that is, by cancelling common fac­ 5
tors in the numerator and denominator (which 7
D)
is equivalent to dividing by 1), or multiplying 5
numerator and denominator by a convenient (Medium-hard) The expression on the left side of
factor (which is equivalent to multiplying by 1). the equation is obnoxious and in desperate need of
• Factoring and cancelling common factors: simplification:
2 x 2 - 18 2(x + 3)(x - 3) 2(x - 3)
2x2 - 1 8
x2 + 5x + 6 (x + 2)(x + 3) x+2
5x 2 - l Ox - 1 5
• 2(x + 3)(x - 3)

_!_ ( _!_ -.!.)


Multiplying by a common factor: Factor:
5(x + l)(x - 3)
+ _!_ + x 3x
x+3 2(x + 3)
�= 3 x Cancel common factor
(okay since x > 3): 5(x + l)
2 2 x 3x 6x
2-
Divide numerator and (x + 3)
5
denominator by 5:
(Medium) This question is asking us to translate an x+l
expression i n x i nto a n expression in a, which requires
making a substitution. However, it is a bit simpler if This last step, which may seem strange, 1s important
we don't substitute right away, but instead simplify the because it shows us how the two sides of the equation
given expression: "match up." I f this equation is to b e true for "all x > 3"
then a must equal 2/5 and b must equal 3. Therefore,
x 2 - 36 ab = (2/5)(3) = 6/5, and the correct answer is (C ) . Bonus
(x - 6) 2 question: Why did the question mention that x > 3?
Factor: (x + 6)(x - 6)
(x - 6)(x - 6)
Cancel common factor: x+ 6
x-6
CHAPTER 9 / THE SAT MATH: ADVANCED MATHEMATICS 403

Lesson 1 4: Solvi ng rationa l equations

1 1
I f x > 0 a n d -- - -- = 2, what is the value of x? The function f is defined by the equation f(x) = x2 -
x-l x+l 3x - 18 and the function h is defined by the equation
[ No calculator] (x)
h(x) = f For what value of x does h(x) = 6?
.
2x - 12
A) .J2
A) -6
B) J3 B) -3
C) JS
C) 0
D) J7
D) 9

When solving an equation that includes fractions (Hard) The first thing we should try to do is simplify the
or rational expressions, you may find it helpful to expression for h(x).
simplify the equation by multiplying both sides
(x)
by the "common denominator" (that is, the h(x) = f
common multiple of the denominators) . 2x - 12
x 2 - 3x - l8
x 1 Substitute f(x) = x2 - 3x - 18: h(x) =
+ 2 = 2x - 1 2
5 x
- -

5x x ( � + �) = 5x
Factor using Product-Sum
(x + 3)(x - 6)
Method: h(x) =
Multiply by 5x: x 2 2(x - 6)
x+3
5x2 5x Cancel common factor: h(x) =
Distribute: + = l OX 2
5 x
- -

x+3
x2 + 5 = lOx Solve for x if h(x) = 6: 6=
Simplify:
2
Notice that, in this case, the equation simplifies to Multiply by 2: 12 = x + 3
a quadratic, which is relatively easy to work with. Subtract 3: 9=x

Therefore, the correct answer is (D) .


(Hard) Let's apply this strategy to our equation:

1
1 - --
-- =2
x-l x+l
Multiply by
(x - l)(x + 1): (x - l)(x + l)
[-
l
-
l
x-l x+l
-J
= 2(x - l)(x + l)

(x - l)(x + 1) (x - l)(x + 1)
D istribute: = 2(x 2 - l)
x-l x+l
Simplify: (x + 1) - (x - 1) = 2x2 - 2
Simplify: 2 = 2x2 - 2
Add 2: 4 = 2x2
Divide by 2 : 2 = x2
Take the
square root: ±../2 = X
But since the equation states that x > 0, the correct
answer is (A) .
404 McGraw-Hill Education: SAT

Exerc ise Set S (No Ca lcu lator)

• •
If � - � = 2:'., what is the value of y? Which of the following is equivalent to _ 2_ _ x_
+
3 5 9 for all x greater than l? 1 - x x -1

I I I I I A)
x+2
x2 - 1
x+2
B)
x-l
• x-2
x 1 25 . C)
If + -- = - and x > 0, what 1s the value of x? x2 - 1
x + l x - l 24
--

x-2
I I I
D)
x-1

Ill
.. For how many distinct i nteger values of n
. n+S >2
?
If _l_ _ _l _ = � , what is the value of x2?
IS
n+2
--

x-2 x+2 5

I I
A) Zero B) One C) Two D) Three


1
If a = - x and a > 1, which of the following is
• 4
If 2 - � = - � , what is the value of z? equivalent to 4(x - 4) 2 ?
z 6 4x 2 - 64
a-4
I I
A)
a+4
a2 - 4
B)
a2 + 4
a-l
C)
Iii
--

a+l
g(x) . a2 - 1
Let g(x) = x2 - 9x + 18 and h(x) = , where a 1s D)
x-a a2 + l
--

a constant. I f h( 4 ) = 2- , what is the value of a?


12

I I I I
..
If _l l_ a for all values of x
=
2
_ _ _

2x - 2 2x + l 4x - 2x - b
greater than 1, what is the value of a + b?

I I I I
CHAPTER 9 I THE SAT MATH: ADVANCED MATHEMATICS 405

Exercise Set S (Ca lculator)

Ill
x 3 I f _c_ -'- c + 1 = �, what is the value of c2?
I f - - - = 2, what is the value of x2 - lOx?
5 x c-1 2c 2
c -1

I I I I I I I I I I
Ill
Ill 4x 2 + 1 a
For how many positive i nteger values of k If = 2x - 1 + for all values of x, what
2x + l 2x + l
-- --

. 1
IS - > 0.001? is the value of a?
lOk

I I I I I I I I I

IFI
(x) Which of the following is equivalent to � � for all
I f g (x) = x2 - 9x + 18 and h(x) = -g-, what is the b 2
-

x2 + 3 positive values of b?
value of h(9)?
b2 - 1
A)
2-b
I I I I B)

b3 - 2
C)
2b
Ill
1 1 . the value of 9x ? 2 - b3
If + -- = 9, what 1s D)
x+l x-1 x2 - 1 2b
-- --

I I I I
406 McGraw-Hill Education: SAT

• --- 1111
__!:_ __!:_ = 2
_
I f one proofreader takes n hours to edit 3 0 pages and
a b another takes m hours to edit 50 p ages, and together
they can edit x pages per hour, which of the follow­
1 1 ing equations must be true?
-+-= 8
a b
30 50
A) - + - = x
n m
Given the system above, what is the value of a + b?
30 50 1
1 B) -+-=-
A) n m x
15
n m
C) - + - = x
B) 30 50
8
n m 1
8 D) - + - = -
C) 30 50 x
15
8
D)
5
CHAPTER 9 I THE SAT MATH: ADVANCED MATHEMATICS 407

EXE RCISE SET 5 ANSWE R KEY

No Calculator 7. D
Since (1 - x) = - (x - 1):
1. or --- l:'_ 2 -2
6/5 1 .2
s 9 x x
3 -- + -- = -- + --
Multiply by 4S: lS - 9 = Sy 1-x x-1 x-1 x-1
(4S is the least common multiple of the denominators. ) x-2
Simplify: 6 = Sy x-1
D ivide by S: 6/S = y
n+S
x 1 2S 8. c -- > 2
2. 7 -- + - = ­ n+2
Multiply by 24(x + l)(x - 1): x + l x - 1 24
Recall from Chapter 7, Lesson 9, on solving inequalities,
24x(x - l)+ 24(x + 1) = 2S(x + l)(x - 1) that we need to consider two conditions. First, if n + 2 is
Distribute: 24x2 - 24x + 24x + 24 = 2Sx2 - 2S positive (that is, n > -2), we can multiply on both sides
Gather like terms: 0 = x2 - 49 without "flipping" the inequality: n + S > 2n + 4
Add 49: 49 = x2 Subtract n and 4: 1>n
Take square root: ±7 = X So n must be between -2 and l, and the i nteger values
Since x must be p ositive, x = 7. of -1 and 0 are both solutions. Next, we consider
the possibility n + 2 i s negative (that is, n < -2), and
1 8 therefore multiplying both sides by n + 2 requires
3 . 1 3/2 o r 6.5
x-2 x+2 s
Multiply by S(x - 2)(x + 2): "flipping" the inequality:
S(x + 2) - S(x - 2) = 8(x - 2)(x + 2) n + S < 2n + 4
Distribute: Sx + 10 - Sx + 10 = 8x2 - 3 2 Subtract n and 4: l<n
Subtract 20 and simplify: O = 8x2 - S2 But there are no numbers that are both less than -2 and
Add S2: S2 = 8x2 greater than 1, so this yields no new solutions.
Divide by 8: S2/8 = 13/2 = x2
Remember, the question asks for the value of x2, not x, so 4(x - 4) 2
9. c
don't worry about taking the square root. 4x 2 - 64
4(x - 4) 2
1 s Factor:
4(x - 4)(x + 4)
4. 6/ 1 7 or .353 2 - - = -­
z 6 x-4
Multiply by 6z: 12z - 6 = -sz Cancel common factors:
Add Sz and 6: 17z = 6 x+4
Divide by 17: z = 6/ 17 Substitute x = 4a:
4a - 4
4a + 4
4 a-1
s. 28 h(4) = g ( ) = _!_ Cancel common factor:
4-a 12 a+l
Use definition of g : 4 2 - 9(4) + 18
4-a 12
Simplify : -2 Calculator
4-a 12 x 3
4 - a = -24 10. 15 ---=2
Cross-multiply : s x
Add 24 and a: 28 = a Multiply by Sx: x2 - l S = lOx
Add lS, subtract lOx: x2 - lOx = lS
6. 5 Notice that you should not worry about solving for x!
2x - 2 2x + l
11. 1
(2x + l) - (2x - 2) 2 - > 0.00 1
Combine fractions: lOk
(2x - 2)(2x + l) Use common base: 10- k > 10 - 3
Simplify : 3 Exponential Law #10: -k > -3
4x 2 - 2x - 2
Multiply by - 1 : k<3
. 3 a
Smee must equa[ for all values Therefore, the two positive integer solutions are 1 and 2.
4x 2 - 2x - 2 4x 2 - 2x - b
of x, a = 3 and b = 2, so a + b = S.
408 McGraw-Hill Education: SAT

g(9) 1 b2
12. 3/ 1 4 or .2 1 4 h(9) = 16. D
92 + 3 b 2
Use definition o f g: h(9) =
92 - 9(9) + 18
84
Common denominator: -2 - -
b3
2b 2b
Simplify: h(9) = � = 2_ 2 - b3
84 14 Combine:
2b

1 1 17. 1 1
c ---=2
1 3 . 8 1 /2 or 40.5 -- + -- = 9 a b
x+l x-1
1 1
(x + l) + (x + l) -+-=8
Combine fractions: =9 a b
(x + l)(x - 1) 2
�=9 Add equations: - = 10
Simplify: a
x2 - 1 Multiply by a: 2 = lOa
9x 81 Divide by 10: 1 /5 = a
-b = - 6
Multiply by 9/2:
x2 -1 2 Subtract equations:
-2

c c+l 10 Multiply by -b: 2 = 6b


14. 5 1 /3 = b
c - 1 2c c2 - 1 Divide by 6:
Therefore, a + b = 1/5 + 1 /3 = 8/ 1 5 .
-- 2c --
c X -- 10
Convert to x: =
C - 1 c + l c2 - l 18. A The number of pages they c a n edit together
2c2 10 i n an hour must equal the sum of the number of pages
Multiply: t hey can edit separately. The number of pages the first
c2 - 1 c2 - 1 proofreader can edit per hour i s 30/n, and the num­
Multiply by c2 - 1 : 2c2 = 10 ber of pages the second proofreader can edit per hour
Divide b y 2 : c2 = 5 is 50/m . Since they can edit x pages per hour together,
30 50
1 5 . 2 Notice that the right-hand side of the equation - + - = x.
n m
is the "proper" form of the "improper" fraction on the NOTE: You can avoid the most common mistakes with
left, and that a is the remainder when the division of
this problem by paying attention to the units of each
the polynom ials is completed :
term. The units of two sides, as well as the unit of each
term i n a sum or difference, must "match." Notice that
2x - l
2x + l ) 4x2 + ox + l
the unit for all of the terms is pages/hour.

4x2 + 2x
- 2x + l
- 2x - l
2
TH E SAT MATH :
ADD ITIO NAL TO P I C S

1. U n d e rsta n d i n g Geo m etric Re l a ti o n s h i ps 41 1

2. U nd e rsta nd i ng Basic Tri g o n o m etry 437

3. U n d e rsta n d i n g C o m p l ex N u m be rs 443

409
41 0 McGraw-Hill Education: SAT

The SAT Math:


Additional Topics
What other special topics are included o n the SAT Math test?

About 10% (6 out of 58 points) of the SAT M ath questions are "Additional Topics" questions. These include
topics like
analyzing triangles using the Pythagorean Theorem
graphing circles and other figures i n the xy-plane
• analyzing areas, circumferences, chords, and sectors of circles
measuring angles and arcs in radians
working with area and volume and their formulas
using the theorems of congruence and similarity
working with basic t rigonometric relationships including cofunction identities
calculating with imaginary and complex numbers

Why are these topics important?

These topics from geometry, trigonometry, and advanced analysis are crucial to work i n engineering, physics,
architecture, and even design. Although they are not essential to every college m ajor, they do provide tools for
understanding and analyzing advanced concepts across the curriculum.

Sound intimidating? It's not.

Some of you h ave already spent some time i n math class studying these topics. If not, the three skills described
i n these 12 lessons will give you the knowledge and practice you need to master them.
CHAPTER 1 0 / THE SAT MATH: ADDITIONAL TOPICS 41 1

S ki l l 1 : U ndersta nding Geometric Relation s h i ps


Lesson 1: I n tersecti ng and pa ra l lel l i nes

D c

180 - X 0

A B E
I n the figure above, ABCD is a parallelogram, and Helpful Tip
point B lies on AE . I f x = 40, what is the value of y7 When dealing with p arallel lines, especially i n
A) 40 B) 50 C) 60 D) 70 complicated figures, w e c a n simplify t hings by
considering angles in pairs. The important pairs
(Medium) Since ABCD is a parallelogram, we can take for m one of four letters: F, Z, C , or U.
advantage of the Parallel Lines Theorem.

The I ntersecting Lines Theorem

When two lines cross, four angles are formed. The


vertical angles are congruent and adjacent angles
are supplementary (that is, they have a sum of 180°).

1 80 - X 0

1 80 - X 0

The P arallel Lines Theorem

When two parallel lines are crossed by a third


line, eight angles are formed. I f the third line i s
perpendicular to o n e of the parallel lines, then
it's perpendicular to the other and all eight
angles are right angles. Otherwise, all four acute First, let's mark up the diagram with what we know from
angles are congruent, all four obtuse angles are the Parallel Lines Theorem.
congruent, and any acute angle i s supplementary
to any obtuse angle. D c
Yo 1 80 - Yo

1 80 - y0
A B E

Since the pairs of opposite sides are parallel, the consec­


utive angles in the parallelogram must b e supplemen­
tary (that is, have a sum of 180°). Notice that these pairs of
412 McGraw-Hill Education: SAT

consecutive angles form "U"s or "C''s as mentioned i n the (Hard) Although our diagram includes parallel lines,
previous Helpful Tip. This implies that opposite angles it doesn't seem to show any of the parallel line " letter
are congruent in a parallelogram . pairs" that we discussed above, because no line directly
connects the parallel lines. We can fix this problem by
Since ABE is a straight ( 1 8 0 ° ) angle: drawing an extra line that's parallel to l and m through
y + x + x + x 180
= the vertex of the angle.
=
Substitute x 40 and simplify: y + 1 2 0 180
=

Subtract 1 20 : y = 60
Therefore, t h e correct answer is (C ) .

Now we h ave two "Z" pairs of angles (otherwise


m known as "alternate interior" pairs) that show that the
middle angle is actually the sum of two smaller angles
I n the figure above, lines l and m are parallel. What of 36° and 43°, and therefore, x = 3 6 + 43 = 79, and the
is the value of x? correct answer is (B).

A) 43 B) 79 C) 86 D) 101
CHAPTER 1 0 / THE SAT MATH: ADDITIONAL TOPICS 413

Lesson 2: Tria ngles

Angle S u m Theorem Exterior Angle Theorem


The sum of the measures of the a ngles in any tri­ If the side of a triangle is extended b eyond a ver­
angle is 180°. tex, it makes an exterior angle with the adj acent
side. The measure of this exterior a ngle is equal
to the sum of the two remote i nterior angles.

(a+b)0
We can prove this with the "draw an extra line"
trick. If we take any triangle, pick any of its ver­
tices, and draw a line through that vertex that is
p arallel to the opposite side, we get a picture like The Triangle Inequality
the one above. Since the line we've drawn is a
180° angle, and since the "Z" angle pairs must be The sum of any two sides of a triangle must always
=
congruent, we've proven that a + b + c 180. be greater than the third side.

Side-Angle Theorem

The largest a::igle in a triangle is always across a+b>c


from the largest side, and the smallest angle is
b+c>a
always across from the smallest side.
c+a>b

a
c
a<b<c

50°
b B

Isosceles Triangle Theorem

If two sides in a triangle are congruent, the two


angles across from those sides are also congru­
ent. Converseiy, i f two a ngles in a triangle are
congruent, the two sides across from them are XO Yo
c
also congruent. A D

I n the figure above, if A D = DB = DC, what is the


value of x + y?

A) 72
B) 90
C) 96
D) 108
414 McGraw-Hill Education: SAT

(Medium) Since angle ADB and angle BDC


are supple­
mentary and AD = DB = DC, we can take advantage of
the Isosceles Triangle Theorem to mark up the diagram.

The figure above shows three i ntersecting lines.


XO Y0 What is the value of c i n terms of a and b?
A D C A) 180 - a - b
B) 180 - a + b
Now let's look at triangle ABC. Since its interior angles
must have a sum of 180°, x + x + y + y = 180, and therefore, C) 90 + b - a
2x + 2y = 180 and x + y = 90. So the correct answer is (B).
D) a+b
Notice that this fact is independent of the measures of the
other two (108° and 72° ) angles. As long as AD = DB = DC, (Easy) First, we should notice that two of the angles are
this relationship will hold. We can see these angle relation­ "vertical" to two i nterior angles of the triangle, and the
ships if we notice that these three segments could all be other is a n exterior angle.
D.
radii of a circle centered at

D
Since the c0 angle is an exterior angle to the triangle, the
Exterior Angle Theorem tells us that c = a + b, so the cor­
rect answer is (D).
Alternately, we could just choose reasonable values
You may remember from studying geometry that any = =
for a and b, like a 50 and b 90, and then analyze
"inscribed" angle (an angle inside a circle with a vertex the diagram i n terms of these values. Thi s would i mply
on the circle) i ntercepts an arc on the circle that is twice that the interior angles of the triangle are 50°, 90°, and
its measure. Since angle ABC is an inscribed angle that 40°, and c0 would then b e the measure of the supplement
i ntercepts a 180° arc, it must have a measure of 90° and of 40°, which i s 140°. I f we then plug these values for a and
therefore, x + y 90.= b i nto all of the choices, the only one that yields 140 i s D.
CHAPTER 1 0 / TH E SAT MATH: ADDITIONAL TOPICS 41 5

Les son 3: The xr-plane

y
Finding Slopes. To find the slope of a line in the
A B
xy-plane from a ny two points on the line, use the
slope formula.

Note: Figure not drawn to scale.


I

I n the xy-plane above, points A and B lie on the 1 rise = y - y1


I 2
graph of the line y = 6 . I f OB has a slope of ..!. and I
2 - - - - - - - �
A B = 5, what is the slope of OA ? run = x - x1
--+------�--
2 x

I I I I I
0

rise y - y1
(Medium-hard) To analyze this diagram, we must recall slope = - = ---
2
run x - x1
the definition of slope from Chapter 7, Lesson 5. 2
y
Finding Areas. Remember that the area of a
A 5 B
figure is just the number of unit squares that fit
inside it. You don't always need to use a special
formula to find the area of a figure. Even for very
0 7 C 5 D complicated shapes, you can sometimes find the
1 2- area just by counting squares.
First, let's drop two perpendicular segments from A
y
and B to points C and D, respectively, on the x-axis. Since
A and B lie on the line y = 6 1 they are both 6 units from the I�
x-axis, and so A C = ED = 6. Then, since the slope of OB
is V21 BD/ OD = V2 1 and th erefo re , OD = 12. Since AB = 5,
CD = 5 als �and therefore, OC = g - 5 = 7. (Don't •""-
worry that OC looks shorter than CD in the diagram.
Remember, the figure is not drawn to scale!) This gives us
everything we need to find the slope of OA , which con­ Area = 1 4 . 5 square units
I I I I I I I I I I �
nects (0, 0) to (7, 6). By the slope formula from Chapter 7, � x
Lesson 5, slope = (6 - 0)/(7 - O) = 6/7 = 0.857. I I I I I I I I I I

Working in the Coordinate Plane

Finding S egment Midpoints. To find the coordi­


nates of a midpoint, just average the coordinates
of the endpoints.

--+------_. x
0
41 6 McGraw-Hill Education: SAT

y (Medium) To find the m idpoint of a segment, we just


+ need to take �e average of the endpoints. Point M, the

( -- 2-+ 4 , -
12 + 0 )
D ( l 4, 14) midpoint of AB, therefore has coordinates

- = (1, 6) ,
( -14 -+-14 , -
6 + 14 )
and point N, the m idpoint
2- 2
C ( l 4, 6) - has coordinates
of CD, - = (14, 1 0).
2 2
x By t h e Slope Formula, then, t h e slope of M N i s
0 B (4, 0) 10 - 6 4
-- = - = 0.307 or 0.308 .
14 - 1 13
I n the figur� bove, point M (not shown) is the
midpoint of AB and point N (not shown) is the mid­
point of CD . What i s the slope of MN ?

I I
CHAPTER 1 0 I THE SAT MATH: ADDITIONAL TOPICS 41 7

Lesson 4: The Pythagorean Theorem a nd


the Dista nce Formula

y
At(2, 18)
The Pythagorean Theorem

B(l5, 18) If a , b, and c represent the sides of a right triangle

l �
in which c i s the longest side (the hypotenuse),

A
D(-3, 6) C(24, 6) x
0
b

What is the perimeter of quadrilateral ABCD in the c a


B
figure above?

I I I I I Special Right Triangles

The SAT Math test expects you to be familiar with


(Medium) The perimeter of a figure is the distance a round fou r fam ilies of special right triangles: 45°-45°-
its edges. It's easy to find the lengths ofAB and DC 900 triangles, 30°-60°-90° triangles, 3-4-5 tri­
angles, and 5-12-13 triangles. Take some time
because they are horizontal. The length of a horizontal
segment is just the difference between the x-coordinates to familiarize yourself with these particular side­
AB 15 - 2 13,
of its endpoints. The length of is = and the side relationships and side-angle relationships
length ofDC 24 - (-3) 27.
is = To find the lengths of so that you can use these relationships when you
recognize these triangles in SAT Math questions .
AD and BC , we can drop two vertical lines from points
� nd to the bottom edge. This shows that
B AD and
BC are hypotenuses of two right triangles as shown in

the figure b elow. 4 C'
S '\/ 2
s x
y 4

t s x J3
A(2, 18) B(l5, 18)

� 5y
13 I

12 1 I

...__ _x
5_ 3x
D(-3, 6) C(24, 6) �
_ --
�--1---- x
0 4x 1 2y

(Take a minute to confirm the lengths of all the segments


(3x)2 + (4x)2 (5x)2 = (5y)2 + (12y)2 = (13y)2
for yourself.) With this information, we can find AD and
BC by the Pythagorean Theorem.
9x2 + 16x2 2 5x2 = 25y2 + 144y2 169y2
=
41 8 McGraw-Hill Education: SAT

So, according to our diagram: AD2 = 52 + 122 = 169 Take the square root: AD = 13
BC2 = 92 + 122 = 225 BC= 15
Notice that triangle on the left is a 5-12-13 special right Therefore, t h e perimeter o f ABCD i s 13 + 15 + 27 +
triangle, and the triangle on the right is a 3-4-5 special 13 = 68.
right triangle. Noticing these relationships provides a
shortcut to using the Pythagorean Theorem .

The Distance Formula

We can generalize the technique we used in the previous problem to find the distance b etween any two points
in the xy-plane. Just think of this distance as the length of the hypotenuse of a right triangle, as i n the figure
b elow. I n other words, the Pythagorean Theorem and the D istance Formula are one and the same.
y

d
lY2 - Yi l

_ _ _ _ _ _ J�
x
0

By the Pythagorean Theorem:

Take the square root:

The 3 - D Distance Formula

If we need to find the distance between two points in three- dimensional xyz-space, we just need to use a modi­
fied version of the distance formula that i ncludes the extra z-dimension. You can see where this formula comes
from if you imagine trying to find the length of the longest diagonal through a rectangular box.
A
I
h
I
B J.... � - - - - -
D c

The length of this diagonal, AC, is also the hypotenuse of right triangle ABC, and so its length is given by the
Pythagorean Theorem.
Pythagorean Theorem for ABC: AC = )(AB)2 + (BC)2
Pythagorean Theorem for BDC: (BC)2 = (BD)2 + (DC)2
Substitute: AC = )(AB)2 + (BD)2 + (DC)2
Since AB = h, BD = w, and DC = l
AC= )t2 + w2 + h2
CHAPTER 1 0 I THE SAT MATH: ADDITIONAL TOPICS 419

Exerc ise Set 1 : Geometry (No Calcu lator)

M Q N

p 0

In the figure above, MNOP is a square and Q is the


Lines l and m are parallel in the figure above.
midpoint of MN. If QO .J20 3= , what is the area of Which of the following expresses the value of in y

square MNOP? terms of x?


A) 95 - 2x

I I I I I B) 165 - 2x
C) 175 - 2x
II D) 185 - 2x

..
y

(3, 13) (12, 13)


(3, 5) (18, 5)
I I I I --t----- x
0
Lines l and m are parallel in the figure above. What
is the value of x?
In the figure above, what is the distance between the
• midpoints (not shown) of the two line segments?
A) J68 B) J73 C) .fi6
D) J78

Ill
240 °
What is the perimeter of an equilateral triangle
inscribed in a circle with circumference 2411?
A) 36.J2 B) 30J3
C) 3 5.fj D) 24)6

In the figure above, what is the value of a + b + c?


420 McGraw-Hill Education: SAT

Exerc ise Set 1 : Geometry (Ca lc u la tor)

Questions 7-9 a re based o n the figu re below. • ---

y
A(3, 15)
B(lO, 12) x
3m
Yo

C(lO, 2) 2m + 5
0
Note: Figure not drawn to scale.
Note: Figure not drawn to scale. In the figure above, what is the value of x?
A) 5../2 B) 5J3 C) 15../2 D) 15J3

In the figure above, what is the perimeter of quadri­
lateral ABCO, to the nearest integer?
Questions 1 2-1 5 a re based on the situation
desc ribed below.

In the xy-plane, ABCD is a square. Point A has coor­


dinates (-1, 2) and point B has coordinates (3, 5).
Ill
In the figure above, what is the area, in square units,
ofABCO? Which of the following could be the coordinates
of C?
I I I I I A) (0, 9) B) (6, O) C) (2, -2) D) (-4, 6)
.. Ill
In the figure above, point K (not shown) is the mid­ What is the area of square ABCD?
point of OA_.'.._and point M (not shown) is the
midpoint of AB. What is the slope of KM?
A) 25 B) 28 C) 30 D) 32

What is the slope of BC?
M
4 m 3 q 3
3 4 4 D) 34
1111
In the xy-plane, point H has coordinates (2, 1) and Ill
point J has coordinates (11, 13). If HK is parallel to Wh� is the distance between C and the midpoint
the x-axis and JK is parallel to the y-axis, what it the of AB?
perimeter of triangle HJK?
A) JSO B) ms C) J50
4 4 2
D) ms
2
CHAPTER 1 0 / THE SAT MATH: ADDITIONAL TOPICS 42 1

EXE RCISE SET 1 : GEOMETRY ANSWE R KEY

No Calculator 4. C Using the Crossed Lines Theorem and the


Parallel Lines Theorem, we can mark up the diagram
or or
1. 1 6/9 1 .7 7 1 . 78 If we define x as the like this:
length of QN , then the length of one side of the square is
2x, and so the area of square MNOP is (2x)(2x) = 4x2• To
find this value, we can apply the Pythagorean Theorem
to right triangle QNO:
x (2x)2
2
+ = [ �r m

Simplify: 5x = -209
2

Divide by 5: x 4520 49
2
= - This shows that x + y + x + 5 = 180, and soy = 175 - 2x.
4x2 = -169 = 1. 7 7 or 1. 7 8
( 3 � 12 ' 13 : 13 ) [ l; ' 13 } and the midpoint of
Multiply by 4: 5. B The midpoint of the top segment is
the bottom segment is ( 3 � 18 ' 5 : 5 ) = [ 221 ' 5 }
=

2. 1 33The key is to notice simple relationships


between angles until we get around to x.

[ 21 15 ) + c 13 - 5) = ff+B2 = m
therefore, the distance between them is
2
2 2 2

2-2

6. C To solve this problem we must draw a diagram


and find the relationship between the radius of the circle
and the sides of the triangle. By the Isosceles Triangle
Theorem, if all three sides of a triangle are congruent, then
3. 210 Draw three lines as shown: all three angles must be congruent. Since these angles
also must have a sum of 180°, they must each be 60°. Ifwe
draw the bisectors of each of these angles, we divide the
triangle into six smaller triangles. These smaller triangles
are congruent 30°-60°-90° triangles, as shown here:

Since the polygon divides into 3 triangles, the sum of its


internal angles is (3)(180°) = 540°. Therefore a + b + + c

240 + 90 = 540, and so a + b + 210.


c =

Since the circumference of the circle (2nr) is 2411, its


radius is 12. Since each of the hypotenuses of our right
triangles is also a radius of the circle, we can find all
of the sides of these triangles using the 30°-60°-90°
relationships. Each side of the equilateral triangle is
therefore 2 ( 6 ../3 ) = 12 ../3, and its perimeter is therefore
2(12 ../3 ) 36../3.
=
422 McGraw-Hill Education: SAT

Calculator have measure 15. Using the Pythagorean Theorem or the


7. 43 Using the distance formula, we can calcu­ 45°-45°-90° shortcut, we can see that x = 15..J2.
late the lengths of each segment. OA = -J234 ;::::; 15. 3 0, 12. A The key to questions 12 through 15 is a good
AB = J58 ;::::; 7. 6 1, BC = 10, and OC = Ji04 ;::::; 10. 20. diagram in the xy-plane that represents the given
Therefore, the perimeter is approximately 15. 3 0 + 7. 6 1 + information:
10 + 10. 2 0 = 43.11, which rounds to 43.
8. 1 07 Since we do not have a formula that directly
calculates the area of such an odd-shaped quadrilateral,
we must analyze its area in terms of simpler shapes.
The simplest way to do this is by drawing a box around
it. This turns the area of interest into a rectangle minus
three right triangles, all of which have areas that can be
easily calculated. , C,(6, l )
x

y
A (3, 15) (10, 15)
B(lO, 12)
I If ABCD is a square, then the points A, B, C, and D must
appear in that order around the square. Notice that to get
1 50 - 2 2 . 5 - 1 0 . 5 - 1 0 = 1 07 from point A to point B, we must move 4 units to the right
and 3 units up. This means that, in order to get to point
C along a perpendicular of the same length, we must go
I C(lO, 2) either 3 units right and 4 units down, or 3 units left and
4 units up. This puts us either at (6, 1) or (0, 9).
0
13. A The diagram shows that AB is the length of the
hypotenuse of a right triangle with legs 3 and 4. You
9. 6/5 1 .2 _lhe midpoint of OA is (1.5, 7.5) and
or should recognize this as the special 3-4-5 right triangle.
the midpoint of AB is (6. 5 , 13.5); therefore, the slope If AB = 5, then the area of the square is 52 = 25.
of the segment between them is 6/5.
10. 36 If point K is on the same horizontal line as (2, 1), 14. A Notice that the slope of BC is the same regard­
less of which option we choose for C. In either case, the
it must have a y-coordinate of 1, and if it is on the same slope formula tells us that the slope is -4/3.
vertical line as (11, 13), it must have an x-coordinate
of 11. Therefore, K is the point (11, 1), and so HK= 9, JK = 15. D The midpoint of AB ( point M above) is (1, 3.5).
12, and HJ = �92 + 12 2 = ..fii5 = 15. Notice that it is a We can use the distance formula to find the distance
3-4-5 triangle! between this point and either of the possible locations
of C. (Notice that the distance is the same either way.)
11. C Since the sum of the interior angles of any tri­ Alternately, we might notice that MC is the hypotenuse
angle is 180°, y + y + 2y = 4y = 180, and therefore y = of a right triangle with legs 5 and 2. 5 . Either way, we get a
45. Therefore, this is a 45°-45°-90° right triangle. Since value of ms
two angles are equal, the two opposite sides must also be
equal, so 3m = 2m + 5 and so m = 5 and the two legs each
--

2 .
CHAPTER 1 0 / THE SAT MATH: ADDITIONAL TOPICS 423

Lesson 5 : Circ les

Which of the following equations represents a circle What is the area, in square centimeters, of a circle
in the xy-plane that passes through the point (1, 5) with circumference of lfrrr centimeters?
and has a center of (3, 2)? A) 811
A) ( x - 3)2 + (y - 2)2 = .Ji3 B) 1611
B) (x - 3)2 + (y - 2)2 = 13 C)3211
C) (x - 1)2 + ( y - 5)2 = 13
D 6411
)
D) (x - 3)2 + (y - 2)2 = 25 ( Easy) If the circumference of the
circle is 1611 centimeters: 211r = 1611
Divide by 211: r= 8
Equations of Circles
Therefore, by the
area formula: Area = 11r2 = 11(8)2 = 6411
(x - h)2 + (y - k)2 = r2 So the correct answer is (D) .
A circle is defined as the set of all points in a plane
that are a fixed distance, r, from a fixed point,
(h, k). The distance r is called the radius and (h, k)
is the center.
Circumference of a Circle
y
circumference = 11d = 21\r

11 = circumference
diameter
0

Therefore, by the Distance Formula, any point


on
(x, y) the circle must
satisfy the equation The number 11 c� 3.14159 . . .) is defined as the
r = �( x h)2 + (y - k)2 number of diameters it takes to get around a
11
circle. Put another way, is the ratio of the cir­
-

cumference of any circle to its diameter. Since any


or (x h)2 + (y - k)2 = r2
- diameter is twice the radius,
11 = circumference circumference
diameter = 2r and so
circumference = 211r
(Easy) Since our circle has a center at (3, 2), its equa­
tion must have the form (x - 3)2 + (y - 2)2 = r2, which Area of a Circle
eliminates choice (C). To find r, the radius, we sim­
ply have to find the distance between the center and A = 1'r2
any point on the circle. By the distance formula, this Ifwe cut any circle into tiny enough sectors, and
is �(3 - 1)2 + (2 - 5)2 = )4 + 9 = .Ji3, and therefore, reassemble them as shown below, we can create
r2 = ( .Ji3 )2 = 13. The correct answer is (B) . If you chose a parallelogram-like shape that has a height of
r and a length that is half of the circumference,
(A) , keep in mind that the equation for a circle has r2 on or ,.,,
the right side, not r.
424 McGraw-Hill Education: SAT

(Hard) The first thing we should do is draw radius OP.


Since this is a radius to the point of tangency, it is perpen­
dicular to the tangent. We should also write in the given
measures.
The area of the circle is 10011: 11(OP)2 = 10011
Divide by 71 ; (OP)2 = 100
Since the area of any parallelogram is equal to its base Take square root: OP = 10
times its height, the area of a circle is ('11r)(r) = 7tr. M
Tangents to Circles

A tangent line to a curve is a line that touches the Q


curve without crossing it. A tangent line to any
circle is perpendicular to the radius at the point
of tangency.

Notice t h at OP th igh of triangle MOQ if MQ


is e he t
is taken as its base. Ifwe can find the length of base MQ,
bh .
we can simply use the triangle angle formula A = 2
To find MQ, we can use the Pythagorean Theorem to find
MP and PQ and just add them together.
102 + (MP)2 = ( J269 )2
Pythagorean Theorem for
triangle OPM:
M
102 + (PQ)2 = ( JZ44 )2
Pythagorean Theorem for
triangle OPQ:
Simplify: 100 + (MP)2 = 269
Q 100 + (PQ)2 = 244
Subtract 100: (MP)2 = 169
(PQ)2 = 144
Take square root: MP = 13
In the figure above, MQ is tangent to the circle at
PQ = 12
point P, MO= J269, and OQ = JZ«. If the circle Therefore MQ =
has an area of 10011, what is the area of triangle MOQ? MP + PQ = bh
13 + 12 = 25, so: Area of MOQ=-2 = 25 2x l0 = 125
--

I I
CHAPTER 1 0 / THE SAT MATH: ADDITIONAL TOPICS 425

Lesson 6: Radians, chords, a rcs, and sectors

What is the degree measure of an angle that mea­


sures 4. 5 radians? 90° = 90° 7r radians
x = 7r radians
- .
A) 4.5'rr 0 B) 40 C) 810° D) 9 180° 2
7r 180° = 180°
11 radians = 11 11 radians
x

(Medium) Although many students will get this question . = 37r 180° = 270°
37r radians
wrong, it is very simple if you know how to convert radi­ - x
ans to degrees. All we need to do to convert any radian 2 -

2 7r radians
measure to a degree measure is to multiply it by the
conversion factor 180° (as explained below). 180° = 360°
211 radians = 211 11 radians
x
7r radians 180° 810°
Therefore, 4. 5 radians = 4. 5 radians radians
x ----
7r 7r
and the correct answer is (C).
Take some time to memorize the radian mea­
The Radian sures of the common degree measures above.
A radian is simply the radius of a circle used as a Put them on flashcards and study them until
"measuring stick" for an arc on the circle and for you've mastered them.
its corresponding central angle.

B
1 radius

The circle above has an area of 10011 square centi­


Because circumference = 211r, a full rotation meters. If chord A� is 8 centimeters long, how far,
of 360° equals 211 radians, and 180° equals in centimeters, is AB from the center of the circle?
11 radians. A) 6 B) 8 C) J72 D) J84
Therefore, we may use 7r radians
1800 as a conversion
factor to convert a degree measure to radians, (Medium) First, let's draw three extra line segments:
180° as a conversion factor to convert
and 7r radians
a radian measure to degrees.
30° = 30° 7r radians
x = 7r radians
- .
180° 6
45 = 45° 7r radians
o

x
7r .
180° = -radians
4 Since nr2 = 10011, r = 10. If we draw a perpendicular from
the center to the chord, the length of this segment is the
7r ' distance from the center to the chord. This segment also
600 = 600 7r radians
x
180° = -3 ra d1ans bisects the chord, dividing it into two equal segments of 4
centimeters each. This allows us to use the Pythagorean
Theorem to find this distance: 42 + x2 = 102
426 McGraw-Hill Education: SAT

Simplify: 16 + x2 = 100 (Medium) Since the circle has a radius of 12, its circum­
Subtract 16: x2 = 84 ference is 211(12) = 24n. Since AC is a diameter, then the
Take square root: x=../84 measure of arc AC is half the circumference, or 12n. If
the length of arc AB is 711, then the length0 of arc BC is
Therefore, the correct answer is (D) . 12n - 711 = 511. Since the central angle of x is the same
fraction of 360° as its arc BC is to the x 571"
entire circumference, 360 2471"
Chords Cross multiply: 2411x = l,800n
A chord is any line segment that connects two Divide by 241t: X = 75
points on a circle. The longest chord in a circle is
its diameter, which passes through the center. Therefore, the correct answer is (C) .
The perpendicular segment from the center of Arcs and Sectors
the circle to a chord always bisects that chord.
An arc is a continuous part of the circumference of a
circle. Every arc has a corresponding central angle. The
ratio of an arc length to the circumference is equal to
the ratio of its central angle to 360° (or, in radians, 21i) .

B
mAB x
,.---.._

B 27rr 360
A sector is a part of the interior of a circle bounded by an
arc and two radii. The ratio of a sector area to the area
c
of the circle is equal to the ratio of its central angle to
360° (or, in radians, 21i).
Note: Figure not drawn to scale.
In the figure above, AC is a diameter of the circle
with center 0, OB = 12, and the length of arc AB is
7n. What is the value of x?
A) 60
B)72
C 75)
D 78) area of sector x
7rr 2
360
CHAPTER 1 0 I THE SAT MATH: ADDITIONAL TOPICS 427

(Medium-Hard) Since OA, OB, and OC are all radii,


triangles AOB and BOC are isosceles. Therefore, we
can analyze the diagram with the Isosceles Triangle
Theorem and the Angle Sum Theorem:

Note: Figure not drawn to scale.


In the figure above, AC is a diameter of the circle
with center 0, OB = 7, and the measure ofACE is 20°.
What is the area of the shaded sector?
A) l47r
12 B) l47r9 C) 497r
12
Since the central angle of the sector is 40°, the area of the
sector is 40°/360° = 1/9 the area of the circle. Since the
D) 497r9 area of the circle is 11(7)2 = 4911, the area of the sector is
4911/9 square units. Therefore, the correct answer is (D).
428 McGraw-Hill Education: SAT

Lesson 7: Areas and vol u mes

Reference Information

Every SAT Math section will include the following Reference Information. Take some time to familiarize
yourself with these area and volume formulas.

2� x
G Dw � b� b a
b'

A = nr' A = lw c' = a' +
Special Right Triangles
C = 2nr

w ;t
v �
4
V = lwh V= nr'h V = - nr3 V= � nr2h V = � lwh
3 3 3

The number of degrees of arc in a circle is 360.


The number of radians of arc in a circle is 211.
The sum of the measures in degrees of the angles of a triangle is 180.

(Medium) To find the mass of the tube, we must multiply


its density by its volume. To find its volume, we must sub­
tract the volume of cylindrical hole from the volume of
the wooden cylinder. The large cylinder has a radius of 2
(remember, the diameter is 4 and so the radius is 4 -:- 2 =
2) and a length of 10, so its volume is 11(2)2(10) = 4011. The
cylindrical hole has a radius of 1 (because its diameter
is 2) and a length of (0. 4 0)(10) 4, so the volume of the
=

The figure above shows a wooden cylindrical tube hole is 11(1)2(4) 411. Therefore the total volume of
=

with a length of 10 centimeters and a diameter of the closed tube is 4011 - 411 3611 113.1. Since the
= �

4 centimeters with a cylindrical hole with a diam­ mass is equal to the volume times the density, its mass
is (113.1)(4. 2) = 475 grams, so the correct answer is (D).
eter of 2 centimeters that extends 40% of the length
of the tube. The tube is closed on the end opposite
to the hole. The density of the wood is 4.2 grams per
cubic centimeter. What is the mass of this tube, to the
nearest gram? (Recall that mass = density volume)
x

A) 151 grams B) 343 grams


C) 468 grams D) 475 grams
CHAPTER 1 0 / THE SAT MATH: ADDITIONAL TOPICS 429

y In this case, we can look at this area in two different

t
ways: as a rectangle plus two right triangles, or as a bigger
rectangle minus two right triangles:
A (2, 1 8) B ( l5, 1 8)

l \
y
t
A (2, 18) B ( l 5, 1 8)
D ( -3, 6) C (24, 6) I 13 I
x 1 12
0 I
1 2 I1
5
D ( - 3,
--+------ x
6) C (24, 6)
What is the area, i n square units, of quadrilateral 0
ABCD above?

I I I I I y
(Medium) You m ight remember this figure from Lesson A (2, 1 8) B ( l 5, 1 8)
---
4, in which we found its perimeter. Now we are asked to 13 9 [j
I
find its area. Unfortunately, we are not given any formula 1 12
for calculating the area of this kind of quadrilateral. (You I

might remember that its technical name is a trapezoid,


but i n fact we don't need to know a nything special about D ( -3,
--+----- x
6) 27 C (24, 6)
trapezoids to solve this problem.) In such situations, it 0
helps to remember the Strange Area Rule.

We should get the same result from either method.


Strange Area Rule
With the first method, the area of the trapezoid is the
When asked to find a "strange area," that is, the area of the rectangle plus the areas of two right trian­
area of a region for which you do not have a sim­ gles. This gives us a total area of ( 1 2)(13) + (1 /2)(5) ( 1 2) +
ple formula, try to analyze the region into the (1/2)(9)(12) = 156 + 30 + 54 240. With the second
=

sum or the difference of simpler shapes. method, the area of the trapezoid is the area of the large
rectangle minus the areas of the two right triangles. This
gives us a total area of (l2)(27) - (1 /2)(5)(12) - (1 /2)(9)(12) =

324 - 30 - 54 240. Bingo!


=
430 McGraw-Hill Education: SAT

Lesson 8: Similar figu res

D (Medium) The key to this question is noticing that the


two triangles are similar. That is, that they are the same
B
shape, although they may be different sizes.
Similarity

In geometry, similar really just means the same


shape. 1\vo figures are similar if and only if all
A pairs of corresponding angles are congruent, and
all pairs of corresponding sides are proportional.
c
Helpful Tip

Always be on the lookout for similar triangles in


In the figure above, AB and CD are line segments SAT M ath diagrams!
that intersect at point P. What is the value of m?

AA (Angle-Angle) Similarity Theorem

If two triangles have two pairs of congruent corresponding angles, then (1) the remaining pair of corresponding
angles must be congruent, and (2) the triangles must be similar.
B
E

�-
" ABC "' " DEF

LACB � L.DFE
0

D AB
-
AC BC
A F DE DF EF

SAS (Side-Angle- Side) Similarity Theorem

If two triangles have two pairs of proportional corresponding sides, and the corresponding angles between
those sides are also congruent, then the triangles must be similar.
B


E
" ABC "' " DEF
AB AC LACB � L.DFE
DE DF
L.ABC � L.DEF
D -
AB AC BC
A F
DE DF EF
c

Perimeters, Areas, and Volumes of Similar Figures

If two polygons are similar with corresponding sides in ratio of a:b, then the corresponding perimeters of
those figures have a ratio of a:b and their corresponding areas have a ratio of a2:b2. If two solids have corre­
sponding lengths in a ratio of a:b, then their volumes have a ratio of a3:b3.
CHAPTER 1 0 I THE SAT MATH: ADDITIONAL TOPICS 431

Coming back to our diagram, if AB and CD i nter­ 8 7


sect at point P, the Crossed Lines Theorem tells us that 10 m
LAPC and LBPD must b e congruent, and so, by the AA Cross multiply: 8m = 70
Theorem, MPC /::,BPD. Therefore, the corresponding

Divide by 8: m = 70/8 = 8.75
sides are proportional:
432 McGraw-Hill Education: SAT

Exerc ise Set 2: Geometry (No C a l c u l ator)

• Iii
A cereal company sells oatmeal in two sizes of cylin­ Which of the following equations represents a circle
drical containers. The radius of the larger container in the xy-plane that intersects the x-axis at (3, 0)
and
is twice that of the smaller, and the height of the (9,O) ?
larger container is 50%
greater than the smaller. If
A) (x - 6)2 + (y - 4)2 = 25
the smaller container holds 1 0 ounces of oatmeal,
how many ounces can the larger container hold? B) (x - 3)2 + (y - 9)2 = 25
C) (x - 6)2 + (y - 4)2 = 36
I I I I I D) (x - 3)2 + (y - 9)2 = 36
• --

• �---

p N

In the figure above, P and N are the centers of the


Note: Figure not drawn to scale. circles and PN = What is the area of the shaded
6.
region?
I n the figure above, CD is tangent to both circles,
which are tangent to each other. If the smaller circle A) 1811 - 9.f3 B) 2411 - 9.f3
has a circumference of and the larger cir � has
411
a circumference of 1611,
what is the length of CD? C) 2411 - rn.J3 D) 3611 - 18.J3

I I y

..
What is the area, in square inches, of a circle with
diameter6112 inches?
A) 9114 B) 9715

• 0 (3, 0) (9, 0)
What is the length of the longest line segment that
connects two vertices of a rectangular box that is The diagram above shows a hexagon with all sides
6 4 2
units wide, units long, and units tall? congruent and all angles congruent. What is the
value of k?
A) .Jl2 B) J48 C) J56
C) 12J2 D) 12.J3
D) .J58
CHAPTER 1 0 I THE SAT MATH: ADDITIONAL TOPICS 433

Exerc ise Set 2 : Geometry (Ca lcu lator)

Ill Ill
The Great Pyramid i n Giza, Egypt, has a height of
140 meters and a volume of 2 . 6 million cubic meters.
D ( l 4, 1 4) If a scale model of the Great Pyramid i s to b e built
that is 2 meters h igh, what will b e the volume, in
cubic meters, of this model?

I I I I I
C ( l 4, 6)

0 B (4, 0)

Which of the following equations defines a circle


What is the area, in square units, of the quadrilat­ that is tangent to the y-axis?
eral above?
A) (x - )2 3)2 2
2 + (y + =

I I I I I B) (x - 2 )2 + (y + 3)2 3
C) (x 2)2 + (y + 3)2 4
-
=

D) (x - 2)2 + (y + 3)2 9
=


What is the degree measure, to the nearest whole
degree, of an angle that measures 5.6 radians? Questions 1 3 and 1 4 refer to the diagram below.

I I I I I E -------�

9
F

B
1111 i-------.r C
15

A D G

0
N
The figure above shows two rectangles that share
a common vertex, and AF i s a line segment that
passes through C.
M

,.---..__
Ill
In the figure above, arc MN has a length of 1 1 . 5 . To What is the ratio of the area of rectangle ABCD to the
the nearest integer, what i s the value of x? area of rectangle AEFG?

I I I I I
A) 3:5
B) 9:25
C) 5:8
D) 25:64
434 McGraw-Hill Education: SAT

ID
If CD = 9, what is the perimeter of rectangle AEFG?
A) 67. 2
B) 72.6
C) 76.2
D) 78.6

Point 0 is the center of the circle above. What is the


area of the shaded region?

A) 647r J3
-

3 - 16 3
B) l 67r
- J3
3 -8 3
C) 647r - J3
-

3 12 3
D) 647r
-
J3
-8 3
3
CHAPTER 1 0 I THE SAT MATH: ADDITIONAL TOPICS 435

EXE RCISE SET 2: GEOMETRY ANSWE R KEY

No Calculator 5. A All of the equations are clearly equations of


circles, so our only task is to verify that one of these
1.
60 If the smaller cylinder has a radius of and r equations is satisfied by both point (3, 0) and point (9, 0).
a height of h,
its volume is 11r2h.
The larger cylinder Simply by plugging these coordinates into the equations,
2r
therefore must have a radius of and a height of I. S h, we can verify that only the equation in (A) is true for both
and a volume of 11(2r)2(1.Sh) = 611r2h.
Since this is 6 times points: (3- 6)2 -4)2 = 25
+ (0 and (9 - 6)2 - 4)2 = 2 5.
+ (0
the volume of the smaller cylinder, it must hold 10 x 6 =
60 ounces of oatmeal.
6. C I n this problem, we have to take advantage of
the Strange Area Rule from Lesson 7. First we should
2.
8 First, let's d raw the radii to the points of tan­ draw the segments from P and N to the points of
gency, the segment joining the centers, and the segment intersection. Since each of these segments is a radius,
from the center of the smaller circle that is perpendicu­ they have equal measure (6), and form two equilateral
lar to the radius of the larger circle. Since the tangent 60°-60°-60° triangles.
segment i s perpendicular to the radii, these segments
form a rectangle and a right triangle.

The shaded region is composed of two circle "segments,"


each of which is a sector minus a triangle, as shown
in the figure above. The sector, since it has a 1 20° cen­
1/3 (1/3)
( J3 ) = J3
tral angle, has an area of the whole circle, or
(11(6)2) = 1211and the triangle has area 3 3 9 .
Since the circumference of the smaller circle is its 411, (2) ( 12n -9J3 ) = 24n -18J3.
Therefore, the shaded region h a s a n area of
radius is 2,
and since the circumference of the larger
circle is 1611, 8.
its radius is The hypotenuse of the right
=
triangle is the sum of the two radii: + 2 8=
10. One of 7. B Each side of the hexagon has length 9 - 3 6.
Each interior angle of a regular hexagon has measure
the legs of the right triangle is the difference of the two
radii: 8 2 = 6 .
-
(6- 2)(180°)/6 = 1 20°, so the segments shown form two
Pythagorean Theorem: + x2 52 = 102 30°-60°-90° triangles with lengths shown below.
Simplify: + 36 100 x2 =
Subtract 36: x2 = 64 y

Take square root: X= 8 ___ P(9, k)


3. B Diameter = 2r: 2r= 6�
Divide by 2: r=�
Area formula: 11r2 = 11(3�)2
Simplify: 11r2 = 11(91\4) 0 (9, 0)
Simplify: 11r2 = 9115 (3, 0)

4. From the 3 - D Distance Formula back in


C Therefore, k = 3 J3 + 3J3 = 6J3.
4,
Lesson the length of the diagonal is 6 2 + 2 + 2 � 4 2=
�36 + 16 + 4 = .J56
436 McGraw-Hill Education: SAT

Calculator 14. A I f CD = 9, we can find AD by the Pythagorean


Theorem. (AD)2 + (CD)2 = (AC)2
8. 1 42 First, let's draw a rectangle around the figure
as shown. Substitute: (AD)2 + 92 = 1 52
Simplify: (AD) 2 + 8 1 = 225
y Subtract 8 1 : (AD) 2 = 144
t 16 Take square root: AD = 1 2
D( l 4, 1 4 ) This means that the perimeter o f ABCD i s 1 2 + 9 + 1 2 +
8 9 = 42. Since the ratio of the perimeters of similar
42 5
figures equals the ratio of corresponding sides,
C( l 4, 6) p 8
Cross multiply: Sp = 336
Divide by 5: p = 67. 2
6 B(4, 0 ) 10
1 5 . A The two radii and the chord form an isosceles
This shows that the area we want is the area of the rect­ triangle. x + x + 4x = 180
angle minus the areas of the three triangles: (16)(14) - Simplify: 6x = 180
(1/2)(2)(16) - (1 /2)(1 2)(6) - (1 /2)(10)(6) = 224 - 1 6 - 36 - Divide by 6: x = 30
30 = 142. Therefore, the diagram should look like this:

9. 3 2 1 To convert any angle from radians


to degrees, we just multiply by the conversion factor
(180°)/(11 radians). 5.6 x 180°/11 = 320.86 � 321°.

10. 6 6 I n a ci rcle with radius 10, and arc of length


1 1 . 5 has a radian measure of 1 1 .5/10 = 1 . 1 5 radians.
In degrees, this equals 1 . 1 5 x 180°/11 = 65.89° � 66°.

1 1 . 7. 5 8 I f two similar solids have sides in ratio of


a:b, then their volumes are i n a ratio of a3:b3. The ratio
of the heights is 140:2 = 70: 1 , so the ratio of volumes is As we saw i n question 6, this portion of the circle is
703 : 1 3 = 343,000 : 1 . This means that the volume of the called a "segment," and we find its area by taking the
model is 2 , 600,000 -:- 343,000 � 7. 58 cubic meters. area of the sector minus the area of the triangle. The sec­
tor has area (1 20/360)(1182) = 6411/3, and the triangle has
12. C As a quick sketch will verify, i n order for a ( )
area ( l / 2) s J3 C4) = 1 6 J3 , so the segment has an area
circle to b e tangent to the y-axis, its radius must equal
the absolute value of the x-coordinate of its center. of 64n/3 - 1 6 J3 .
Since the center of each square is (2, -3), the radius must
be 2. The only circle with a radius of 2 is (C).

1 3 . D By the AA Theorem, triangle ACD is similar


to triangle AFG, and so rectangle ABCD is similar to
rectangle A EFG. The ratio of the corresponding sides is
equal to the ratio of their diagonals, which is 15:24 = 5:8.
Therefore, the ratio of their areas is 52:82 = 25:64
CHAPTER 1 0 I THE SAT MATH: ADDITIONAL TOPICS 437

Ski l l 2: U ndersta nding Basic Trigonometry


Lesson 9: The basic trigonometric functions

Which of these is equivalent to cos �4 - sin �6 ? (No The Unit Circle


calculator) The unit circle is just a circle with radius 1 centered
at the origin of the xy-plane. When using the unit
A) 1 -2.J3 circle, all angles must be in standard position,
that is, with vertex at the origin and measured

B) .fi -2 .J3 counterclockwise from the positive x-axis (just


like the angle e in the following figure).
C) .fi2- 1 y
(0, 1 )
D)
.J3 - .fi (x1 , y) = (cos 8, sin B)
2
---+----f-....... -+--- x
( 1 , 0)

The Basic Trigonometric Functions


(0, - 1 )
Any of the three basic trigonometric functions, like
all functions, takes an input number and transforms
it into an outpute number. A trigonometric function When an angle, 8, is in standard position, its
takes an angle,e , as an input and constructs a right terminal ray intersects the unit circle in the point
triangle with as one of its acute angles. The out­ (x11 y ). If we drop a vertical line segment from this
put is then the ratio of two sides of that triangle as pointi to the x-axis, we form a right triangle with
defined by the mnemonic SOH-CAH-TOA. legs oflength x1 and y1 and a hypotenuse oflength
1 (as shown above).
So now let's go back to the definitions of the
opposite basic trigonometric functions. In terms of this
right triangle, what are the sine, cosine, and
tangent of 8?
adjacent opposite = Y1 = Yi
sin e = hypotenuse 1
opposite
. e = -�-- e adjacent adjacent
S i ll
hypotenuse cos = -�-­ hypotenuse cos e = -�---
hypotenuse .5_1 = xi
tan e = opposite
---
adjacent opposite = 1J_
tan e = adjacent Xi

But these definitions are limited, because they This suggests three important theorems:
only work when e is an acute angle. What if it's
a larger angle, like 135°, or even a negative angle, • The sine of any angle is the y-coordinate of its
like -20°? To find the trigonometric ratios for corresponding point on the unit circle.
these angles, we use the unit circle. • The cosine of any angle is the x-coordinate of
its corresponding point on the unit circle.
• The tangent of any angle is the ratio of the
y-coordinate to the x-coordinate of its cor­
responding point on the unit circle.
438 McGraw-Hill Education: SAT

(Medium-hard ) First, you may find it useful to convert this situation on the unit circle so we c a n visualize it.
the angles to degree measures using the conversion fac­ (We don't want to confuse the angles called x and y in the
tor (180°/TI radians). This gives us TI/4 radians = 45° and problem with the x- coordinates and y-coordinates in the
TI/6 radians = 30°. We should recognize these as a ngles xy-plane. For this reason, let's label the terminal rays for
in two Special Right Triangles: the angles "angle x" and "angle z. 11 ) Recall that the sine
of any angle is the y-coordinate of the point on the unit
circle that corresponds to that angle. If sin x = a, then
a is the y-coordinate of the point on the unit circle that
45° s fi. 2x corresponds to "angle x, 11 as shown in the diagra m . If
s x
sin z = -a, then -a is the y-coordinate of the point on the
unit circle that corresponds to "angle z. 11 There are two
45° possible locations for "angle z" as shown in the diagram.
s
anglex y
Using the definitions for sine and cosine above, these Quadrant
s 1 .Ji
triangles show us that cos 45° = r;:: = r;:: =
II
- and
s...; 2 v 2 2
x 1 7T . 7T .Ji 1 ----+---T-->---+--� x
= -. Therefore, cos - - sm - =

sm 30 = 0
- - --= ( 1 , 0)
2x 2 4 6 2 2
.Ji - 1
--- and the correct answer is (C). Quadrant
IV
2
If sin x = a and sin z = -a, where x and z are in radi­ angle z? angl e z?
ans, and .'..1:. < x < 7T , which of the following could be
2
the value of z in terms of x? At this point, it may be easiest to simply pick a value for
"angle x" that is between TI/2 (::: 1 . 57) and TI (::: 3 . 14), such
A) TI - X as x = 2. Since sin 2 ::: 0.909 (remember to put your calcu­
B) X - TI lator into "radian mode"), a = 0.909. Now we just need to
find which angle among the choices has a sine of 0.909 -

C) 2TI + x
7T A) sin(TI - 2) = 0.909
D) x--
2 B) sin(2 - TI) = -0.909

(Hard ) The statement .'..1:. < x < 7T indicates that x is


C) sin(2TI + 2) = 0.909
2 D) sin(2 - TI/2) = 0.4 1 6
an angle in quadrant II, where the sine (the y-value
of the points on the unit circle) is positive. Let's draw Therefore, t h e correct answer is (B).
CHAPTER 1 0 / THE SAT MATH: ADDITIONAL TOPICS 439

The Pythagorean Identity

sin 2 x + cos 2 x 1 for all values of x


=

An identity is an algebraic equation that is true for all values of the unknown, and not just for particular values .
W e can prove t h e Pythagorean Identity b y j u s t applying t h e Pythagorean Theorem to o u r right tria ngle and
thinking about the t rigonometric ratios.

hypotenuse
opposite

(J

adj acent

Apply Pythagorean Theorem: (opposite)2 + (adjacent)2 = (hypotenuse)2

D ivide by (hypotenuse)2:
( opposite
hypotenuse
+
)2 (
adjacent
hypotenuse
)2 (
=
hypotenuse
hypotenuse
)2
Simplify using trig definitions: sin2 (J + cos2 (J = 1

1 Now we'll have to find the value of sin b cos b, which


I f b is a n angle measure such that sin b = -- ,
3 cos b we can find with the given equation.
what i s the value of (si n b - cos b)2?
. 1
1 Given equation: sm b = --

A) - 3 cos b
3
Multiply by cos b: sin b cos b = _!
B) J23 Substitute sin b cos b = _!
3

3
1 1
C) J3 - 1 . . . .
mto ongma l express10n: 1 -
.
2 sm b cos b = 1 - 2 x - = -

3 3 3

D) J3 + J2 So the correct answer is (A).


3
(Medium) The expression we are trying to evaluate
includes squared t rigonometric ratios, so we will prob­
ably h ave to take advantage of the Pythagorean Identity.
Expression to evaluate: (sin b - cos b)2
FOIL: sin2 b - 2sin b cos b + cos2 b
Rearrange using Commu-
tative Law of Addition: sin2 b + cos2 b - 2sin b cos b
Substitute sin2 b + cos2 b 1 :
= 1 - 2sin b cos b
440 McGraw-Hill Education: SAT

Lesson 1 0: The trigonometry of com plementa ry a ngles

(Medium-hard) Let's start by d rawing a picture of this


� and 0 < y < '.'.:__ , which of the following situation. Since y is the measure of an acute angle, we can
I f sin y =

[� - y) ?
b 2 imagine it as the interior angle of a right triangle. Since
its sine is equal to alb, we can say that the opposite side
is equal to sin has measure a and the hypotenuse has measure b.

A) Ja 2 - b 2
a b

B) Jb2 a- a 2 a

y
C) Ja 2 - b2
b

D) Jb2 - a 2
Now we can find the length of the remaining leg
(let's call it k) in terms of a and b using the Pythagorea n
b Theorem: k2 + a2 = b2
Subtract a2: k2 = b 2 - a2

Trigonometry of Complementary Angles Take the square root: k = b2 - a 2 J


Also, we know that the other acute angle has a mea­
sure of .:'."_ _ Y ' so let's complete the picture:
2

b 7r
The two acute angles in a right triangle are com­ - -y
plements of one another, that is, they have a sum 2 a
of 90° (or, i n radians, .:'.".) So, if one of the angles
.

2
has a radian measure of x, the other has a
measure of .:'."_ _ x .
2
I f we look at the trigonometric ratios for this new
angle, we see that these ratios are related to the Now, finding the value of sin ( � - y ) is just a matter
trigonometric ratios of its complement by the
of using the definition of sine: SOH.

[ )
following rule: The trigonometric ratio of any
angle equals the co/unction of its complement. Jr
sin 2 - y =
�b 2 _ a 2
b

c
7r
So the correct answer i s (D).
- - x
2 b

.
S ill
(2 )
Jr
-X = -;;- = CO S X
a

COS
(2 )
7r
- X =
b
-;;- = Sill
.
X
CHAPTER 1 0 I THE SAT MATH: ADDITIONAL TOPICS 441

Exerc ise Set 3 : Trigonometry (No C a l c u lator)

• •
What is the greatest possible value of f if If sin x = !!__b and 0 < x < .!.!:.,2 which of the following
f(x) = 8 sin2 2x _!_2 ?
_

expressions is equal to !?.. ?


a

I I I I I A) sin (�)
..
cos ( � - x l
1

If cos ( � ) = what is the value of ( � r ?


B)
a,

C) 1 - sin2x
I I I I I D) sin ( � - x )

n
If (sin x - cos x)2 = 0.83, what is the value of •
(sin x + cos x)2? If sin b which of the following could be the value
= a,

of cos (b + 11)?
I I I I I A) �
B) a2 -1

sm. ( -7r) C) -

cos ( � l
Which o f the following i s equivalent to ___ 6 ? D) 1 -
a
2

1 1
J3
A) J6 B) J3 C) .J2 D) 1 Ill
If 0 < x < 2 and 1 -cosx
-
7r
= 3
sin x 2 , what is the value of
-
2

.. cos x?
y A) -
9
II B) -
3
----+---- x 4
0 C) 9
III IV
D) 2
3
If sin () < O and sin () cos () < 0, then () must be in
which quadrant of the figure above?
A) I B) II C) III D) IV
442 McGraw-Hill Education: SAT

EXE RCISE SET 3 : TRIGONOMETRY ANSWE R KEY

No Calculator a negative times a positive i s a negative). Since cos () is


only positive in quadrants I and I V (because cosine cor­
1 . 7/2 o r 3.5 The discussion in Lesson 9 about the responds to the x-coordinates on the unit circle), () must
definition of the sine function and the unit circle made it b e i n quadrant IV
clear that the value of the sine function ranges from -1 to 1 .
Therefore, t h e maximum value of 8 sin 2X 1 is 6. B First, notice that alb a nd b/a are recipro­

(� - x)
8(l) ..!. 7_ or 3 . 5 . 2 2 cals. Next, we can use the identity in Lesson 1 0 that
_
=
2 2 2 sin x = cos to see that choice (B) i s just the
2 . 1 /36 or .027 or .028 An radian measure of 11/3 reciprocal of sin x. Alternately, we can just choose a value
is e quivalent to 60°. I f you haven't memorized the fact of x, like x = l, and evaluate sin 1 = 0.84 1 . The correct
that cos(60°) = V2, you can derive it from the Reference answer is the expression that gives a value equal to the
Information at the beginning of every SAT Math section, reciprocal of 0.841, which i s 1 /0.841 = 1 . 1 9 . Plugging
which includes the 30°-60°-90° special right triangle. i n x = 1 gives (A) 0.841,(B) (D)
1 . 19, (C) 0.292, 0 . 540.
Since a = V2, (a/3)2 = (1 /6)2 = 1 /36.
7. C Recall from the Pythagorean Identity that
3. 1 .1 7 (sin x - cos x)2 = 0.83 J
cos b = ± 1 - sin 2 b . Substituting sin b = a gives
FOIL: sin2 x - 2sin x cos x + cos2 x = 0.83 cos b = ± F"d2 . The angle b + 11 is the reflection of
Regroup: sin2 x + cos2 x - 2sin x cos x = 0.83 angle b t hrough the origin, so cos(b + 11) is the opposite
of cos b, which means that cos(b + 7r ) = ± �·
Simplify: 1 - 2sin x cos x = 0.83
Subtract 1 : -2sin x cos x = - 0. 17
Multiply by - 1 : 2sin x cos x = 0.17
Evaluate this expression: (sin x + cos x)2 8. D Recall from the Pythagorea n Identity that
sin2 x + 2sin x cos x + cos2 x cos2 x = 1 - sin2 x.
FOIL: COS X 3
Regroup: sin2 x + cos2 x + 2sin x cos x 2
1 - sin x 2
Substitute: 1 + 0. 17 = 1 . 17
Substitute cos2 x = 1 - sin2 x: COS X 3
4 . D sin(11/6) V2 and cos(11/3) V2, so sin(11/6)/ cos 2 x 2
cos( 11/3) = 1 . Cancel common factor: 1 3
COS X 2
5. D If sin () < 0, then () must be either in quadrant I I I Reciprocate: 2
COS X = -
or i n quadrant IV. (Remember that s i n e corresponds t o 3
the y-coordinates on t h e unit circle, so it is negative in
those quadrants where the y-coordinates are negative.)
If sin () cos () < 0, then cos () must be positive (because
CHAPTER 1 0 / THE SAT MATH: ADDITIONAL TOPICS 443

S ki l l 3 : U n dersta nding Complex N u m bers


Understanding
Lesson 1 1 : the imaginary number i a nd
the complex pla ne

Imaginary Numbers Given that i = � , which of the following is equal


to 2- ?
i=H i3
The imaginary number i is defined as the princi­ A)
pal square root of -1. B) -i
The square root of a negative number is not on C)
the real number line, because the square of a D) - 1
real number cannot be negative. Therefore, the
square roots of negative numbers must reside (Medium-hard) To answer this question, we just need to
on their own number line, which we call the know the basic exponent rules and the definition of i.
"imaginary axis," which is perpendicular to the
real axis, intersecting it at the origin. The plane Original expression:
defined by the real axis and the imaginary axis is Factor:
called the complex plane. i 2 x i'
Imaginary
Substitute i 2 = - 1

Multiply by iii:

Substitute i2 = - 1 : - (-i) = i
--+--+--+-- Real
-i 1 Therefore, the correct answer is (A).
444 McGraw-Hill Education: SAT

Lesson 1 2: Add i ng, m u ltiplyi ng, d ivid ing, a nd simplifyi ng


complex n u m bers

Complex Numbers Multiply numerator and


The sum of a real number and an imagi­ denominator ( 2 + i )( 3 + i )
nary number is called a complex num­ by conjugate of denominator: ( 3 - i )( 3 + i )
ber. All complex numbers can be expressed
in the form a + bi where a and b are real FOIL: 6 + 2i + 3i + i2
numbers and i � .= 9 + 3i - 3i - i2
Every complex number a + bi corresponds to Simplify: 5 + 5i
the point (a, b) on the complex plane. 10

Imaginary Distribute division: 1 1 .


- + -z
2 2
3i - - ' 2 + 3i Powers of i

The successive powers of i (i1, i2, i3, i4, i5, i6, (l )


2i
• • •

cycle counterclockwise around the unit circle in


...-+--+---+--- Real the complex plane.
-i 1 2

Adding Complex Numbers

To add complex numbers, just combine "like"


terms.
Original expression: (3 - 2i) + (2 + 6i)
We can verify this by expanding any positive
Regroup with Commutative integer power of i.
and Associative Laws of Expression to be
Addition: (3 + 2) + ( -2i + 6i)
evaluated: i13
Simplify: 5 + 4i
E�md: 00000000000000000000000000
Multiplying Complex Numbers
Group in pairs: (i i)(i i)(i i)(i i)(i i)(i i)(i)
x x x x x x

=
Substitute i2 - 1 : (- 1)(- 1)(- 1) (- 1)(- 1)(- l)(i)
To multiply complex numbers, just FOIL and Simplify:
combine like terms. =
This implies that in 1 if n is a multiple of 4.
Original expression: (3 - 2i)(2 + 6i)
= = =
(That is, i4 1, i8 1, i12 1, etc.) This gives us a
FOIL: 3 2 + 3 6i
( )( ) ( )( ) (+ -2i)(2) + ( -2i)(6i)
convenient way to simplify large powers of i: just
Simplify: 6 + 18i - 4i - 1 2 i2
replace the exponent with the remainder when it
Substitute i2 = - 1 : 6 + 18i - 4i - 1 2 ( - 1 )
= =
is divided by 4. For instance, i39 i3 -i, because
Combine like terms: 18 + 14i
3 is the remainder when 39 is divided by 4.
Dividing Complex Numbers

To divide complex numbers, express the quo­


tient as a fraction, multiply numerator and
denominator by the complex conjugate of the
denominator, and simplify. The complex conju­
gate of a + bi is a - bi.
Original expression: 2+i
3-i
CHAPTER 1 0 / THE SAT MATH: ADDITIONAL TOPICS 445

If 1 - i = __!5_, Which of the following is NOT equal to i6 - i2?


1 + i where i = �, which of the following A) is - i
is equal to K2? B) i4
A) 2i C) 2i3 + 2i
B) 4i
D) 1 + i6
C) 4 + i (Medium) Here, we have to use our knowledge about
powers of i. Since = (i i)(i i)(i i) = (-1)(-1)(-1) = -1,
i6 x x x
D) 4 and i2 = -1, the given expression, i6 - i2, is equal to

(Medium) Given equation: l - i= __!5_ (-1) - (-1) = 0. Simplifying each choice gives us
1+i A) is - i = i - i = 0
Multiply by 1 + i: (1 - i)(l + i) = K B) i4 = 1
FOIL: l + i - i - i2 = K C) 2i3 + 2i = -2i + 2i = 0
Simplify: l - (-1) = 2 = K D) l + i6 = 1 + (-1) = 0
Square: 4 = K2 Therefore, the correct answer is (B).
Therefore, the correct answer is (D).
446 McGraw-Hill Education: SAT

Exerc ise Set 4: Complex N u m be rs ( No Ca l c u la to r)

• ..
+
I f a bi = (1 +
2 i)(3 - 4i), where a and b are con­ Which of the following expressions is equal to
stants and i = H, what is the value of a + b? (2 + 2 i)2?

I I I I I
A) 0
B) 4i
C) Bi
D) 4 - 4i

4+i
If a + bi = , where a and b are constants and •
2-i
If B(3 + i) = 3 - i, what is the value of B?
i = H, what is the value of a?
-+-t
A) 3 4.

I I I I I
5 5
B) -4 + -3 1.
5 5
C) 3 4.
!

..
- - -

5 5
+ +
For what value of b does (b i)2 = 80 18i? D) 4 3 [.
- - -

5 5

I I I I I •
x2 + kx = - 6

• If one o f the solutions t o the equation above i s


x = 1 - i JS , what is the value o f k?
The solutions of the equation x2 - 2x + 1 5 = 0 are
A) - 4
x=a + and x = a -
ifb ifb,
where a and b are
positive numbers. What is the value of a + b?
B) -2
C) 2

I I I D) 4


• I f im = -i, which of the following CANNOT b e the
Given that i = H, which of the following is equal value of m?
A) 1 5
to _1_?
(1 + i) 2 B) 18

1 1. C) 19
A) ---l
2 2 D ) 27
B) 1 .
--l
2
C) 1 .
-l
2
- + -t
D) 1 1.
2 2
CHAPTER 1 0 I THE SAT MATH: ADDITIONAL TOPICS 447

EXE RCISE SET 4: COMPLEX N UMBERS ANSWE R KEY

No Calculator
1. 13 (1 + 2i)(3 - 4i) Substitute i2 = -1: 1
FOIL: (1)(3) + (l)(-4i) + (2i)(3) + (2i)(-4i) l + i + i + (-1)
Simplify: 3 - 4i + 6i - Bi2 Simplify:
2i
Substitute i2 = -1: 3 - 4i + 6i -B(-1)
Combine like terms: 11 + 2i Multiply by iii:
Therefore, a = 11 and b = 2, so a + b = 13.
Substitute i2 = -1: i 1. = l

2. 7/5 or 1 .4
4+i -2 2
- - -

2-i
Multiply conj ugate: ( 4 + i)(2 + i) 6. c (2 + 2i)2
(2 - i)(2 + i) FOIL: (2 + 2i)(2 + 2i) = 4 + 4i + 4i + 4i2
Substitute i2 = -1: 4 + Bi - 4 = Bi
FOIL:
B +4i + 2i + i2
4 +2i- 2i - i2 7. D B(3 + i) = 3 - i
B + 4i + 2i - 1 Divide by 3 + i: B=-- 3-i
Substitute i2 = -1: 4 +2i - 2i + 1 3+i
7 + 6i
Combine like terms:
56 . FOIL: B = 99 -- 3i3i -+ 3i3i +- i2i2
-----

7
Distribute division: -+-1
5 5 Substitute i2 = -1: B = 99 3i3i +- 3i3i +- (-1)
-

3. (b + i)2 - (-1)
B = 10 -5 3i
B - 6i 4
9
FOIL: (b + i)(b + i) = b2 + bi + bi + i2 Simplify: -- = --

Substitute i2 = -1: b2 + bi + bi - 1
Combine like terms: (b2 - 1) + 2bi
Since this must equal BO + 18i, we can find b by solving
Distribute division: B='!_-5 �5 i
either b2 - 1 = BO or 2b = 18. The solution to both equa­ B. B x2 + kx = -6
tions is b = 9. Add 6: x2 + kx + 6 = 0
4. The equation we are given is a quadratic equa­
15 Substitute x=l - ds: ( 1 - i-!5 ) 2 + k ( 1 - i-!5 ) + 6=0
tion in which a = l, b = -2, and c = 15. Therefore, we can
use the quadratic formula: FOIL: ( 1 - 2i-!5 + 5i2 ) + k ( l - i-!5 ) + 6=0
Quadratic Formula: - b ± Jb2 - 4ac Simplify: ( -4 - 2i-!5 ) + k ( l - i-!5 ) + 6 = 0
2a -4 - 2i-!5 + k - ik-!5 + 6 = 0
2 ± J(-2)2 - 4(1)(15)
Distribute:
Substitute:
2(1) Collect terms: (2 + k) - ( 2-JS + k-JS ) i=O
Simplify: 2 ± r-56 Therefore, both 2 + k = 0 and 2-JS + k -J5 = 0 . Solving
2
2 ± 2iJ14 either equation gives k = -2.
Simplify:
2 9. B As we discussed in Lesson 10, the powers of i are
Distribute division: l±iJ14 "cyclical," and im = -i if and only if is 3 more than a
m

Therefore, a = 1 and b = 14, so a + b = 15. multiple of 4. The only number among the choices that is
not 3 more than a multiple of 4 is (B) lB.
5. B 1
( 1 + i)2
FOIL:
(1 + i)(l + i) 1 + i + i + i2

You might also like